Revista (format .pdf, 6.5 MB)

104
Anul XIII, Nr. 1 Ianuarie Iunie 2011 RECREA Ţ II MATEMATICE REVIST Ă DE MATEMATIC Ă PENTRU ELEVI Ş I PROFESORI 1 = π i e Asocia ţ ia “Recrea ţ ii Matematice” IA Ş I - 2011

Transcript of Revista (format .pdf, 6.5 MB)

Page 1: Revista (format .pdf, 6.5 MB)

Anul XIII, Nr. 1 Ianuarie – Iunie 2011

RECREAŢII MATEMATICE

RE V IS TĂ DE M AT E M AT I CĂ P E NT R U E L E V I Ş I P R O F E S O R I

1−=πie

A s o c i a ţ i a “ R e c r e a ţ i i M a t e m a t i c e ” I AŞ I - 2 0 1 1

Page 2: Revista (format .pdf, 6.5 MB)

Semnificaţia formulei de pe copertă: Într-o formă concisă, formula 1−=πie leagă cele patru ramuri fundamentale

ale matematicii: ARITMETICA reprezentată de 1 GEOMETRIA reprezentată de π ALGEBRA reprezentată de i ANALIZA MATEMATICĂ reprezentată de e

Redacţia revistei :

Petru ASAFTEI, Dumitru BĂTINEŢU-GIURGIU (Bucureşti), Temistocle BÎRSAN, Dan BRÂNZEI, Alexandru CĂRĂUŞU, Constantin CHIRILĂ, Eugenia COHAL, Adrian CORDUNEANU, Mihai CRĂCIUN (Paşcani), Paraschiva GALIA, Paul GEORGESCU, Mihai HAIVAS, Gheorghe ILIE, Gheorghe IUREA, Lucian-Georges LĂDUNCĂ, Gabriel MÎRŞANU, Alexandru NEGRESCU (student, Iaşi), Gabriel POPA, Dan POPESCU (Suceava), Florin POPOVICI (Braşov), Maria RACU, Neculai ROMAN (Mirceşti), Ioan SĂCĂLEANU (Hârlău), Ioan ŞERDEAN (Orăştie), Dan TIBA (Bucureşti), Marian TETIVA (Bârlad), Lucian TUŢESCU (Craiova), Adrian ZANOSCHI, Titu ZVONARU (Comăneşti)

COPYRIGHT © 2011, ASOCIAŢIA “RECREAŢII MATEMATICE” Toate drepturile aparţin Asociaţiei “Recreaţii Matematice”. Reproducerea integrală sau

parţială a textului sau a ilustraţiilor din această revistă este posibilă numai cu acordul prealabil scris al acesteia. Se consideră că autorii materialelor trimise redacţiei revistei sunt, în mod implicit, de acord cu publicarea lor, îşi asumă responsabilitatea conţinutului lor şi cedează Asociaţiei “Recreaţii Matematice” dreptul de proprietate intelectuală asupra acestora.

TIPĂRITĂ LA BLUE SIM&Co IAŞI Bd. Carol I, nr. 3-5 Tel. 0332 111021, 0721 571705 E-mail: [email protected]

ISSN 1582 - 1765

Page 3: Revista (format .pdf, 6.5 MB)

Anul XIII, Nr. 1 Ianuarie – Iunie 2011

RECREAŢ I I MATEMATICE

RE V IS TĂ DE M AT E M AT I CĂ P E NT R U E L E V I Ş I P R O F E S O R I

1−=πie

Revistă cu apariţie semestrială

EDITURA “RECREAŢII MATEMATICE”

IAŞI - 2011

Page 4: Revista (format .pdf, 6.5 MB)

.

Page 5: Revista (format .pdf, 6.5 MB)

Un secol de excelentaa Invatamantului Superior Electrotehnic la Iasi

Pot scruta orizontul viitoruluidoar cei care au privilegiul ascederiipe un piedestal edificatdin munca si realizarile predecesorilor!

La 1 noiembrie 1910, ın cadrul Facultatii de Stiinte a Universitatii din Iasi, se des-chideau cursurile Scolii de Electricitate Industriala. Programa analitica ambi-tioasa si conditiile severe de admitere o certifica indubitabil drept prima unitate deınvatamant superior electrotehnic din Romania. Cel dintai director al acestui nucleugenerator de progres si dezvoltare a fost Profesorul Dragomir Hurmuzescu, doctorın fizica la Sorbona, cel care a inventat dielectrina si a realizat electroscopul ce-ipoarta numele.

Aceasta prioritate a revenit ın mod firesc Iasului, fiind consecinta a unor demersurivizionare ıntreprise, ın cel de-al patrulea mandat al sau, de primarul junimistNicolaeGane, ales mai apoi, ca si Dragomir Hurmuzescu, membru al Academiei Romane. Inanii 1895-1899, la Iasi, s-a inaugurat bijuteria numita Teatrul National (deservita depropriul generator de electricitate), Uzina Electrica Comunala si s-au finalizat studiilepentru utilizarea tramvaiului electric ın transportul public (prima linie, de la Gara laHala centrala, inaugurata ın 1900). Realizarea acestor obiective, ambitioase pentru

1

Page 6: Revista (format .pdf, 6.5 MB)

sfarsitul secolului XIX, a plasat orasul Iasi ın sfera civilizatiei europene, conferindu-iaura de citadela a progresului tehnic.

Putem spune acum, la ceas aniversar, ca Iasul si-a asigurat statutul de impor-tant centru universitar, Alma Mater Iasiensis, racordat la cercetare si nou, refuzandanatema ”sancta mediocritas” la care parea ca este condamnat de un plasament geo-politic mai putin generos.

Dezvoltarea ınvatamantului superior electrotehnic la Iasi este legata, pe ıntregparcursul ultimului secol, de vicisitudinile unei istorii mai putin blande cu oameniiacestor meleaguri:

Infiintarea ın 1938, dupa miscari greviste de amploare, a Scolii Politehnice”Gheorghe Asachi”, Facultatea de Electrotehnica avandu-l ca prim decan pe sa-vantul Stefan Procopiu, pe nedrept vaduvit de Premiul Nobel pentru descoperireamagnetonului ce-i poarta numele;

Mutarea ın 1942 la Cernauti, pentru a marca revenirea Bucovinei de Nord ıngranitele nationale;

Refugiul dureros la Turnu-Severin;Reforma ınvatamantului din 1948 si ınfiintarea Institutului Politehnic Iasi,

primul rector fiind Cezar Parteni Antoni, fost decan al facultatii;Marea industrializare din anii 1960-1970;In 1993 Institutul devineUniversitatea Tehnica ”Gheorghe Asachi”, marcand

racordarea ferma la standarde si norme de excelenta europeana.

Plasata pe impresionantul piedestal al celor 100 de ani de existenta, Facultateade Electrotehnica, redenumita din anul 2009 Facultatea de Inginerie Electrica, Ener-getica si Informatica Aplicata, se prezinta acum cu un corp profesoral de mare valoare,cu programe de studii de ınalt nivel, flexibil conectate la solicitarile venite din mediuleconomic si cu o baza materiala moderna, care pot sustine atat procesul formativ catsi demersul sine qua non al cercetarii stiintifice.

Puntea constituita ıntre trecutul si prezentul scolii iesene de inginerie electricacertifica o nobila si onoranta identitate care ne capaciteaza imaginatia si resurseleıntru proiectarea unui viitor fata de care avem obligatii responsabil asumate. Neasumam astazi ımpreuna, magistri si discipoli, mediu economic si mediu academic,comunitate si administratie locala, provocarile unui viitor pe care avem dreptul sa-lvisam, viitor care asteapta raspunsuri bune, emergente din rigoarea si inventivitateace au caracterizat dintotdeauna scoala ieseana de inginerie electrica.

Prof. dr. ing. Alexandru SALCEANUDecan al Facultatii de Inginerie Electrica,

Energetica si Informatica Aplicata

2

Page 7: Revista (format .pdf, 6.5 MB)

Restaurarea mormantului lui C. Climescu,fost rector al Universitatii din Iasi

Un gest de pios si respectuos omagiu

In seria evenimentelor Iasului, care s-au succedat pe parcursul anului 2010, marcamdoar trei iesite din comun ca importanta si semnificatie: Centenarul SeminaruluiMatematic ”Al. Myller” - sarbatorit ın cadrul unei conferinte, 21-26 iunie -, 150de ani de existenta a Universitatii ”Alexandru Ioan Cuza” - sarbatoriti ıncadrul Dies Academici, 20-30 octombrie - si Centenarul Invatamantului Elec-trotehnic Superior la Iasi - 25-30 octombrie.

Programele acestor manifestari au cuprins: conferinte, expozitii, vernisajulMuzeu-lui Universitatii, dezvelirea busturilor unor ilustri profesori, programe artistice etc.Iasul a fost gazda unor personalitati din domeniul stiintelor sau artelor sau tehnicii,din tara sau din strainatate, carora li s-a acordat titlul de Doctor Honoris Causa sicare au conferentiat ın aulele iesene. Pe linie de matematica, acest titlu a fost acordatprof. univ. dr. Haım Brezis, Universitatea Pierre et Marie Curie-Paris, Franta.

In ziua de 24 octombrie, o delegatie a Senatului Universitatii din Iasi, condusade rectorul Vasile Isan, a depus coroane de flori pe mormintele din cimitirul Eter-nitatea ale unor fosti rectori ai Universitatii, printre care se numara si urmatoriirectori-matematicieni: Nicolae Culianu (rector ın perioada 1880-1898), CostantinClimescu (1901-1907), Alexandru Myller (1944-1945) si Ion Creanga (1955-1972). Primii doi fac parte din generatia de dascali care a pus temeliile ınvatamantuluiromanesc modern si au deschis stiintei romanesti drumul catre cercetarea originala.Sacrificand posibilitatile proprii de afirmare stiintifica, si-au orientat activitatea ındirectia predarii si publicarii de cursuri universitare sau alcatuirii de manuale desti-nate ınvatamantului secundar. Numele lor este legat de revista Recreatii stiintifice(1883 - 1888), prima revista stiintifica din tara ce se adreseaza tineretului, cu uncontinut predominant matematic si care ın dictionarele de periodice romanesti estementionata ca avand o contributie importanta ın pregatirea matematica a tineretuluistudios. Cu modestie si clarviziune ın nr. 1(1888) al revistei se spune ın cuvinte sim-ple si frumoase: Credem ca noi am tras cea ıntai brazda care conduce catra lucrarioriginale. Brazda ′i mica si ıngusta, dar exista!

Constantin Climescu (30 nov. 1844 – 6 aug. 1926) a fost nu numai membrufondator, ci si principalul realizator al acestei reviste. A publicat ın paginile ei ar-ticole din toate ramurile matematicii: aritmetica, geometrie elementara si analitica,algebra, analiza matematica. S-a nascut la Bacau, a urmat Facultatea de Stiintea Universitatii din Iasi si apoi, ın 1870, obtine licenta ın matematici si licenta ınstiinte fizice la Sorbona. (De mentionat faptul ca el si I. Vargolici sunt primii romaniprimiti la Sorbona.) Revenit ın tara, este numit la Catedra de geometrie analiticasi trigonometrie sferica a Facultatii de Stiinte din Iasi, la data de 22 sept. 1871, sidetine aceasta functie pana la pensionare, 1 aprilie 1909.

A fost un excelent profesor. Multi ani a fost decan al Facultatii de Stiinte, iar apoia fost rector al Universitatii din Iasi. Cursul sau de Geometrie analitica. Sectiuniconice, publicat ın 1898, este a doua carte de acest fel aparuta ın tara. A predat sialte cursuri ın cadrul facultatii si a functionat si la alte institutii de ınvatamant din

3

Page 8: Revista (format .pdf, 6.5 MB)

Iasi. Este autor al mai multor manuale de algebra si geometrie pentru uzul elevilordin gimnazii, licee si al candidatilor la bacalaureat. Publica ın vol. XXII si XXIIIale Gazetei Matematice (iar ın 1921, separat, ıntr-o brosura) un studiu asupra unuimanuscris vechi, Curs ıntreg de matematica curata, scris ın chirilice.

In 1892 este ales membru corespondent al Academiei Romane. A fost membru ıncolegiul de redactie al Gazetei Matematice.

Constantin Climescu a fost deputat de Bacau ın 1884 si senator de Bacau dela 1889 la 1910. Statul roman i-a conferit distinctiile: Ofiter al Ordinului SteauaRomaniei si Comandor al Ordinului Coroana Romaniei.

** *

Asociatia ”Recreatii Matematice” publica ıncepand cu anul 1999 revista RecreatiiMatematice ca o continuare peste timp a vechii reviste Recreatii Stiintifice.Constantin Climescu, sustinatorul principal al Recreatiilor Stiintifice, este veneratde membrii asociatiei ca strabun spiritual al lor. Pentru cinstirea sa si a strabuneireviste, ın anul 2008, la ımplinirea a 125 de ani de la aparitia primului numar deRecreatii Stiintifice, a fost realizata prin scanare reeditarea colectiei complete a aces-tei reviste (care se poate accesa liber la adresa www.recreatiistiintifice.ro).In toamna anului trecut,Asociatia ”Recreatii Mate-matice” a avut initiativarestaurarii mormantului luiConstantin Climescu -omagiu si semn de recunos-tinta adus acestuia. Aso-ciatia a asigurat resurselebanesti necesare si a suprave-ghiat executarea lucrarilorde restaurare ca o modes-ta contributie prilejuita deımplinirea a 150 de ani deexistenta a Universitatii dinIasi. La 16 oct. 2010, a fostıncheiata restaurarea mormantului lui C. Climescu. O delegatie a Senatului Univer-sitatii a depus, pe data de 24 octombrie, o coroana de flori la mormantul proaspatrestaurat. In organizarea Asociatiei ”Recreatii Matematice” a avut loc pe data de 7noiembrie sfintirea mormantului restaurat, cu participarea acad. Radu Miron, prof.univ. O. Carja - decanul Facultatii de Matematica, prof. univ. V. Oproiu - directorulSeminarului Matematic ”Al. Myller”, prof. univ. D. Branzei - presedintele filialeiIasi a Societatii de Stiinte Matematice, altor personalitati din ınvatamantul superioriesean, precum si a unor distinsi profesori din ınvatamantul preuniversitar.

Prof. univ. Temistocle BIRSANPresedintele Asociatiei ”Recreatii Matematice”

4

Page 9: Revista (format .pdf, 6.5 MB)

Conditii suficiente pentru inele Boole

Dorel MIHET1

Abstract. The main result of this Note is Proposition 1 which provides a set of sufficientconditions such that a ring is a Boolean ring.

Keywords: ring, Boolean ring.

MSC 2000: 06E20.

Un inel Boole este un inel (A,+, ·) ın care x2 = x pentru orice x ∈ A. In carteaMatematica pentru grupele de performanta - Clasa a XII-a [3] sunt prezentate douaconditii suficiente pentru ca un inel A cu proprietatea x2m+2n = x ∀x ∈ A (m,n ∈N∗) sa fie inel Boole. Rezultatul principal al acestei note este Propozitia 1, din carese pot obtine si alte caracterizari ale acestor inele.

In demonstratia Propozitiei 1 folosim urmatoarele doua leme:

Lema 1. Fie (A,+, ·) un inel, iar m,n ∈ N∗. Daca x2m+2n = x ∀x ∈ A, atunci

x2m+1

= x ∀x ∈ A si x2n+1

= x ∀x ∈ A.Demonstratie. Din (−1)2

m+2n = −1 rezulta 1 = −1. Fie x un element arbitraral lui A. Tinand seama de faptul ca 2y = 0 ∀y ∈ A, se demonstreaza imediat prin

inductie ca (x+ 1)2k

= x2k + 1 pentru orice k ∈ N.Asadar au loc egalitatile:

x+ 1 = (x+ 1)2m+2n = (x+ 1)2

m

(x+ 1)2n

=

(x2m + 1)(x2n + 1) = x2m+2n + x2n + x2m + 1 = x+ x2m + x2n + 1,

de unde deducem ca x2m = x2n si atunci (x2m)2 = (x2n)2 = x2m+2n , adica x2m+1

=

x2n+1

= x.

Lema 2. Fie (A,+, ·) un inel, m,n ∈ N∗ si x ∈ A, astfel ıncat xm = x si xn = x.Atunci x(m−1,n−1)+1 = x.

Demonstratie. Se arata imediat prin inductie ca xk(m−1)+1 = x si xl(n−1)+1 = xpentru orice k, l ∈ N. Daca (m − 1, n − 1) = d, atunci exista k, l ∈ N astfel ıncatk(m− 1)− l(n− 1) = d, deci x = xk(m−1)+1 = xd+l(n−1)+1 = xd ·xl(n−1)+1 = xd ·x =xd+1.

Propozitia 1. Fie (A,+, ·) un inel, m,n ∈ N∗, iar d := (m + 1, n + 1). Daca

x2m+2n = x ∀x ∈ A, atunci x2d = x ∀x ∈ A.Demonstratie. In conformitate cu Lema 1, pentru orice x ∈ A au loc egalitatile

x2m+1

= x si x2n+1

= x. Din Lema 2 deducem ca

x(2n+1−1, 2m+1−1)+1 = x ∀ x ∈ A

1Prof. dr., Departamentul de Matematica, Universitatea de Vest, Timisoara

5

Page 10: Revista (format .pdf, 6.5 MB)

si acum concluzia rezulta dintr-un rezultat clasic de teoria numerelor:

(2p − 1, 2q − 1) = 2(p,q) − 1.

Cazuri particulare

1) Deoarece (n+1, n+2) = 1, din Propozitia 1 rezulta ca un inel A cu proprietatea

x2n+2n+1

= x ∀x ∈ A (n ∈ N∗)

este boolean. Mai general, daca m si q sunt doua numere ıntregi pozitive fixate, iar

x2q(m+1)+2m = x ∀x ∈ A,

atunci A este inel boolean ([4], Prop. 3).

In particular, orice inel A cu proprietatea

x4n+2 = x ∀x ∈ A

este boolean.2) ([1, Theorem A], [2]) Fie m,n ∈ N, m > n astfel ıncat numarul p = 2m+2n−1

este prim impar. Daca A este un inel cu proprietatea x2m+2n = x ∀x ∈ A, atunci Aeste boolean.

Demonstratie. Remarcam mai ıntai ca m si n sunt pozitive. Fie d = (m+1, n+1). Vom arata ca d = 1.

Din p+ 1 = 2m + 2n rezulta ca (2m+1 − 1) + (2n+1 − 1) = 2p, deci 2d − 1 este undivizor impar al lui 2p. Prin urmare 2d−1 | p si cum 2d−1 ≤ 2n+1−1 < 2m+2n−1 = p,rezulta ca 2d − 1 = 1, deci d = 1.

3) ([4], Prop. 4) Fie m, q ∈ N∗, iar r ∈ N, r < m + 1. Daca A este un inel cu

proprietatea x2q(m+1)+r+2m = x ∀x ∈ A, atunci x2r+1

= x ∀x ∈ A.Demonstratie. Fie d cel mai mare divizor comun al numerelor q(m+1)+r+1 si

m+1. Din Propozitia 1, x2d = x pentru orice x ∈ A si se demonstreaza imediat prin

inductie dupa l ca x2ld = x, pentru orice l ∈ N∗ si x ∈ A (pentru pasul de inductie

observam ca x2ld = x ⇒ (x2ld)2d

= x2d = x, iar (x2ld)2d

= x2ld·2d = x2ld+d

= x2(l+1)d

).Deoarece (q(m + 1) + r + 1,m + 1) = (r + 1,m + 1), d divide numarul r + 1, iar

daca r + 1 = kd, din cele de mai sus rezulta ca x2r+1

= x2kd

= x ∀x ∈ A, ceea cetrebuia demonstrat.

Bibliografie

1. R. Ayoub, C. Ayoub -On the Commutativity of Rings, The American MathematicalMonthly, Vol. 71 (3) (1964), 267-271.

2. D. Isac - Conditii suficiente pentru inele Boole, GMB 3/1995, 541-544.

3. V. Pop, V. Lupsor (coordonatori) - Matematica pentru grupele de performantaClasa a XII-a, Ed. Dacia Educational, 2004.

4. J.-S. Shiue, W.-M Chao - On the boolean rings, Yokohama Math. J. 24 (1976),93-96.

6

Page 11: Revista (format .pdf, 6.5 MB)

Asupra calcularii unor integrale definiteMihai DICU 1

Abstract. The aim of this paper is to calculate the integrals of type (1), when the constantsm,n, p, q satisfy certain conditions.

Keywords: parallelogram identity, homographic function.

MSC 2000: 26A42.

Scopul propus este calcularea unor integrale de forma

(1) I =

Z b

a

ln(mx+ n)

x2 + px+ qdx

folosind schimbarea de variabila x = φ(t), unde φ este functia omografica

(2) φ : [a, b] → [a, b], φ(t) =αt+ β

γt+ δ(αδ − βγ = 0 si γt+ δ = 0, ∀t ∈ [a, b]).

Cazul particular p = 0 este tratat ın [1]; preocupari asemanatoare gasim ın [2]. Vorfi impuse anumite conditii constantelor m,n, p, q si coeficientilor functiei φ, unelenaturale si altele menite sa asigure derularea etapelor caculului.

Consideram m = 0, n = 0, p = 0, δ = 0 si cerem functiei φ sa ındeplineascaconditiile: φ(a) = b si φ(b) = a. Urmeaza ca αa+ β = γab+ δb si αb+ β = γab+ δa,

de unde, prin scadere, obtinem α + δ = 0. Notand u =β

δsi v =

γ

δ, gasim ca φ are

forma:

(3) φ(t) =u− t

1 + vt, t ∈ [a, b],

iar a, b, u, v sunt legate prin egalitatea

(4) a+ b+ abv = u.

Observam ca 1 + vt = 0 pentru t ∈ [a, b], 1 + uv = 0 si ca φ′(t) = − 1 + uv

(1 + vt)2.

Efectuand schimbarea de variabila x = φ(t) cu φ data de (3), vom avea

(5) I = −(1+uv)

Z a

b

ln[(nv −m)t+ (mu+ n)]− ln(1 + vt)

(1− pv + qv2)t2 + (−p− 2u+ puv + 2qv)t+ (q + pu+ u2)dt.

Pentru reducere la integrala initiala impunem conditiile:

(6) nv −m = 0 si

(7) 1− pv + qv2 =−p− 2u+ puv + 2qv

p=

q + pu+ u2

q= k,

1Profesor, Colegiul National ”Fratii Buzesti”, Craiova

7

Page 12: Revista (format .pdf, 6.5 MB)

care asigura eliminarea unui termen la numarator si coincidenta numitorului (pana lafactorul k) cu numitorul din integrala (1).

Tinand seama de (6) si (7), relatia (5) se scrie

I =1 + uv

k

�Z b

a

ln(mu+ n)

t2 + pt+ qdt−

Z b

a

ln(1 + vt)

t2 + pt+ qdt

�.

Cum am presupus n = 0, din (6), (4) si (7) obtinem pentru u, v, k expresiile:

(8) v =m

n, u = mab+ n(a+ b), k =

1

n2(qm2 − pmn+ n2).

Inlocuind ın a doua integrala din paranteza patrata pe v cu expresia data de primarelatie din (8), obtinem succesiv:

I =1 + uv

k

�Z b

a

ln(mu+ n) + lnn

t2 + pt+ qdt− I

�,

(9) I =1 + uv

1 + k + uvlnn(mu+ n)

Z b

a

dx

x2 + px+ q

(se considera n > 0 si mu+ n > 0).

Calculul integralei din membrul doi fiind cunoscut, rezulta ca formula (9) rezolvaproblema pusa. Indeplinirea conditiilor (7) cu u, v, k dati de (8) limiteaza eficientaacestei formule.

In continuare, vom urmari detaliat calculul integralei I, discutand dupa valorilelui q.

I. Cazul q = 0. Conditiile (7) revin la relatiile:

(7′) u(p+ u) = 0, p(2− pv − uv) = −2u.

Daca u = 0, gasim v =2

psi din (4) rezulta ca a + b + abv = 0; deci p =

− 2ab

a+ b. Numitorul din integrala, g(x) = x2 + px, x ∈ [a, b], se anuleaza ın interiorul

intervalului: g(a)g(b) = (a2+pa)(b2+pb) = ab(p+a)(p+ b) = − a2b2

(a+ b)2(a− b)2 < 0.

In acest caz integrala nu are sens.

Daca u = −p, este verificata si a doua relatie din (7′). Constatam cu usurinta caavem k = 1 + uv, iar din (9) obtinem ın acest caz

I =1

2lnn(n−mp)

Z b

a

dx

x2 + px(n > 0, n−mp > 0).

8

Page 13: Revista (format .pdf, 6.5 MB)

Exemplu.

Z −2

−3

ln(x+ 6)

x2 + 4xdx =

1

2ln 12

Z −2

−3

dx

x2 + 4xetc. (cu formula precedenta

sau direct cu schimbarea de variabila x = −6t+ 4

t+ 6).

II. Cazul q = 0. Conditiile (7) conduc la sistemul

(7′′) q(1− pv + qv2) = q + pu+ u2, p(1− pv + qv2) = −p− 2u+ puv + 2qv.

Inmultind prima relatie cu −p si pe a doua cu q si apoi adunand, obtinem

(pu+ 2q)(p+ u− qv) = 0.

II.1. Daca u = −2q

p, ınlocuind ın prima relatie din (7′′) obtinem ecuatia ın v:

(pv − 2)(pqv − p2 + 2q) = 0 si avem v =2

psau v =

p2 − 2q

pq.

In cazul u = −2q

psi v =

2

p

(6)=

m

n, conditia (4) se scrie p(a + b) + 2ab = −2q.

Notand ∆ = p2 − 4q (discriminantul trinomului x2 + px + q), prin calcule de rutinase obtin egalitatile:

(a2 + pa+ q)(b2 + pb+ q) = −∆

4(a− b)2, (ma+ n)(mb+ n) =

m2

4∆.

Daca ∆ > 0, atunci prima spune ca trinomul se anuleaza ın intervalul [a, b] de in-tegrare; daca ∆ < 0, a doua spune ca binomul mx + n are aceeasi proprietate. Inambele cazuri, integrala I nu are sens. Daca ınsa ∆ = 0, I se calculeaza usor prinparti.

In cazul u = −2q

psi v =

p2 − 2q

pq=

m

n, prin calcul obtinem 1 + uv = k = −∆

p2si

n(mu + n) = −n2

p2∆. Integralele I pentru care ∆ = p2 − 4q < 0 sunt date, conform

formulei (9), de

I =1

2ln

�−n2

p2∆

�Z b

a

dx

x2 + px+ q.

Exemplu.

Z 2

0

ln(x+ 1)

x2 − x+ 1dx =

1

2ln 3

Z 2

0

dx

x2 − x+ 1(cu formula precedenta ın

care luam m = n = −p = q = 1, v = 1, u = 2 sau folosind schimbarea x =2− t

1 + t).

II.2. Daca v =p+ u

q

(6)=

m

n, obtinem 1 + uv = k =

1

n2(n2 − mnp + m2q) si

n(mu+ n) = n2 −mnp+m2q. Ca urmare,

I =1

2ln(n2 −mnp+m2q)

Z b

0

dx

x2 + px+ q.

9

Page 14: Revista (format .pdf, 6.5 MB)

Exemplu.

Z 2

1

ln(3− x)

x2 − x− 4dx =

1

2ln 2

Z 2

1

dx

x2 − x− 4(se utilizeaza formula prece-

denta sau schimbarea de variabila x =7− 3t

3− t).

Observatie. Daca n = 0 (caz exclus mai sus), vom calcula integrala I =Z b

a

lnx

x2 + px+ qdx (0 < a < b) efectuand o schimbare de variabila x = φ(t) cu o

functie de forma φ(t) =α

t. Din φ(a) = b si φ(b) = a gasim α = ab, iar din conditiile

de proportionalitate avem q = ab. Se obtine

I =1

2ln ab

Z b

a

dx

x2 + px+ q.

In concluzie, pentru a calcula integrale de tipul (1) putem proceda astfel:

(i) cautam schimbarea de variabila x = φ(t) cu φ : [a, b] → [a, b] si φ(t) =αt+ β

mt+ n;

(ii) impunem conditiile φ(a) = b si φ(b) = a si determinam α si β;(iii) daca α si β gasiti ındeplinesc si celelalte conditii puse ın evidenta mai sus ın

fiecare caz ın parte, atunci schimbarea de variabila este eficace si calculul integraleieste posibil ın acest mod.

Bibliografie

1. M. Dicu - O generalizare a unei integrale, G.M.-2/2000, 74-76.

2. T. Tamaian - O metoda pentru calculul unor integrale, G.M.-2-2004, 63-66.

Reconstituiti adunarea:

* * ** * *

* * *

ın care trebuie sa folositi toate cifrele de la 1 la 9 o singura data, iar rezultatul sa fie

format din cifre consecutive. Cate posibilitati exista?

Neculai Stanciu, Buzau

(continuare la pag. 22)

10

Page 15: Revista (format .pdf, 6.5 MB)

O identitate cu numere complexe si consecintelesale geometrice

Nicolae BOURBACUT1

Abstract. In this paper one proves the relation (2.1), and one shows a few particular cases ofthis relation.

Keywords: parallelogram identity, Leibniz relation, Euler inequality.

MSC 2000: 97D50.

1. Introducere. Desi reprezinta o notiune cu caracter algebric, numerele com-plexe au devenit un instrument extrem de util si ın studiul geometriei. Multe relatiiın care intervin numerele complexe au primit interpretari geometrice care au condusla rezultate foarte interesante. Una dintre cele mai cunoscute este urmatoarea:

(1.1) |z + w|2 + |z − w|2 = 2 |z|2 + 2 |w|2 ,

valabila pentru orice z, w ∈ C. Relatia, cunoscuta sub numele identitatea parale-logramului, a primit ulterior o generalizare, cunoscuta sub numele relatia Leibniz-Lagrange, care se poate gasi ın [4], [7] sau [8].

In acest articol vom demonstra ca aceasta generalizare are loc ın conditii mai slabe.Ca o consecinta vom obtine demonstratii nu foarte dificile pentru cateva identitatisau inegalitati geometrice mai mult sau mai putin cunoscute.

2. Identitati cu numere complexe. Acest paragraf este rezervat rezultatelorlegate de numerele complexe.

Propozitia 2.1. Fie n ∈ N, n > 2 si numerele reale a1, a2, . . . , an cu proprietateaa1+a2+ . . .+an = 1. Pentru orice numere complexe z1, z2, . . . , zn are loc identitatea:

(2.1) |a1z1 + a2z2 + . . .+ anzn|2 =nX

k=1

ak |zk|2 −X

16k<l6n

akal |zk − zl|2.

Demonstratie. Avem:

|a1z1 + a2z2 + . . .+ anzn|2 +X

16k<l6n

akal |zk − zl|2 =

= (a1z1 + a2z2 + . . .+ anzn) (a1z1 + a2z2 + . . .+ anzn)+

+X

16k<l6n

akal (zk − zl) (zk − zl) =nX

k=1

a2kzkzk +X

16k<l6n

akal (zkzl + zlzk)+

+nX

k=1

ak

�nX

l=1,l =k

alzkzk

�−

X16k<l6n

akal (zkzl +l zk) +nX

k=1

ak

nX

l=1

al |zk|2!

=

=nX

k=1

ak |zk|2, ceea ce ıncheie demonstratia.

1Sarmizegetusa, Romania

11

Page 16: Revista (format .pdf, 6.5 MB)

Corolarul 2.2. Fie n ∈ N , n > 2 si numerele reale a1, a2, . . . , an cu proprietateaa1 + a2 + . . .+ an = 1 . Atunci pentru orice numere complexe z, z1, z2, . . . , zn are locidentitatea:

(2.2) |z − a1z1 − a2z2 − . . .− anzn|2 =nX

k=1

ak |z − zk|2 −X

16k<l6n

akal |zk − zl|2.

Demonstratie. Se aplica 2.1 pentru numerele complexe z−z1, z−z2, . . . , z−zn.

Corolarul 2.3. Fie n ∈ N, n > 2. Atunci pentru orice numere complexez, z1, z2, . . . , zn are loc identitatea:

(2.3)

����z − z1 + z2 + . . .+ znn

����2 =1

n

nXk=1

|z − zk|2 −1

n2

X16k<l6n

|zk − zl|2.

Demonstratie. Se aplica 2.2 pentru numerele a1 = a2 = . . . = an =1

n.

Corolarul 2.4. Fie n ∈ N , n > 2 si numerele reale b1, b2, . . . , bn . Atunci pentruorice numere complexe z1, z2, . . . , zn are loc identitatea:

(2.4)

|b1z1 + b2z2 + . . .+ bnzn|2 = (b1 + b2 + . . .+ bn)nX

k=1

bk |zk|2

−X

16k<l6n

bkbl |zk − zl|2 .

Demonstratie. Fie S =nX

k=1

bk. Daca S = 0 atunci rezultatul este consecinta

directa a aplicarii Propozitiei 2.1 pentru numerele a1 =b1S, a2 =

b2S, . . . , an =

bnS.

Daca S = 0 se verifica printr-un calcul direct.Din Corolarul 2.4 obtinem fara dificultate urmatoarele rezultate:

Corolarul 2.5. Fie n ∈ N , n > 2 si numerele reale pozitive b1, b2, . . . , bn. Atuncipentru orice numere complexe z1, z2, . . . , zn are loc inegalitatea:

(2.5) |b1z1 + b2z2 + . . .+ bnzn|2 6 (b1 + b2 + . . .+ bn)nX

k=1

bk |zk|2.

Corolarul 2.6. Fie n ∈ N, n > 2 si numerele reale pozitive b1, b2, . . . , bn . Atuncipentru orice numere complexe z1, z2, . . . , zn are loc inegalitatea:

(2.6)X

16k<l6n

bkbl |zk − zl|2 6 (b1 + b2 + . . .+ bn)nX

k=1

bk |zk|2.

Daca ın (2.3) alegem n = 2 si z = 0 obtinem chiar identitatea paralelogramului.Daca alegem n = 3 obtinem

1

9|z1 + z2 + z3|2 =

1

3

�|z1|2 + |z2|2 + |z3|2

�− 1

9

�|z1 − z2|2 + |z1 − z3|2 + |z2 − z3|2

�,

12

Page 17: Revista (format .pdf, 6.5 MB)

care combinata cu (1.1) conduce la

(2.7) |z1|2 + |z2|2 + |z3|2 + |z1 + z2 + z3|2 = |z1 + z2|2 + |z1 + z3|2 + |z2 + z3|2 ,

cunoscuta sub numele de identitatea lui Hlawka.

3. Identitati geometrice. Relatiile din paragraful anterior au consecinte geo-metrice deosebite. Pe tot parcursul acestui paragraf vom nota cu zA afixul punctuluiA si analoagele, iar pentru elementele unui triunghi vom folosi notatiile uzuale.

Fie punctele A,B,C necoliniare. Se cunoaste ca pentru orice punct P din planultriunghiului ABC exista si sunt unice numerele reale α, β, γ cu α+ β + γ = 1, astfelıncat zP = αzA + βzB + γzC (vezi [1]). Pentru orice alt punct M din acelasi planavem

(3.1) MP 2 = αMA2 + βMB2 + γMC2 − αβAB2 − αγAC2 − βγBC2,

ca o consecinta a relatiei (2.2). Relatia (3.1) admite diferite particularizari:

Pentru α = β = γ =1

3, obtinem P = G si

MG2 =1

3

�MA2 +MB2 +MC2

�− 1

9

�a2 + b2 + c2

�,

adica relatia lui Leibniz. Daca ın plus alegem M = O obtinem

OG2 = R2 − 1

9

�a2 + b2 + c2

�,

care conduce apoi la inegalitatea a2 + b2 + c2 6 9R2.

Pentru α =a

a+ b+ c, β =

b

a+ b+ c, γ =

c

a+ b+ cobtinem P = I si

MI2 =aMA2 + bMB2 +MC2 − abc

a+ b+ c.

Pentru M = O si folosind abc = 4RS si S = pr obtinem

OI2 = R2 − 2Rr,

care apoi conduce la R > 2r, adica inegalitatea lui Euler.

Pentru α =−a

−a+ b+ c, β =

b

−a+ b+ c, γ =

c

−a+ b+ c, obtinem P = IA,

adica centrul cercului exınscris corespunzator laturii BC si

MI2A =−aMA2 + bMB2 + cMC2 + abc

−a+ b+ c.

Pentru M = O obtinem

OI2A = R2 +abc

−a+ b+ c.

13

Page 18: Revista (format .pdf, 6.5 MB)

Pentru un punct P ∈ (BC), cu β =BP

BC, γ =

CP

BC, α = 0 si M = A obtinem

AB2 · PC −AP 2 ·BC +AC2 · PB = PB · PC ·BC,

adica relatia lui Stewart.Fie acum patrulaterul ABCD si E,F mijloacele diagonalelor AC respectiv BD.

In (2.4) alegem n = 4, b1 = b2 =1

2, b3 = b4 = −1

2, z1 = zA, z2 = zC , z3 = zB ,

z4 = zD si obtinem

EF 2 =1

4

�AB2 +BC2 + CD2 +DA2 −AC2 −BD2

�,

adica relatia lui Euler ın patrulater.In partea a doua a acestui paragraf vom prezenta cateva rezultate general valabile

ın orice poligon cu n laturi.

Propozitia 3.1. Daca M este un punct oarecare ın planul poligonului A1A2 . . . An,iar G este centrul sau de greutate, atunci are loc relatia:

MG2 =1

n

nXk=1

MA2k − 1

n2

X16i<j6n

AiA2j .

Demonstratie. Este interpretarea geometrica a relatiei (2.3).

Corolarul 3.2. Daca A1A2 . . . An este un patrulater ınscris ıntr-un cerc de razaR, atunci are loc inegalitatea: X

16i<j6n

AiA2j 6 n2R2.

Demonstratie. Din 3.1. obtinemnX

k=1

MA2k − 1

n

X16i<j6n

AiA2j > 0 si apoi con-

cluzia prin alegerea lui M ca fiind centrul cercului.

Propozitia 3.3. Daca M,N sunt doua puncte din planul poligonului A1A2 . . . An

si a1, a2, · · · , an , respectiv b1, b2, · · · , bn numere reale astfel ıncat a1+a2+· · ·+an = 1,

b1 + b2 + · · ·+ bn = 1 , zM =nX

k=1

akzk si zN =nX

k=1

bkzk, atunci are loc relatia

MN2 = −X

16i<j6n

(bj − aj) (bi − ai) |zi − zj |2,

unde zk reprezinta afixul varfului Ak al poligonului.Demonstratie. Avem

MN2 = |zM − zN |2 =

�����zM −nX

k=1

bkzk

�����2 =nX

k=1

bk |zM − zk|2 −X

16i<j6n

bibj |zi − zj |2.

14

Page 19: Revista (format .pdf, 6.5 MB)

Dar |zM − zk|2 =

����� nXi=1

aizi − zk

�����2 =nX

i=1

ai |zk − zi|2 −X

16i<j6n

aiaj |zi − zj |2. Atunci

obtinem

MN2 =nX

k=1

bk

nXi=1

ai |zk − zi|2−nX

k=1

bkX

16i<j6n

aiaj |zi − zj |2−X

16i<j6n

bibj |zi − zj |2.

DarnX

k=1

bk = 1, deci MN2 =X

16i<j6n

(aibj + ajbi − aiaj − bibj) |zi − zj |2, ceea ce este

echivalent cu concluzia.

4. Inegalitati geometrice. Identitatile anterioare conduc la cateva inegalitatiremarcabile, dintre care o parte sunt mentionate ın continuare.

Propozitia 4.1. Intr-un plan consideram punctele distincte A1, A2, · · · , An sinumerele reale β1, β2, · · · , βn. Pentru orice punct M din plan are loc inegalitatea:

(4.1)

nX

k=1

βk

! nX

k=1

βkMA2k

!>

X16k<l6n

βkβlAkA2j .

Demonstratie. Se aplica (2.6) pentru numerele complexe z−z1, z−z2, · · · , z−znunde z,z1, z2, · · · , zn, reprezinta afixele punctelor M , A1, · · · , An.

In cazul particular al unui triunghi ABC si al numerelor reale α, β, γ, (4.1) devine

(4.2) (α+ β + γ)�αMA2 + βMB2 + γMC2

�> αβAB2 + αγAC2 + βγBC2.

Corolarul 4.2. Pentru orice punct M din planul triunghiului ABC are loc ine-galitatea

aMA2 + bMB2 + cMC2 > abc.

Demonstratie. Se aplica (4.2) pentru numerele reale a, b, c. O alta solutie segaseste ın [2].

Corolarul 4.3. Pentru orice punct M din planul triunghiului ABC are loc relatia

aMB ·MC + bMA ·MC + cMA ·MB > abc.

Demonstratie. Se aplica (4.2) pentru numerele reale α =a

MA, β =

b

MB,

γ =c

MCsi obtinem

�a

MA+

b

MB+

c

MC

�(aMA+ bMB + cMC) > abc2

MA ·MB+

ab2c

MA ·MC+

a2bc

MB ·MC, adica

�a

MA+

b

MB+

c

MC

�(aMA+ bMB + cMC) >

abc

MA ·MB ·MC(aMA+ bMB + cMC) , ceea ce conduce la concluzie. O alta solutie

se gaseste ın [2].

15

Page 20: Revista (format .pdf, 6.5 MB)

Corolarul 4.4(Weitzenbock). In orice triunghi are loc relatia

a2 + b2 + c2 > 4S√3.

Demonstratie. In (4.2) alegem α =a2

a2 + b2 + c2, β =

b2

a2 + b2 + c2, γ =

c2

a2 + b2 + c2si M = O, obtinem R2 > 3a2b2c2

(a2 + b2 + c2)2 , adica a2 + b2 + c2 > abc

√3

Rsi

concluzia.

Bibliografie

1. T. Andreescu, D. Andrica - Complex Numbers from A to Z, Birkhauser, Boston,2006.

2. T. Andreescu, D. Andrica - Proving Some Geometric Inequalities by Using Com-plex Numbers, Educatia Matematica, vol. 1, nr. 2 (2005), 19-26.

3. V. Bandila - O generalizare a unei relatii a lui Leibniz si aplicarea ei la calculareadistantelor dintre unele puncte remarcabile ale unui triunghi, Gazeta Matematica, 90(1985), nr. 2.

4. O. Bottema, R.R. Janic, D.S. Mitrinovic - Geometric Inequalities, Wolters-Noordhoe Publishing, Groningen 1969.

5. D. Marinescu - O identitate cu numere complexe, Gazeta Matematica, seria B, nr.12(1995).

6. D. Marinescu, I. Serdean - Inegalitati geometrice. Aplicatii, Recreatii Matematice2002, nr. 1.

7. C.P. Niculescu - Interferente ıntre mecanica si geometrie, Gazeta Matematica, seriaA, Vol XIX, nr. 2.

8. O. Pop - About Bergstrom ′s inequality, Journal of Mathematical Inequalities, Vol.3,nr. 2(2009).

9. R. Weitzenbock - Uber eine Ungleichung in der Dreiecksgeometrie, MathematischeZeitschrift, 5(1919), no. 1-2, 137-14.

Vizitati pagina web a revistei Recreatii Matematice:

http://www.recreatiimatematice.ro

16

Page 21: Revista (format .pdf, 6.5 MB)

Proprietati caracteristiceale triunghiului echilateral

Florina TOMA1

Abstract. It is well-known that in a equilateral triangle the point O,H,G, I do coincide. Oneobtains several characterizations of the equilateral triangle as converse properties of this sentence.

Keywords: circumcenter, orthocenter, centroid, incenter.

MSC 2000: 51M04.

Scopul notei de fata este gasirea unor proprietati caracteristice triunghiului echi-lateral. Se stie ca ıntr-un triunghi ABC echilateral punctele importante: O,G,H, I,Γ(Gergonne), N (Nagel), K (Lemoine) etc. coincid. Ca urmare coincid si triunghiurilepedale corespunzatoare (adica triunghiurile determinate de picioarele cevienelor con-curente ın fiecare dintre aceste puncte importante ın parte). In notarea acestora vomfolosi indici; astfel, punctului H (ortocentru) ıi corespunde ∆AHBHCH (triunghiulortic) avand punctele importante, cu semnificatii evidente, OH , GH ,HH , IH etc. Re-ciproc, daca doua puncte importante, luate din △ABC sau din diferite triunghiuripedale, coincid, putem afirma ca △ABC este echilateral? Din multimea cazurilor ceapar ın aceasta formulare generala a problemei, avem ın vedere doar pe acelea ın careun punct X ∈ {O,G,H, I} coincide cu unul dintre punctele importate ale △AIBICI ,△AHBHCH , △AGBGCG sau △AOBOCO. Chiar si asa, teoretic gasim 4 · 4 · 4 = 64cazuri posibile, motiv pentru care vom rezolva numai o parte dintre ele. Vom avea ınvedere numai triunghiuri ascutitunghice.

Mai ıntai, enuntam fara demonstratie un rezultat simplu si util:

Lema. Fie punctele M ∈ (BC), N ∈ (CA) si P ∈ (AB). Daca △MNP esteechilateral si △ANP , △BPM , △CMN sunt isoscele, cu varfurile A,B si C, atunci△ABC este echilateral.

1. Cazul I si △AIBICI . Vom vedea ca △ABC este echilateral, daca I coincidecu unul dintre punctele OI , II , GI , HI .

Propozitia 1.1. Daca I ≡ OI , atunci △ABC este echilateral.

Demonstratie. Fie D = prBCI, E = prCAI si F = prABI. Avem: △IDAI ≡A

B CD

I

F

E

AI

CI B

I

△IEBI ≡ △IFCI(IC). Deci ÖIAID ≡ ÕIBIE ≡ ÕICIF ,

relatie care revine la ÒC +bA2

≡ ÒC +ÒB2

≡ bA+ÒC2. Rezulta

ca bA ≡ ÒB ≡ ÒC, adica △ABC este echilateral.

Propozitia 1.2. Daca I ≡ II , atunci △ABC esteechilateral.

1Eleva, cl. a X-a, Colegiul ”National”, Iasi

17

Page 22: Revista (format .pdf, 6.5 MB)

Demonstratie. AAI este bisectoare pentru bA si ØBIAICI . Ca urmare,△AAICI ≡ △AAIBI (ULU) si avem: ABI = ACI , adica △ABICI este isoscel, siAIBI = AICI . La fel obtinem ca△BCIAI si △CAIBI sunt isoscele si BICI = BIAI ,CIAI = CIBI . Conform Lemei, △ABC este echilateral.

Propozitia 1.3. Daca I ≡ GI , atunci △ABC este echilateral.

Demonstratie. I fiind centru de greutate ın △AIBICI , rezulta ca bisectoareaAAI trece prin mijlocul laturiiBICI ; deci△ABICI este isoscel cu varful ınA. Analog,△BCIAI este isoscel cu varful ın B si △CAIBI este isoscel cu varful ın C. In acelasitimp, AAI ⊥ BICI , BBI ⊥ CIAI si CCI ⊥ AIBI , adica ın △AIBICI medianele suntsi ınaltimi; deci, △AIBICI este echilateral. Conform Lemei, obtinem ca △ABC esteechilateral.

Propozitia 1.4. Daca I ≡ HI , atunci △ABC este echilateral.

Demonstratie. Avem AI ⊥ BICI si AI bisectoare ın △ABICI . Acest triunghiva fi isoscel de varf A. Afirmatii analoage relativ la △BCIAI si △CAIBI . Din celedeja vazute, AAI , BBI , CCI sunt mediane ın △AIBICI . Cum, din ipoteza, ele suntsi ınaltimi, rezulta ca △AIBICI este echilateral. Utilizand Lema, deducem ca △ABCeste echilateral.

2. Cazul G si △AGBGCG (triunghiul median). Observam ca punctul G estecentru de greutate atat pentru △ABC cat si pentru △AGBGCG, adica G ≡ GG.Altfel spus, coincidenta G ≡ GG nu impune triunghiului dat vreo restrictie. Darcoincidentele G ≡ OG, G ≡ HG, G ≡ IG revin la GG ≡ OG, GG ≡ HG, GG ≡ IG;ca urmare, triunghiul median va fi echilateral daca una dintre aceste ultime relatiiare loc. In final △ABC, care este asemenea cu cel median, va fi echilateral daca areloc una dintre coincidentele G ≡ OG (OG este centrul cercului lui Euler), G = HG,G ≡ IG.

3. Cazul H si △AHBHCH (triunghiul ortic). Observam acum ca ortocentrul Heste centrul cercului ınscris (punctul IH) al △AHBHCH , adica H ≡ IH . Ca si ıncazul precedent, coincidenta H ≡ IH nu impune triunghiului ABC nicio restrictie,dar oricare dintre conditiile: H ≡ OH (OH este centrul cercului lui Euler), H ≡ GH ,H ≡ HH conduce la faptul ca △ABC este echilateral (se utilizeaza Lema!).

4. Cazul O si △AOBOCO. Vom utiliza mai jos faptul ca ınaltimea din A sidiametrul prin A fac unghiuri egale cu laturile AB si respectiv AC.

Propozitia 4.1. Daca O ≡ OO, atunci △ABC este echilateral.

Demonstratie. Avem △OAOB ≡ △OBOA (LUL). Ca urmare, ÕOAOB

A

B C

O

OA

OBO

C

HA

≡ ÕOBOA. Dar m(ÕOAOB) = m(ÕBAHA) =π

2−B (s-a consi-

derat, ca ın figura, m(ÒB) ≥ m(ÒC). Deci, m(ÕOBOA) =π

2−

B. Dar, ın triunghiul OBC avem m(ÕBOC) = 2m( bA) si, ca

urmare, m(ÕOBOA) =π

2−A. Va rezulta ca A = B si ın final

△ABC va fi echilateral.

18

Page 23: Revista (format .pdf, 6.5 MB)

Propozitia 4.2. Daca O ≡ HO, atunci △ABC este echilateral.

Demonstratie. Rezulta ca AO ⊥ BOCO. Asadar, m(ØABOCO) =π

2−

m(ÕOABO) =π

2−�π2−B

�= B, adica BOCO este antiparalela cu BC sim(ÕOBOA) =

m(ÖOAOB) =π

2− B (patrulaterul ABOAOB este inscriptibil). Se continua ca ın

propozitia precedenta si se deduce ca △ABC este echilateral.

Propozitia 4.3. Daca O ≡ GO, atunci △ABC este echilateral.

Demonstratie. Cu teorema sinusurilor, aplicata ın△ABAO si△ACAO obtinem:AOB

AOC=

c sin[A− (π2 − C)]

b sin(π2 −B)=

c cosC

b cosB=

sin 2C

sin 2B; analog,

BOC

BOA=

sin 2A

sin 2Csi

COA

COB=

sin 2B

sin 2A. De aici, obtinem relatiile: BOA =

b sin 2C

sin 2A+ sin 2C, COA =

c sin 2B

sin 2A+ sin 2B.

Acum, sa observam ca, ın conformitate cu ipoteza, AO trece prin mijlocul Mal segmentului BOCO. Cu teorema sinusurilor, relativ la △AMBO si △AMCO,

vom obtine: 1 =MBO

MCO=

sin(π2 −B)ABO

sin[A− (π2 −B)]ACO=

cosB ·ABO

cosC ·ACO. Deci, cosB ·

ABO = cosC · ACO si, tinand seama de expresiile gasite pentru ABO si ACO,b cosB sin 2C

sin 2A+ sin 2C=

c cosC sin 2B

sin 2A+ sin 2B. Numaratorii fiind egali, obtinem sin 2C = sin 2B

si apoi B = C. La fel obtinem C = A, ceea ce ıncheie demonstratia.

Observatii. 1) In cazul O ≡ IO calculele se complica si mai mult; propunemcititorilor sa se ocupe de aceasta situatie.

2) Daca vom ,,ıncrucisa” cazurile – sa cerem coincidenta unor puncte importantedin triunghiuri pedale diferite (de exemplu, IG = HO etc.) – vor aparea dificultatimari, imposibil de abordat cu mijloace elementare.

Bibliografie

1. C. Cocea – 200 de probleme din geometria triunghiului echilateral, Editura ,,Gh.Asachi” Iasi, 1992.

2. V. Nicula – Geometrie plana (sintetica, vectoriala, analitica). Culegere de probleme,Editura Gil, Zalau, 2002.

3. Gh. Titeica – Probleme de geometrie, Editura Tehnica, Bucuresti, 1961.

19

Page 24: Revista (format .pdf, 6.5 MB)

A Group-Permutation Algorithm to Solvethe Generalized SUDOKU1

Florentin SMARANDACHE2

Abstract. Sudoku can be generalized to squares whose dimensions are n2 × n2, where n ≥ 2,using various symbols (numbers, letters, mathematical symbols, etc.), written just one time on eachrow and on each column; and the large square is divided into n2 small squares with the side n × nand each will contain all n2 symbols written only once. In this paper we present an elementarysolution for the generalized sudoku based on a group-permutation algorithm.

Keywords: permoutation, group, sudoku.

MSC 2000: 00A08, 97A20.

Sudoku is a game with numbers, formed by a square with the side of 9, and oneach row and column are placed the digits 1, 2, 3, 4, 5, 6, 7, 8, 9, written only one time;the square is subdivided in 9 smaller squares with the side of 3× 3, which, also, mustsatisfy the same condition, i.e. each square to contain all digits from 1 to 9 writtenonly once.

The Japanese company Nikoli has popularized this game in 1986 under the nameof sudoku, meaning ”single number”.

Sudoku can be generalized to squares whose dimensions are n2×n2, where n ≥ 2,using various symbols (numbers, letters, mathematical symbols, etc.), written justone time on each row and on each column; and the large square is divided into n2

small squares with the side n × n and each will contain all n2 symbols written onlyonce.

An elementary solution of one of these generalized Sudokus, with elements (sym-bols) from the set

S = {s1, s2, . . . , sn, sn+1, . . . , s2n, . . . , sn2}

(supposing that their placement represents the relation of total order on the set ofelements S), is:

Row 1: all elements in ascending order

s1, s2, . . . , sn, sn+1, . . . , s2n, . . . , sn2

On the next rows we will use circular permutations, considering groups of n elementsfrom the first row as follows:

Row 2:

sn+1, sn+2, . . . , s2n; s2n+1, . . . , s3n; . . . , sn2 ; s1, s2, . . . , sn

1Articolul a fost publicat de autor ın cartea sa Frate cu meridianele si paralelele, vol. IV, pp.201-202, Offsetcolor, Rm. Valcea, 2008. Autorul are acordul editurii de republicare a articolului ınrevista Recreatii Matematice.

2University of New Mexico, Gallup Campus, USA

20

Page 25: Revista (format .pdf, 6.5 MB)

Row 3:

s2n+1, . . . , s3n; . . . , sn2 ; s1, s2, . . . , sn; sn+1, sn+2, . . . , s2n,

.....................................................................................................................Row n:

sn2−n+1, . . . , sn2 ; s1, . . . , sn; sn+1, sn+2, . . . , s2n; . . . , s3n; . . . , sn2−n.

Now we start permutations of the elements of row n +1 considering again groupsof n elements.

Row n+ 1:

s2, . . . , sn, sn+1; sn+2, . . . , s2n, s2n+1; . . . ; sn2−n+2, . . . , sn2 , s1

Row n+ 2:

sn+2, . . . , s2n, s2n+1; . . . ; sn2−n+2, . . . , sn2 , s1; s2, . . . , sn, sn+1

.....................................................................................................................Row 2n:

sn2−n+2, . . . , sn2 , s1; s2, . . . , sn, sn+1; sn+2, . . . , s2n, s2n+1; . . .

Row 2n+ 1:

s3, . . . , sn+2; sn+3, . . . , s2n+2; . . . ; sn2+3, . . . , sn2 , s1, s2

and so on.Replacing the set S by any permutation of its symbols, which we ′ll note by S′,

and applying the same procedure as above, we will obtain a new solution.The classical Sudoku is obtained for n = 3.Below is an example of this group-permutation algorithm for the classical case:

1 2 3 4 5 6 7 8 94 5 6 7 8 9 1 2 37 8 9 1 2 3 4 5 62 3 4 5 6 7 8 9 15 6 7 8 9 1 2 3 48 9 1 2 3 4 5 6 73 4 5 6 7 8 9 1 26 7 8 9 1 2 3 4 59 1 2 3 4 5 6 7 8

For a 42 × 42 square we use the following 16 symbols:

{A,B,C,D,E, F,G,H, I, J,K,L,M,N,O, P}

and use the same group-permutation algorithm to solve this Sudoku.

21

Page 26: Revista (format .pdf, 6.5 MB)

From one solution to the generalized Sudoku we can get more solutions by simplydoing permutations of columns or/and of rows of the first solution.

A B C D E F G H I J K L M N O PE F G H I J K L M N O P A B C DI J K L M N O P A B C D E F G HM N O P A B C D E F G H I J K LB C D E F G H I J K L M N O P AF G H I J K L M N O P A B C D EJ K L M N O P A B C D E F G H IN O P A B C D E F G H I J K L MC D E F G H I J K L M N O P A BG H I J K L M N O P A B C D E FK L M N O P A B C D E F G H I JO P A B C D E F G H I J K L M ND E F G H I J K L M N O P A B CH I J K L M N O P A B C D E F GL M N O P A B C D E F G H I J KP A B C D E F G H I J K L M N O

Bibliografie

1. Z. Pitkow – Sudoku: Medium to Hard, Chronicle Books, 2006.

2. F. Longo – Absolutely Nasty Sudoku Level 4 (Mensa), Puzzlewright, 2007.

3. P. Gordon, F. Longo – Mensa Guide to Solving Sudoku: Hundreds of Puzzles PlusTechnique to Help You Crack Them All, Sterling, 2006.

(continuare de la pag. 10)

Solutie. Avem adunarea abc + a′b′c′ = ABC. Prima conditie din ipoteza este:a+ b+ c+ a′ + b′ + c′ +A+B+C = 1+2+ 3+ 4+ 5+ 6+ 7+ 8+ 9 = 45 si se scrie:A+B + C = 45− (a+ b+ c+ a′ + b′ + c′) (1). Avem patru cazuri:

I. a+ a′ = A, b+ b′ = B, c+ c′ = C(1)⇒ A+B + C =

45

2, contradictie;

II. a + a′ + 1 = A, b + b′ + 1 = 10 + B, c + c′ = 10 + C(1)⇒ A + B + C =

27

2,

contradictie;

III. a+ a′ = A, b+ b′ + 1 = B, c+ c′ = 10 + C(1)⇒ A+B + C = 18.

IV. a+ a′ + 1 = A, b+ b′ = 10 +B, c+ c′ = C(1)⇒ A+B + C = 18.

(continuare la pag. 27)

22

Page 27: Revista (format .pdf, 6.5 MB)

Graphes et matrices de Moore

Adrien REISNER1

Abstract. Let G be any graph with maximum degree d and diameter D = 2. A Mooregraph is defined as a graph for which n = d2 + 1, where n is the order of G. Some elementaryproperties of these graphs are presented. It is proved the Hoffman-Singleton theorem, which assertsthat d ∈ {2, 3, 7, 57}. A particular attention is given to the Moore graphs with degree 2 and 3.

Keywords: Moore graph, Moore matrix, Petersen graph, Hoffman-Singleton graph.

MSC 2000: 05C50.

I. Quelques definitions de la theorie des graphes. Un graphe non orienteG = (S,A) est defini par un ensemble fini quelconque S dont les elements sont appelesles sommets et par un sous - ensemble A de S×S dont les elements sont appeles lesaretes. Le cardinal de S est l′ordre du graphe. Les sommets a et b sont adjacents si(a, b) ∈ A. Le nombre de sommets adjacents au sommet a est appele le degre de a,note d(a). Le degre maximal de G est d = Maxs∈Sd(s). Un chemin de longueur nentre les sommets a et b est une suite (a = x0, x1, . . . , xn = b) de sommets adjacents.Pour toute paire de sommets a, b de G la distance de a a b est la longueur du pluscourt chemin entre a et b. Le diametre D du graphe G est la distance maximaleentre deux sommets distincts de G. Ayant numerote les sommets d′un graphe Gd′ordre n : S = {xi|i : 1, . . . , n} la matrice d′adjacence du graphe G est la matriceAdj(G) = (aij)1≤i,j≤n ∈ Mn(R) definie par: aij = 1 si xi, xj sont adjacents et aij = 0sinon.

1

2 3

4

Exemple. A=

�0 1 0 11 0 1 10 1 0 01 1 0 0

�est la matrice d′adjacence du graphe

Soit un graphe G d′ordre n de diametre D = 2 (deux sommets quelconques sontdonc a une distance 1 ou 2) et de degre maximal d.

Proposition 1. On a : n ≤ d2 + 1.En effet, fixons un sommet s ∈ S. Ce sommet s possede au plus d voisins et

chacun de ceux-ci possede au plus d− 1 voisins autres que s. Donc le graphe possedeau plus d+ 1 + d(d− 1) = d2 + 1 sommets.

II. Graphes et matrices de Moore. On appelle d-graphe de Moore ungraphe non oriente de diametre D = 2 et verifiant: n = d2+1. La matrice d′adjacenced′un tel graphe est une matrice de Moore. Le raisonnement de la proposition 1 montreque dans un d-graphe de Moore chaque sommet est de degre d et est relie a toutsommet distinct par exactement un chemin (d′ordre 1 ou 2) (*).

Jn etant la matrice carree d′ordre n dont tous les coefficients valent 1, on a letheoreme:

Theoreme 2. Une matrice de Moore A est une matrice binaire telle que: i) Aest symetrique, ii) trA = 0, iii) il existe d ≥ 1 verifiant: A2 +A− (d− 1)In = Jn.

1Centre de Calcul E.N.S.T., Paris; e-mail: [email protected]

23

Page 28: Revista (format .pdf, 6.5 MB)

Etant donne A = (aij) une matrice de Moore, demontrons d′abord le lemmesuivant ou A2 = (αij):

Lemme. αij est le nombre de chemins de longueur 2 du sommet i au sommet j.

En effet, αij =nP

k=1

aikakj = aikajk la matrice A etant symetrique. αij est le

nombre de 1 communs a la ligne i et a la ligne j: c′est aussi le nombre de cheminsde longueur 2 du sommet i au sommet j compte tenu de la definition meme de lamatrice d′adjacence A du graphe de Moore.

Un d-graphe de Moore G etant non oriente, sa matrice d′adjacence est symetrique.D′apres (*) aii = 0 pour tout i : 1, . . . , n et par suite trA = 0. Si j = i, on a aii = 0

et αii =nX

k=1

a2ik = d nombre de sommets voisins du i-eme sommet; donc aii+αii = d.

Si i = j, soit aij = 0 et alors les sommets i et j sont relies par un chemin delongueur 2 et αij = 1, soit aij = 1 et les sommets i et j ne sont pas relies par unchemin d′ordre 2, donc αij = 0. Dans tous les cas on a: aij + αij = 1. Finalement:aij + αij = 1 + (d− 1)δij soit A2 +A− (d− 1)In = Jn.

On se propose d′etudier le spectre de la matrice A, i.e. l′ensemble des valeurspropres de cette matrice, et de montrer que le degre maximal d d′un d-graphe deMoore appartient obligatoirement a l′ensemble {2, 3, 7, 57}.

Soit une matrice de Moore A = (aij). U etant le vecteur colonne dont tous lescoefficients sont egaux a 1, on a:

Theoreme 3. d est valeur propre de la matrice A associe au vecteur propreU : AU = dU .

Avec les notations precedentes αii est le nombre de 1 dans la ligne i et d′autrepart - voir demonstration du theoreme 2− αii = d, d′ou le theoreme 3.

Soit Ed le sous-espace propre associe a d . b et c etant les racines du polynome

X2+X− (d−1) : b =−1−

√4d− 3

2< 0 et c =

−1 +√4d− 3

2> 0, on a le theoreme

suivant:

Theoreme 4. Le spectre Sp(A) de la matrice A est tel que Sp(A) ⊂ {b, c, d}.Soit λ ∈ Sp(A) et X un vecteur propre associe, i.e. AX = λX. On a alors:

JnX = (λ2 + λ − (d − 1))X. Par suite X est aussi vecteur propre de la matriceJn pour la valeur propre λ2 + λ − (d − 1). Or cette matrice Jn est de rang 1 etdiagonalisable: Sp(Jn) = {0, n}. Il en resulte alors soit que les vecteurs X et U sontlies et dans ce cas λ = d compte tenu du theoreme 3, soit X = KerJn et dans ce casλ ∈ {b, c}.

La matrice A etant symetrique reelle, elle est diagonalisable dans une base Borthonormee. Le sous-espace propre associe a la valeur propre d est la droite vectorielleRU .

Theoreme 5 (Hoffman et Singleton, 1960). On a d ∈ {2, 3, 7, 57} (voir [1]pour une demonstration utilisant des outils de la theorie des graphes).

24

Page 29: Revista (format .pdf, 6.5 MB)

Soient Eb et Ec les sous – espaces propres associes aux valeurs propre b et crespectivement. Designons par β = dim Eb ∈ N∗. Il vient alors, la matrice A etantdiagonalisable et avec dim Ed = 1: dim Ec = n − 1 − β = d2 − β. Exprimons que

tr A = 0 : d + βb + (d2 − β)c = 0, avec β ∈ N∗. On obtient: d + β−1−√4d−3

2 +

(d2 − β)−1+√4d−3

2 = 0, d′ou

(1) (d2 − 2β)√4d− 3 = d2 − 2d soit β =

d(2− d) + d2√4d− 3

2√4d− 3

∈ N2

β etant un entier non nul, on a d = 2 et donc β = 2, sinon p =√4d− 3 ∈ N∗. La

relation (1) montre alors que p divise d2 − 2d =p2 + 3

4· p

2 + 5

4, donc que 16p divise

d2 − 2d =p2 + 3

4· p

2 − 5

4. Cela impose que p divise 15 soit que p ∈ {1, 3, 5, 15}. Les

seules valeurs possibles pour d sont donc finalement: d ∈ {2, 3, 7, 57} (Le cas p = 1,d = 1 doit etre ecarte car alors n = 2 et le diametre est 1).

Exemple. On se propose de trouver tous les 2-graphes de Moore (donc n = 5).Soit B = {e1, e2, . . . , e5} la base canonique de R5. Pour i, j, k, l,m ∈ {1, . . . , 5}, tousdistincts deux a deux, soit f un endomorphisme tel que Mat(f,B) = A ∈ M5(R) soitune matrice de Moore. Alors f est tel que: f(ei) = ej + ek, f(ej) = ei + el, f(ek) =ei + em, f(el) = ej + em, f(em) = ek + el. Un simple raisonnement combinatoiremontre alors que le nombre N2 des 2-graphes de Moore est: N2 = 2(3 + 2 + 1) = 12.Les douze 2-graphes de Moore (n = 5) sont les suivants:

1

2

3 4

5

1

2

3 4

5

1

2

34

5

1

2

34

5

1

2

3

4

5

1

2

3

4

5

1

2

3

4

5

1

2

3

4

5

1

2 3

4 5

1

2 3

4 5

1

2 3

45

1

2 3

45

III. Applications. A. Graphe de Cayley d′un groupe fini. On considereun groupe fini Γ = {gj} d′element neutre 1 et K une partie generatrice de Γ verifiant:K−1 = K et 1 /∈ K. On appelle graphe de Cayley du groupe associe a K le graphe

...

. .

. ..

. .z1

z2

z =10

z4

z3

z1

z =10

z4

z3

z2

25

Page 30: Revista (format .pdf, 6.5 MB)

Cay(Γ,K) = (S,A) ainsi defini: S = {gj}, (gi, gj) ∈ A ⇔ g−1i gj ∈ K. Soit le groupe

cyclique U5 = {zj = e2iπj

5 }j:0,...,4 et K1 = {z1, z4}, K2 = {z2, z3}. Les graphes deCayley Cay(U5,K1) et Cay(U5,K2) sont les deux 2-graphes de Moore ci-dessus.

B. Matrice de Gram. Soit l′espace vectoriel euclidien R5 rapporte a la baseorthonormee B = {e1, e2, . . . , e5}. On note < u, v > le produit scalaire de deuxvecteurs u et v de R5. On considere tous les vecteurs uj obtenus en ajoutant deuxvecteurs distincts de la base B: uj = eα + eβ ou α = β. On obtient ainsi 10 vecteursdistincts deux a deux (en effet, en utilisant le fait que la famille (ek)k:1,...,5 est libreon a = 10 choix possibles). La matrice de Gram de la famille des vecteurs (uj)j:1,...,10est definie par: M = Gram(uj) = (mij) ∈ M10(R) avec mij =< ui, uj >. Calculonsles produits scalaires < ui, uj > en distinguant trois cas:

1) < ui, ui >=< eα, eα > +2 < eα, eβ > + < eβ , eβ >= 2 si ui = eα + eβ ;2) < ui, uj >=< eα, eα > + < eα, eγ > + < eβ , eα > + < eα, eγ >= 1 si

ui = eα + eβ et uj = eα + eγ , β = γ;3) < ui, uj >= 0 si ui = eα + eβ et uj = eδ + eγ α, β, δ, γ distincts.On verifie alors immediatement la proposition suivante:

Proposition 6. La matrice A = J10−M + I10 est une matrice de Moore. SpA ⊂Z∗.

En particulier, i etant fixe, soit ui = eα + eβ ou α = β. Si uj = eδ + eγ on amij = 1 si et seulement si les quatre indices α, β, δ, γ sont distincts. Pour i donne ily a donc

�32

�= 3 tels coefficients: dans ce cas d = 3 (donc on a bien n = 10). Le

3-graphe de Moore ainsi obtenu est un graphe de Petersen - voir ce graphe ci-dessous(la figure de droite).

C. Configuration de Desargues. Soient ABC et A′B′C ′ deux triangles sanssommets communs. Si AA′, BB′ et CC ′ sont concourantes en I, alors les points

A

B

C

AB

C

I

L

K

J.

.

.

.

.

.

.

.

. .

.A

L

B

A

K

C

C J

IB

J = AB ∩A′B′, K = BC ∩B′C ′ et L = CA ∩ C ′A′ sont alignes: c′est le theoreme

26

Page 31: Revista (format .pdf, 6.5 MB)

de Desargues. Dans cette configuration - configuration de Desargues - il y a 10droites et 10 points; par chaque point il passe 3 droites et sur chaque droite il y a 3points. On represente cette configuration par un graphe G = (S,A). Les sommets dece graphe sont les 10 points de la configuration de Desargues. Deux sommets i et jsont relies par une arete si et seulement si les points correspondants ne sont pas surune meme droite. On obtient ainsi le 3-graphe de Moore, i.e. le graphe de PetersenG ci-dessus.

Remarque. Pour d = 2, d = 3 et d = 7 on a montre l′existence des d-graphes de Moore: graphes de Petersen pour d = 3, n = 10 (voir le site web:http://mathematiques.ac-dijon.fr/ressources/graphes/jca/intro.htm), gra-phes de Hoffman – Singleton pour d = 7, n = 50. Par contre, l′existence d′un 57-graphe de Moore n′a pas ete encore demontre: un tel graphe aurait 3250 sommets.

References

1. A.J. Hoffman, R.R. Singleton - On Moore Graphs with Diameters 2 and 3, IBMJ.Res. Develop., 5(4)(1960), 497-504.

2. S.V. Duzhin - Graphes de Moore (demonstration de Chmutov),http://www.pdmi.ras.ru/ duzhin/papers/ .

3. R.A. Brualdi , H.J. Ryser - Combinatorial matrix theory, Cambridge UniversityPress, 1991.

(continuare de la pag. 22)

Deci nu putem avea decat A+B + C = 18 care, coroborata cu conditia a doua aipotezei, impune rezultatului sa fie format din cifrele 5, 6 respectiv 7, nu neaparat ınaceasta ordine.

Sa gasim toate posibilitatile. Mai ıntai, observam ca daca schimbam a cu a′, bcu b′ si c cu c′ rezultatul nu se modifica (transformarea 1). Asta ınsemna ca, dacafixam un ”exemplu reprezentativ” (sa zicem a < a′, b < b′si c < c′), se obtin 23

posibilitati. Apoi, observam ca, daca are loc abc + a′b′c′ = ABC, atunci are locsi bca + b′c′a′ = BCA (transformarea 2). Asta ınseamna ca pentru un exemplureprezentativ obtinem 2 · 23 = 16 posibilitati.

Urmatoarele doua exemple sunt reprezentative 128+439 = 567 si 218+439 = 657.Pentru primul exemplu obtinem: 128+ 439 = 567, 138+ 429 = 567, 129+ 438 = 567,139 + 428 = 567, 428 + 139 = 567, 438 + 129 = 567, 429 + 138 = 567, 429 + 138 =567 (conform transformarii 1), la care aplicam transformarea 2: 281 + 394 = 675,381+294 = 675, 291+384 = 675, 391+284 = 675, 284+391 = 675, 384+291 = 675,294 + 381 = 675, 294 + 381 = 675 - ın total 16 posibilitati. Analog pentru celalaltexemplu reprezentativ. Avem 2 · 16 = 32 posibilitati.

27

Page 32: Revista (format .pdf, 6.5 MB)

Asupra unor probleme de desfasurare

Marian-Dumitru PANTIRUC 1

Abstract. Three problems of minimum lengths are solved. They have an obvious practicalapplicability (like Pisa type tests).

Keywords: right-angled parallelepiped, right circular cone.

MSC 2000: 51M04.

In aceasta nota vom rezolva cateva probleme de geometrie de distanta minima.Ideea este de a face o desfasurare laterala convenabila. Prin continutul (mai mult saumai putin) practic al problemelor, putem sa le legam de testele de tip Pisa.

Problema 1. In fig. 1 de mai jos sunt prezentate schematic doua blocuri delocuinte de 10 etaje + parter, respectiv de 4 etaje + parter. Blocul de 4 etaje arelungimea de 30 m si este lat de 10m. Blocul de 10 etaje are baza un patrat cu laturade 10m iar distanta dintre blocuri este tot de 10m. Inaltimea unui nivel este de 2,5m.Sa se afle lungimea celui mai scurt cablu de distributie (de semnal TV) care leagapunctele A si M , unde M este mijlocul laturii pe care se afla.

A

P

Q

M

S

R

B

Fig.1

AQ

M

S

B

M

B

S

Fig.2

AQ

M

S

B

M

S

S

Q

Fig.3

Solutie. Bineınteles, nu vom gauri peretii blocului pentru a ıntinde cablul. Vatrebui sa ne ”legam” de marginile blocului pentru a duce firul din A ın M . Sigur caavem mai multe posibilitati de legare si deci de a uni punctele A si M .

Scrierea A−RS−PQ−M ınseamna ca firul pleaca din A, ıl legam prima data pemuchia [RS] s.a.m.d. Este clar ca nu sunt multe drumuri care merita luate ın calcul:

1. A−RS − PQ−M 2. A− PS − PQ−M 3. A− PQ−M4. A− ST − PB −M 5. A− PS − PB −M 6. A− PB −M .

Observam ca orice drum de tip 1 sau 2 este strict mai lung decat un drum de tip 3:suma a doua laturi ale unui triunghi este strict mai mare decat cea de-a treia latura.De asemenea, nu ne putem lega de muchia PB fara a ne lega de una din muchiile STsau PS. Cum punctele A,P, S,B, T sunt coplanare, cel mai scurt drum dintre 4, 5,6 este cel de tipul 6. In acest moment a ramas sa determinam cel mai scurt drum detip 3, cel mai scurt drum de tip 6 si sa le comparam.

Pentru drumul de tip 3 desfasuram partial blocul cu 4 etaje ın planul (APQ), caın fig. 2. Lungimea celui mai scurt drum unind A cu M va fi acum, ın mod clar,

1Profesor dr., Scoala generala nr. 28 ”Mihai Codreanu”, Iasi

28

Page 33: Revista (format .pdf, 6.5 MB)

egala cu lungimea segmentului [AM ′]. Notam cu S′ punctul de intersectie dintrePS si muchia blocului cu 10 etaje situata ın planul (APS). Inaltimea blocului cu10 etaje fiind de 27, 5m si a celui de 4 etaje de 12, 5m, avem ca AS′ = 15m. CumPS′ = 20m, aplicand teorema lui Pitagora ın ∆PAS′, obtinem AP = 25m. Astfel,AB′ = AP + PB′ = 37, 5m si, aplicand teorema lui Pitagora ın ∆AB′M ′, gasimM ′A = 2, 5

√261 m.

Pentru drumul de tip 6 procedam similar: vom desfasura partial blocul de 4 etajeın planul (APB) (fig. 3). Cu o simpla aplicare a teoremei lui Pitagora, obtinemM ′A = 2, 5

√317 m. Cel mai scurt cablu parcurge deci un drum de tip 3 si are

lungimea de aproximativ 40, 4m.Se ridica problema existentei drumului de tip 6 din desfasurarea din figura 3, adica

daca nu cumva M ′A trece pe deasupra punctului P , fara sa taie PS sau PB. Acestlucru ıl lasam cititorilor spre verificare.

Problema 2. In fig. 4 de mai jos este prezentata urmatoarea situatie:- ın punctul V , la ınaltimea de 3

2

√5 m este situat un bec luminand ın noapte o

zona sub forma de con circular drept;- discul luminat pe asfalt are centrul O si raza de 3m;- ın punctul S se afla un soarece suferind de insomnie;- ıntr-un copac, la o anumita ınaltime se afla o bufnita B, ın acelasi plan cu becul,

soarecele si punctul O.In ani de experienta la vanatoare, bufnita stie ca distanta de unde se afla pana la

bec este egala cu distanta pana la punctul A si ca distanta pe care o are de parcurspana la planul (V AC) este de 9m ((V AC) este sectiunea axiala din con pe care bufnitao vede privind soarecele). Aflati cel mai scurt drum pe care ıl parcurge bufnita pentrua ınsfaca soarecele, stiind ca ın orice moment ar patrunde ın interiorul conului delumina, soarecele o vede si scapa.

Fig. 4

Fig. 5

Solutie. Problema devine una simpla deındata ce observam ca pentru a parcurgeun drum cat mai scurt, bufnita trebuie sa treaca neaparat printr-un punct situat fiepe generatoarea V A, fie V C. Pentru a fixa lucrurile, presupunem ca ın drumul eibufnita trece prin generatoarea V A. Cum un con admite o desfasurare plana, vomdesfasura si noi conul ın planul (V BA), generatoarea de ”ınceput” fiind V A iar sensuldesfasurarii fiind A−S−C−A. Notınd cu S′ punctul de pe desfasurare corespunzatorlui S, devine clar ca drumul minim are lungimea egala cu cea a segmentului [S′B] ın

29

Page 34: Revista (format .pdf, 6.5 MB)

planul (BAV ) (fig. 5). Calculul este usor, observand ca masura unghiului obtinut prin

desfasurarea laterala a conului are masura de 240◦ si deci m(ÕS′V A) = 60◦, triunghiulV AS′ fiind astfel echilateral. Lasam calculele ın seama cititorului.

Problema 3. Pe o bobina cu forma de paralelipiped dreptunghic ABCDA′B′C ′D′,se ınfasoara un fir de cupru ın felul urmator: fiecare muchie laterala a paralelipipedu-lui se ımparte ın n parti egale. Firul pleaca din A trece apoi prin primul punct dediviziune de pe BB′, apoi prin al doilea punct de diviziune de pe CC ′ s.a.m.d. panaajunge ıntr-unul din punctele bazei A′B′C ′D′. Stiind ca ABCD este un patrat delatura 4cm si AA′ = 10cm, aflati numarul de parti egale ın care trebuie ımpartitamuchia laterala pentru ca firul de cupru sa aiba lungimea de cel putin 4m.

AB

CD

A B

CD

B1

C2

.

Fig. 6

A B C

DA AB C

D A

Fig. 7

Solutie. Desi putem rezolva problema si fara desfasurare laterala, fig. 7 arata unposibil ”traseu” al firului pe suprafata laterala a bobinei. Este clar ca lungimea firuluieste egala cu de n ori lungimea unei singure parti la o trecere pe oricare din fetelelaterale (ln). Cum lungimea unei astfel de parti, conform teoremei lui Pitagora, este

ln =

Ê42 +

�10

n

�2

,

rezulta ca lungimea firului este egala cu

l = n · ln =p16n2 + 100 cm.

Pentru ca lungimea firului sa fie cel putin 4m, avem succesiv:p16n2 + 100 > 400 ⇒ 16n2 + 100 > 160000 ⇒ n2 >

159900

16⇒

⇒ n >p

9993, 75 ⇒ n ≥ 100.

Faptul ca n este minim 100 mai spune ca grosimea firului de cupru nu poate depasi1mm.

30

Page 35: Revista (format .pdf, 6.5 MB)

Despre numere, corpuri de numeresi cautarea esentei

Marian TETIVA1

Abstract. It is well-known that, under certain obvious assumptions, a sum of radicals of rationalnumbers is not a rational number. We prove this assertion in the case of order two radicals (which isnot at all a new proof) and try to exhibit some issues about mathematical reasoning in connectionwith this beautiful problem.

Keywords: rational numbers, radicals, number fields.

MSC 2000: 97C90, 97D50.

1. Problema. Pe cand eram elev m-am ıntalnit cu urmatorul enunt (usor modi-ficat aici):

Problema 1 (M85 din [1]). Daca c1, c2, . . . , cn sunt numere ıntregi si d1, d2, . . . , dnsunt numere ıntregi pozitive libere de patrate (nedivizibile cu patratul vreunui numarnatural mai mare ca 1) pentru care

c1pd1 + c2

pd2 + · · ·+ cn

pdn = 0

atunci c1 = c2 = · · · = cn = 0.Se gasesc si pe Internet, ın arhivele Kvant (http://kvant.mirror1.mccme.ru/),

problema [5] si solutia ei [3] (dar nu se gaseau defel ”pe vremea mea”); cum acesteasunt ın limba rusa (ceea ce s-ar putea sa va incomodeze putin), folositi cu ıncredere [1].Considerati toti factorii primi distincti ai numerelor d1, d2, . . . , dn, fie ei p1, p2, . . . , pN ,si ganditi-va de ce problema M85 rezulta din urmatoarea (de fapt, problemele 1 si 2sunt, practic, echivalente).

Problema 2. Se dau numerele prime diferite p1, p2, . . . , pN si numerele rationaleaI indexate dupa toate submultimile I ale multimii {1, 2, . . . , N}, pentru care avemX

I⊆{1,...,N}

aI

ÊYi∈I

pi = 0.

(Suma se face dupa toate cele 2N submultimi ale multimii {1, 2, . . . , N}. Produsulcorespunzator multimii vide se considera egal cu 1.) Atunci aI = 0, pentru oriceI ⊆ {1, 2, . . . , N}.

Analizand (cum se va vedea imediat) posibilitatea de a face o inductie dupa Npentru a rezolva aceasta problema, am ajuns la concluzia ca e mai bine sa demonstramastfel (adica inductiv) urmatorul enunt, o idee mai general:

1Profesor, Colegiul National ”Gheorghe Rosca Codreanu”, Barlad

31

Page 36: Revista (format .pdf, 6.5 MB)

Problema 3. Se dau numerele prime distincte p1, p2, . . . , pN si numerele rationaleaI indexate dupa toate submultimile I ale multimii {1, 2, . . . , N}, pentru care avem� X

I⊆{1,...,N}

aIYi∈I

√pi

�2

∈ Q.

Atunci (macar) 2N − 1 dintre numerele aI sunt egale cu 0.Evident, Problema 3 implica Problema 2. Nu cumva si reciproc? In fine, am fost

condus (dupa ceva eforturi) ın miezul problemei - mereu ıi dai ocol cand vrei sa rezolvio problema, mereu te ınvarti pe la periferie ın jurul centrului, esentei acelei probleme(uneori poti obtine cate ceva chiar si neajungand ın miez, doar dand tarcoale).

Lema. Se considera un corp K de numere reale si un numar real α din afara luiK, dar astfel ıncat α2 ∈ K. Fie K(α) cel mai mic corp care contine pe K si α, altfelspus: K(α) = {z : z = x+ yα, x, y ∈ K}. Atunci un element z = x+ yα din K(α)are proprietatea ca z2 ∈ K daca si numai daca x = 0 sau y = 0. (Mentionam caacest enunt se poate da si pentru o extindere oarecare de corpuri K ⊆ L, cu α ∈ L,α ∈ K, α2 ∈ K etc.)

Totul se bazeaza pe aceasta lema! Inainte de a o demonstra, justificati ca, ıntr-adevar, K(α) este multimea numerelor de forma x+ yα, cu x, y ∈ K si ca un elementz ∈ K(α) se exprima ın mod unic ın aceasta forma (adica x1 + y1α = x2 + y2α, cux1, x2, y1, y2 ∈ K, implica x1 = x2 si y1 = y2; sau, echivalent, ca, pentru x, y ∈ K,avem x+ yα = 0 ⇒ x = y = 0; remarcati importanta structurii de corp).

Acum demonstratia Lemei este lesne de facut. Sa presupunem ca (x+ yα)2 = r,unde x, y si r sunt elemente ale corpului K. Obtinem x2 + y2α2 − r + 2xyα = 0,cu x2 + y2α2 − r ∈ K si 2xy ∈ K, deci 2xy = 0, deci x = 0 sau y = 0. Reciprocaeste limpede. Se vede ca important e rezultatul mentionat ın paranteza anterioara:x + yα = 0, cu x, y ∈ K si α ∈ K implica x = y = 0. Dar intentionat l-am lasat ınparanteza, ganditi-va de ce!

Da! asa se ıntampla de multe ori: un rezultat (ai zice ca) banal conduce lasenzationale concluzii.

2. Analiza problemei. Sa ıncerc sa explic cum mi-am dat seama de asta, por-nind de la niste cazuri simple, pe care le stim cu totii de prin clasa a noua.

Exercitiul 1. Sa se arate ca daca x+ y√2 = 0, cu x, y ∈ Q, atunci x = 0 si y = 0.

Rezolvarea e foarte simpla, va recomand sa v-o amintiti. Dar mai ales va recomandsa sesizati, si aici, care este esenta: anume ca

√2 nu este numar rational si ca un

raport de numere rationale este numar rational - ceea ce deja ne conduce inexorabilcatre structura de corp a acestei multimi care le-a fost atıt de draga grecilor antici.Observati, de asemenea, ca Exercitiul 1 e un caz particular al Lemei.

Pe urma, daca ati fost norocosi, v-ati intersectat si cuExercitiul 2. Sa se arate ca, daca a, b, c, d sunt numere rationale si a + b

√2 +

c√3 + d

√6 = 0, atunci a = b = c = d = 0.

32

Page 37: Revista (format .pdf, 6.5 MB)

Acum lucrurile par (si sunt) ceva mai delicate, dar cu cateva ridicari la patrat sifolosind iar structura de corp a multimii numerelor rationale, o veti scoate la capat. Vaspun ca, studiind acest exercitiu (pe care mai ınainte l-am rezolvat de mai multe ori;l-am considerat greu la un moment dat, mi s-a parut din ce ın ce mai simplu pe masurace trecea timpul si ıl reıntalneam; totusi, la fel cum se ıntampla cu oamenii, nu stiamnimic despre el!), am descoperit Lema (mi s-a descoperit), cautand sa-l rezolv altfeldecat traditional. Si Ideea a fost sa scriu egalitatea ın forma (a+b

√2)+(c+d

√2)√3 =

0, de unde sa obtin√3 = −a+ b

√2

c+ d√2= e+ f

√2

cu e si f numere rationale. Dar asta nu se poate decat daca e = 0 sau f = 0, caci(e + f

√2)2 = 3 ∈ Q (observati, va rog, cum apare Lema), iar ambele conduc la

contradictie. Ramane doar ca ecuatia initiala sa nu poata fi rezolvata ın raport cu√3, ceea ce ınseamna ca avem c + d

√2 = 0, si atunci a + b

√2 = 0, de asemenea.

Conform Exercitiului 1 rezulta atunci a = b = c = d = 0, ceea ce trebuia demonstrat.De aici la solutia problemei generale n-a mai fost de facut decıt pasul redactarii.

3. Solutia problemei. De fapt rezolvam Problema 3, echivalenta cu celelalte olasam ın grija cititorului. Pentru N = 1, cu notatiile a∅ = x, a{1} = y, si p1 = p,trebuie sa demonstram ca, daca (x+ y

√p)2 ∈ Q, atunci macar unul dintre x si y este

0. Ceea ce, clar, rezulta din Lema, pentru K = Q si α =√p.

Sa presupunem ca enuntul este adevarat pentru N , si sa consideram ca� PI⊆{1,...,N+1}

aIQi∈I

√pi

�2

este numar rational, p1, . . . , pN+1 fiind numere prime dis-

tincte, iar aI fiind 2N+1 numere rationale (indexate dupa submultimile multimii{1, . . . , N + 1}). Putem scrie aceasta egalitate ın forma (x+ y

√pN+1)

2 ∈ Q, cu

x =X

I⊆{1,...,N}

aIYi∈I

√pi si y =

XI⊆{1,...,N}

aI∪{N+1}Yi∈I

√pi.

Desigur, x si y sunt din corpul Q(√p1, . . . ,

√pN ), si conditia (x + y

√pN+1)

2 ∈ Qimplica (x + y

√pN+1)

2 ∈ Q(√p1, . . . ,

√pN ); alegand K = Q(

√p1, . . . ,

√pN ) si α =√

pN+1, conform Lemei avem x = 0 sau y = 0.Pentru a putea aplica Lema trebuie sa observam ca α2 = pN+1 ∈ Q ⊆ K si α ∈ K;

asta deoarece α ∈ K ar ınsemna

√pN+1 =

XI⊆{1,...,N}

xI

Yi∈I

√pi

pentru anumite numere xI ∈ Q, I ⊆ {1, . . . , N}. Atunci am avea, desigur,� PI⊆{1,...,n}

xI

Qi∈I

√pi

�2

= pN+1 ∈ Q, si ipoteza de inductie ne-ar spune ca 2N−1 din-

tre numerele xI sunt 0, deci ne-ar ramıne o egalitate de forma√pN+1 = xI0

Qi∈I0

√pi

pentru o anume submultime I0 a lui {1, . . . , N}, ceea ce n-ar fi posibil pentru niste

33

Page 38: Revista (format .pdf, 6.5 MB)

numere prime distincte p1, . . . , pN+1 si un numar rational xI0 . Astfel vedem ca pre-supunerea α ∈ K conduce la contradictie - deci este falsa.

Acum x = 0 ınseamna x2 ∈ Q, deci (tot conform ipotezei de inductie) 2N−1 dintrecoeficientii aI , I ⊆ {1, . . . , N} sunt nuli; cum x = 0, de fapt toti cei 2N coeficientisunt 0. De asemenea, conditia initiala se transforma ın (y

√pN+1)

2 ∈ Q, adica y2 ∈ Q.Folosind ınca o data ipoteza de inductie, deducem ca 2N−1 dintre numerele aI∪{N+1},I ⊆ {1, . . . , N} sunt zero. Astfel obtinem, ın final, ca 2N+1 − 1 dintre coeficientii aI ,I ⊆ {1, . . . , N +1}, sunt zero - exact ceea ce am vrut sa demonstram. Cazul y = 0 setrateaza complet analog, si demonstratia se ıncheie aici.

4. Concluzii. E incredibil cum o lema atat de mica destrama un rezultat asa deputernic! (Buturuga mica rastoarna carul mare, nu?)

Pe de alta parte, decalajul dintre Exercitiile 1 si 2 (din punct de vedere al difi-cultatii) si transformarea Problemei 1 (sau 2) ın Problema 3 ne arata ca esenta primeiprobleme este alta decat s-ar putea banui initial: nu este o problema (doar) desprenumere, nu este o problema care caracterizeaza o egalitate. Nu: este o problemacare descrie conditiile ın care se realizeaza apartenenta unui element la o anumitamultime, multime a carei structura de corp este esentiala (a se vedea iar Lema simai ales rezultatul pe care se bazeaza ea: x + yα = 0 cu x, y ∈ K si α ∈ K implicax = y = 0). Practic, o asemenea problema ne conduce obligatoriu la considerareacorpurilor de forma Q(

√p1,

√p2, . . . ,

√pN ) = Q(

√p1)(

√p2) · · · (

√pN ) (veti ıntelege

usor ce ınseamna asta). E clar acum si de ce se poate rezolva Exercitiul 1, dar nusi Exercitiul 2, nu cu aceeasi usurinta: pe primul ıl poti rezolva ramanand ın afaraesentei; dar la al doilea gasesti o solutie simpla (care consta ın aplicarea de douaori a Lemei) doar daca te-ai desprins de pe orbita, cazand fericit tocmai ın nucleu.Calculele efective sunt mai complicate (ceea ce nu ınseamna ca nu se poate si asa; darıncercati un exercitiu similar cu primele doua, care sa reprezinte un caz oarecare alproblemei 3 cu N = 3: ıntelegeti ce vreau sa spun, da?).

O alta solutie (ın afara celor mai sus mentionate) se poate gasi ın [2]. Si maitrebuie spus ca, evident, dupa ce am rezolvat astfel problema si am scris despre eaaceasta nota, am gasit undeva si aceasta abordare [4] - doar ca acolo e formulata cevamai tehnic. Toate-s vechi si noua toate!

Bibliografie

1. H. Banea - Probleme traduse din revista sovietica Kvant, Editura Didactica siPedagogica, Bucuresti, 1983.

2. I. Boreico - Linear independence of radicals, The Harvard College MathematicsReview, http://www.thehcmr.org/issue2 1/mfp.pdf

3. L.N. Kamnev - Irationalitatea unor sume de radicali, (ın limba rusa), Kvant 2/1972,pp. 26-27, http://kvant.mirror1.mccme.ru/1972/02/p26.htm

4. D.J. Newman, H. Flanders - Solution of the Problem 4797, The American Mathe-matical Monthly, Vol. 67, No. 2 (Feb., 1960), pp. 188-189.

5. L.N. Vaserstein - Problema M85Kvant, 5/1971, p. 30,http://kvant.mirror1.mccme.ru/1971/05/p30.htm

34

Page 39: Revista (format .pdf, 6.5 MB)

File din istoria revistei ”Curierul Matematic”

Ultimii ani ai veacului al XIX-lea, dar si primul sfert al veacului urmator, auınsemnat ınceputul emanciparii matematice a natiei romane. Astfel, la 15 ianuarie1883 a vazut lumina tiparului, la Iasi, revista Recreatii Stiintifice, cu aproape 90%continut matematic, existenta sa dainuind pana-n 1888 (v. [2]). In 1895 a aparut laBucuresti Gazeta Matematica, prima publicatie romaneasca dedicata exclusiv stiinteiregine, ca o urmare fireasca a observatiilor unor entuziasti profesori-ingineri, ce con-statasera un nivel stiintific drastic scazut al tinerilor studenti de la Scoala de Podurisi Sosele din Bucuresti. Cinsprezece ani mai tarziu, ın 1910, se pun bazele Societatii”Gazeta Matematica” (devenita apoi Societatea de Stiinte Matematice din Romania),ca o noua etapa a unui ınceput de consolidare a edificiului a carui constructie demarasela trecerea dintre veacuri. Intre timp, peisajul matematic romanesc s-a ımbogatit cuRevista matematica a elevilor din Timisoara, aparuta ın 1920, cu Jurnalul matematic,revista bilunara publicata din 1923 la Arad si Bucuresti de Tiberiu Popoviciu, si cuFoaia matematica de la Chisinau, ce a debutat ın 1924.

Surprinzator pentru cercetatorul istoriei matematice romanesti, este faptul ca asasea revista de matematica este una scoasa doar de tineri elevi ai cursului superior alLiceului ”Dimitrie Cantemir” din Bucuresti (v. [5]) Cu pasiune si profesionalism, darsi cu probitate redactionala deosebita, cum astazi rar mai gasim chiar printre adulti,ei au scos Curierul Matematic, revista ce s-a difuzat cale de 3 ani, ıntre 1925-1928.Primul numar a aparut la 1 martie 1925, iar ultimul a fost cel din 1 februarie 1928.Incetarea existentei Curierului Matematic se datoreaza ıntai de toate consecintelorfinanciare aduse de ajunul crizei economice dintre 1929-1933, dar ın egala masura sizbuciumului prin care a trecut ın acea perioada profesorul Mihail Ghermanescu, ınfond sfatuitorul si ındrumatorul elevilor sai de la Liceul ”Dimitrie Cantemir”, carora leinsuflase dragostea profunda pentru matematica, asa cum a facut-o ın toate institutiilesecundare sau universitare prin care a trecut, si ın ciuda faptului ca era consideratsever si drastic. Disparitia revistei se explica si prin numirea lui M. Ghermanescu, ın1928, la Scoala de geniu din Bucuresti, dar si prin lipsa sa de timp, ocupat fiind si cuabsolvirea, tot ın 1928, a clasei de canto a Conservatorului din Bucuresti, precum sicu pregatirea rolului sau pentru Rigoletto, ın prima sa reprezentatie pe scena Opereibucurestene (v. [1]).

Mentionam ca putin mai tarziu, dar tot ın cursul lui 1925, a aparut si cea de-asaptea revista romaneasca de matematica, intitulata Buletinul Societatii Studentilorın Matematici, asociatie fondata ın februarie 1923 (v. [4]).

Astazi, la mai bine de 85 ani de la publicarea ıntaiului numar al Curierului Mate-matic, ramanem profund impresionati constatand ca acea mana de tineri inimosi augandit continutul destul de variat, au redactat revista ıntr-un format placut vederii(datorita unei caligrafii perfecte), au litografiat-o si au distribuit-o singuri, probabilputin vegheati de catre viitorul profesor universitar Mihail Ghermanescu, pe atunciprofesorul lor la Liceul ”Dimitrie Cantemir”. In primul an al aparitiei sale, Curierul

35

Page 40: Revista (format .pdf, 6.5 MB)

Matematic a fost constituit dintr-un volum cu 12 numere, distribuite regulat la ıntaiale lunii, fiecare avand 16 file, legate ıntr-o coperta cu un frumos chenar ınflorat,care contribuia cu ınca doua file la continutul celorlalte 16. Un abonament pe un anıntreg costa 50 lei, iar un numar separat 6 lei. Orice numar continea articole, notematematice, probleme propuse dar si probleme rezolvate.

La aparitia primului numar, ın 1 martie 1925, comitetul redactional era format dinI. Fantaneanu, fost cantemirist, student-inginer la Scoala Politehnica bucuresteana,si din elevii clasei a VII-a a Liceului ”Dimitrie Cantemir”, respectiv N. Botea siO. Bolgiu. Acesti trei fondatori ai Curierului Matematic, la debutul jurnalului, ınCuvantul ınainte, notau:

Prin publicarea acestei modeste reviste, am avut un scop, acela de a crea o at-mosfera continua, matematica, ıntre studentii de astazi - profesorii sau inginerii demaine - si patura ıncepatoare a elevilor de liceu.

La conducerea revistei, vom alege la finele fiecarui an, pe doi dintre colaboratorii ei,elevi de liceu, care au desfasurat cea mai stralucita activitate la ”Curierul Matematic”.

Revista va publica articole, note si probleme originale cu caracter elementar orisuperior.

Multumim calduros celor care au contribuit la aparitia numarului de fata si rugampe toti prietenii stiintelor matematice sa ne dea binevoitorul lor concurs, pentru carele transmitem anticipat multumirile noastre cordiale.

Nu ne ındoim de concursul revistelor matematice mai vechi: ”Gazeta matematica”,”Revista matematica a elevilor din Timisoara” si ”Foaia matematica de la Chisinau”,la izvorul carora ne-am adapat si ne vom adapa ınca.

Multumim d-lui Leon Brull, asupra caruia cade greaua sarcina administrativa a re-vistei, dar si d-lui D-tru Niculescu, de care sunt legate conditiile de aparitie a Curieru-lui Matematic.

Pe prima pagina a copertei, dupa titlu, se afla un Cuprins, care pentru ıntaiulnumar, pe langa Cuvantul ınainte, mai contine: Proprietati ın legatura cu triun-ghiurile pedale de O. Bolgiu, Asupra izopolului de N. Botea, Asupra produselor nu-merelor formate dintr-o suma de doua patrate de I. Fantaneanu, iar apoi nota mate-matica intitulata Asupra problemei 3053 din Gazeta Matematica XXX, pag. 63, pen-tru ca ın final sa aiba Probleme propuse de: Corneliu C. Ionescu, M. Ghermanescu,Al. Stoenescu, Ovidiu Bolgiu, Al. Mladenovici, I. Fantaneanu, N. G. Botea, Gh. D.Ionescu-Zane.

Pe a doua pagina a copertei se indica persoanele la care vor fi trimise manuscriseledestinate publicarii ın Curierul Matematic, respectiv O. Bolgiu, strada Toamnei,nr. 134, Bucuresti si N. Botea, Calea Floreasca, nr. 28A, Bucuresti. De asemenea,pentru abonamente si difuzare este indicat Leon Brull, strada Calusei, nr. 18, Bu-curesti.

Pe pagina a treia a copertei, se enumera persoanele de la care s-au primit articolesi note matematice sau probleme spre publicare, precum si rezolvitorii.

Pe ultima pagina a copertei primului numar din martie 1925, se relateaza despreo sedinta din 20 februarie 1925 a Societatii Stiintifice ”Dimitrie Cantemir”, aflataın anul IV al functionarii sale, tinuta ın prezenta a 28 de membri, elevi ai liceelorbucurestene. Este foarte interesant de notat ca, la acea data, elevii cautau sa fie la

36

Page 41: Revista (format .pdf, 6.5 MB)

curent cu descoperirile stiintifice, fapt ce rezulta din expunerea lui N. Segarceanuasupra teoriei cuantelor, elaborata de fizicianul german Max Planck (1858-1947).Redactorii N. Botea, O. Bolgiu si administratorul L. Brull au rezolvat, generalizatsau expus probleme cu un caracter geometric, ın timp ce A.T. Ionescu-Bujor si N.Botea au propus noi probleme spre rezolvare si discutare ın sedinta viitoare.

De la 1 iunie 1925, Comitetul de redactie al Curierului Matematic l-a avut camembru si pe L. Brull, iar de la 1 martie 1926, odata cu aparitia volumului aldoilea, acesta se largeste prin cooptarea lui C-tin T. Ionescu-Bujor, cantemirist,dar si a lui Ion C. Vasiliu, de la Liceul din Bacau.

Avand la dispozitie 13 dintre cele 36 numere ale Curierului Matematic, am con-statat cu multa placere consemnarea tuturor rezolvitorilor, ıntre care se aflau atatbaieti cat si fete, ın principal din Bucuresti, dar si din Bacau, Botosani, Caracal,Chisinau, Focsani, Galati, Giurgiu, Ploiesti, Timisoara etc. Erau permanent citateproblemele ınca nerezolvate, aspect al seriozitatii! In paginile 3 si 4 ale copertei, aexistat o posta a redactiei, unde erau consemnate numele celor care au trimis articolesi note, precizate cu titlurile lor, dar si probleme propuse cat si probleme rezolvate.Erau de asemenea citati cei ce deveneau abonati ai revistei, ıntre care existau ingineri,ofiteri, studenti, elevi dar chiar si presedintele Bancii Generale din Brasov, cu totiicontribuind la aparitia jurnalului care dispunea numai de aceste unice fonduri.

Deseori, ındrazneata revista matematica, realizata ın totalitate de elevi, prezentarecenzii ale unor carti moderne pentru perioada respectiva. Desi scrise de elevi, arti-colele si notele lor citau corect bibliografia folosita, fiind un exemplu clasic de mora-litate intelectuala, astazi pierduta aproape definitiv pe plaiurile mioritice. Erau apoiconsacrate rubrici speciale, pentru bibliografia utila liceenilor si studentilor, fapt efec-tiv meritoriu. Apar astfel, frecvent citate: Gazeta matematica, Revista matematica aelevilor din Timisoara, Foaia matematica din Chisinau, Jurnalul matematic din Aradsi Bucuresti, Insemnari matematice de la Cluj, Societatea matematica barladeana,Ziarul matematic de la Chisinau, revista realizata de alti doi elevi inimosi si eminenti,Petru Leescu si Boris Vulpanovici. Nu sunt uitate nici alte reviste straine de mate-matica, anale ale unor universitati, teze de doctorat, culegeri de probleme romanestisi straine, tratate fundamentale ale epocii!

La 1 martie 1926 Comitetul de redactie al Curierului Matematic, ın articolul defond Dupa un an, facea analiza realizarilor revistei cantemiristilor, consemnandu-seaparitia ”cu absoluta regularitate, la ıntai al fiecarei luni”, fapt ce ”va trebui pastratın continuare”. Erau aduse ”vii multumiri d-lui prof. Mihail Ghermanescu, caruia”Curierul Matematic” ıi datoreaza atat de mult si pentru care nu gasim cuvinte destulde potrivite spre a-i exprima recunostinta noastra”. Aceleasi multumiri se ındreptausi spre ”Gazeta Matematica”, ”Revista Matematica a elevilor din Timisoara”, ”FoaiaMatematica de la Chisinau”, pentru ”sprijinul moral si pentru bogatul izvor deinspiratie, fiindu-ne ın acelasi timp si modele de organizare simpla si roditoare”.

S-au adus multumiri cordiale tinerilor diplomati Tiberiu Popoviciu ”pentru bi-nevoitorul dumnisale concurs si pentru ca s-a interesat deosebit de mersul revistei”,sublocotenentului Mihail Focsaneanu, dar si lui Emil Stefanescu, Micu Grunbaum,C.E. Georgescu, N. Segarceanu, precum si elevilor: Ozias Schachter (ce mai apoise va semna Otto Sacter), N. Teodorescu (academicianul de mai tarziu), Gh. Gh.

37

Page 42: Revista (format .pdf, 6.5 MB)

Vasiliu, D. Gh. Nestor, A. Ionescu-Bujor, A. Stoica, Eugen Massler, Paul Bleiweiss,A. Angelescu etc., deja consacrati prin activitatea lor si la alte reviste matematice.

La sfarsitul acestei note, se consemneaza adevaratul crez al tinerilor redactorideosebit de entuziasti, cu dragoste pentru matematica si care notau ca ”prin te-nacitate si munca cinstita, se ajunge la izbanda”, fapt total uitat astazi desocietatea romaneasca aflata ın metastaza profunda ...

In relatarile noastre despre Curierul Matematic, nu putem uita sa subliniem camajoritatea articolelor, notelor si problemelor publicate erau relative la o geometrieelevata, cu notiuni pe care astazi nici cei mai dotati studenti nu le-ar putea manuicu usurinta. Ne-au impresionat apoi, ın mod deosebit, doua mici articole ale luiNicolae A. Segarceanu care ın 1925 si 1926, ıncerca o viziune interdisciplinara ıntreMatematica, Filozofie si Religie. Acestea sunt Consideratii matematice asupra unorprobleme filosofico-religioase, ın care se noteaza: ”Dumnezeu e fiinta ce percepe si apatra dimensiune”, precum si Contributii romanesti la filosofia matematicii, ın carese consemneaza similitudini ıntre V. Conta si H. Poincare, care, printre altele, auaceeasi conceptie asupra infinitului (?) si a sirului de numere naturale.

Cu ıngaduinta cuvenita pentru o revista a elevilor din anii terminali de liceu, darsi pentru matematica ınceputului de veac XX, vom da cateva exemple de problemecu continut de aritmetica, algebra, analiza matematica, combinatorica si geometrieaparute ın Curierul Matematic.

Problema 1 (Problema 45, Ilie Vasiliev). Sa se gaseasca un patrat perfectde patru cifre, astfel ca cifra miilor sa fie radacina patrata a numarului format deultimele sale doua cifre.

(Rezolvata si de Eugen Massler, Gh.Gh. Vasiliu si N. Teodorescu.)

Problema 2 (Problema 62,C.I. Ionescu-Bujor). Sa se arate ca ın orice triunghiexista relatia:

(a/ha) ctgA+ (b/hb) ctgB + (c/hc) ctgC = 2.

(Cu solutii date si de Gh.Gh. Vasiliu si Boris V. Vulpanovici.)

Problema 3 (Problema 95, D.G. Nestor). Ortopolurile laturilor unui patrulatercomplet, fata de triunghiul format de celelalte trei laturi, sunt coliniare.

Problema 4 (Problema 78, D.D. Constantinescu). Sa se calculeze:

limx→a

(a1/6(a1/3 + 2(ax)1/6 − x1/3) + x1/6(a1/3 − 2(ax)1/6 − x1/3))/(x1/3 − a1/3)

(Cu solutii date si de elevii Boris B. Vulpanovici si I. Vasiliev.)

Problema 5 (Problema 24, Gh. Gh. Vasiliu). Sa se calculeze suma seriei:

1 + (x4/5!) + (x8/9!) + (x12/13!) + . . .+ . . .

(Cu o solutie data si de Eugen Massler.)

Problema 6 (Problema 3, Al. Stoenescu). Sa se determine relatiile pe care tre-buie sa le verifice coeficientii a, b, c, astfel ıncat maximul M si minimul m al functiei:

f(x) = x3 + ax2 + bx+ c,

38

Page 43: Revista (format .pdf, 6.5 MB)

sa satisfaca egalitatea αMm+ β(M +m) + γ = 0.(Cu o solutie data si de viitorul acad. prof. univ. dr. N. Teodorescu.)

Problema 7 (Problema 39, N. Botea). Fie functia y(x) = lnxn. Daca y(p)(x) =dpy(x)/dxp, sa se arate ca

limn→∞

n

nX

p=1

1/yp(x)

!= x/ex.

(Pentru care au dat solutii si Al. Mladenovici si N. Teodorescu.)

Problema 8 (Problema 30, I. Fantaneanu). Sa se demonstreze ca

(−1)1/16 =È1/2 + 1/2

È1/2 + 1/2

È1/2 + i

�È1/2− 1/2

È1/2− 1/2

È1/2�,

iar apoi sa se generalizeze.

La aproape un veac de la aparitia acestei a sasea reviste romanesti de matematicaliceala, dar deosebita de suratele ei anterioare pentru ca este scrisa ın ıntregime detineri liceeni din clasele terminale, rememoram dictonul latin ”Historia est magistravitae”, pentru a sublinia ca orice natie trebuie sa-si aminteasca din vreme ın vreme derealizarile ınaintasilor, tocmai pentru a nu-si pierde identitatea si pentru a reınvatadin trecutul ei, ca ındemn spre propasirea generatiilor viitoare. De aceea, privindacum cu mandrie si bucurie intelectuala cele 36 numere ale Curierului Matematiccantemirist, avem deosebita placere de a recomanda profesorilor, studentilor si desigurelevilor, sa reciteasca matematica de calitate a acestui jurnal de ınceput si, luandu-iexemplul de seriozitate, sa ıncerce sa revigoreze multitudinea mediocra a revisteloractuale ale domeniului. Apoi, citandu-l pe Spiru Haret, marele nostru ministrual ınvatamantului, institutie care ın zilele noastre este degradata fara masura, vomındemna la reflectie asupra atentionarii saleCum arata astazi scoala, asa va aratamaine tara, atat de valabila si pentru matematica actuala!

Bibliografie

1. G. St. Andonie - Istoria matematicii ın Romania, v. I-III, Ed. St. si Enc.,Bucuresti, 1965-1967.

2. T. Bırsan, D. Tiba - Recreatii Stiintifice - 125 ani de la aparitie, Recreatii Mat.,anul X, nr. 1, 2008, pg. 1-5.

3. L. Modan - Spiru Haret, reper al spiritualitatii romanesti, Gazeta Mat. Metodica,v. 68, nr. 2, 2001, Bucuresti, pg. 113-118.

4. E. Rogai - Din trecutul revistelor de Matematica, aparute ın tara noastra: Foaiamatematica de la Chisinau, RMT, an XI, nr. 1-2, 1980, Timisoara, pg.5-9.

5. *** - Curierul Matematic, v.1, nr. 1-12, 1925; v.2, nr. 1, 1926.

Eliferie ROGAILaurentiu MODAN

Mihai ROGAI

39

Page 44: Revista (format .pdf, 6.5 MB)

Emil Briul – un dascal de altadata

Emil Briul s-a nascut ın anul 1869, a studiat matematica si a fost cadru didacticla Universitatea din Iasi. In anul 1897 era conferentiar la catedra de calcul diferential,remarcandu-se ca un foarte bun specialist. In anul 1899, pedagogul Ion Gavanescul ılinvita sa faca parte din colectivul de profesori care urmau sa predea la Seminarul Peda-gogic Universitar, nou ınfiintat la Iasi si primul de acest fel din tara. Din colectiv maifaceau parte renumitii profesori Simion Mehedinti-geografie, Ion Simionescu-stiintenaturale, M. Grumazescu-limba romana, Victor Castano-limba franceza si altii.

Seminarul Pedagogic Universitar este echivalentul catedrei de pedagogie dinzilele noastre, cu deosebirea ca se punea mult mai mult accent pe transpunerea ideilorsi teoriilor pedagogice ın practica. De aceea Seminarul Pedagogic Universitar aveapropria lui scoala de aplicatie, unde profesorii specialisti (cel mai adesea universi-tari) tineau ore model, iar studentii care se pregateau pentru meseria de dascal ısidesfasurau practica pedagogica care consta ın foarte multe ore de proba, urmatede discutarea acestora. Studentul practicant pleca de la aceasta scoala cu un dosarsubstantial continand planuri de lectii, exemple de activitati extracurriculare, fise psi-hopedagogice, toate acestea fiindu-i de mare ajutor ın viitoarea lui cariera didactica.Scoala de aplicatie era de asemenea ca un laborator ın care se experimentau metodenoi de predare, noi forme de activitati educative. Activitatea la Seminarul Pedagogicsi implicit la scoala de aplicatie se baza pe urmatoarele idei: primatul educatiei asuprainstructiei, educatia prin munca, accentul pe educatia morala, corelarea cunostintelorde la diferite discipline, dezvoltarea integrala a personalitatii etc. Seminarul Peda-gogic a functionat la ınceput ın casa boierului Aslan din str. Toma Cosma nr. 2,apoi si-a construit propria cladire care si azi se gaseste la aceasta adresa. Scoalade aplicatie, devenita ın 1924 Liceul de aplicatie, a fost o scoala model, bazata peprincipii inspirate ın special din scoala germana, cea mai buna la vremea respectiva.

Activitatea la aceasta scoala a devenit pentru Emil Briul o pasiune de o viata.A predat aici ıntre 1899-1907, 1912-1940, ındrumand totodata practica pedagogica aviitorilor profesori de matematica. Arhiva Seminarului Pedagogic Universitar scoateın evidenta nu doar pe matematicianul E. Briul, ci si pe desavarsitul pedagog E. Briul.Dupa reforma ınvatamantului din 1924, directorul Ion Gavanescul cere MinisteruluiInvatamantului aprobarea pentru un post de director-adjunct care sa se ocupe cumunca educativa si ıl propune pe Emil Briul, acesta fiind un ”profesor foarte bun sicu cea mai mare vechime ın scoala”. Cererea a fost aprobata si din 1928 pana ın anul1940, E. Briul a ramas ın aceasta functie. Ca director-adjunct, s-a ocupat ın specialde disciplina scolara si de activitatea dirigintilor. E. Briul propune ca un elev sa fiecronicarul clasei si sa scrie ıntr-un caiet ce a fost mai interesant ın clasa ın fiecarezi. In problemele controversate, E. Briul gasea solutii de compromis. De exemplu,

40

Page 45: Revista (format .pdf, 6.5 MB)

atunci cand profesorii, ın consiliu, cer pedepsirea elevilor care merg la cinema, rapinddin timpul de ınvatare, directorul adjunct replica: ”Elevii nu mai pot fi opriti de amerge la cinematograf, dar sa li se recomande cand se pot duce si sa consemnezeıntr-un caiet cand si ce au vazut.” Atunci cand ın februarie 1938 (dupa interzicereapartidelor politice), la un elev, s-a gasit o brosura legionara si profesorii se ıntrebauce masura sa se ia pentru pedepsirea elevului respectiv, E. Briul da dovada de tactsi spune ca a cercetat cazul si a aflat ca elevul cumparase brosura de la o librarie,iar ın privinta pedepsei, crede ca pedepsind, profesorii nu vor face decat sa trezeascacuriozitatea celorlalti elevi, iar pe elevul ın cauza l-ar transforma ın erou. Pentru a-iscoate pe elevi de sub influenta politicii, E. Briul propune ca acestia sa fie antrenati ıncat mai multe activitati pregatite chiar de catre ei - de exemplu, disertatii pe diferiteteme si prezentarea lor ın fata clasei. Emil Briul a muncit cu devotament ın aceastascoala pe care a pretuit-o ın egala masura. Drept dovada e faptul ca nu a pregetatsa-si ınscrie proprii fii la acest liceu.

In afara de Ion Gavanescul, S.P.U.a mai avut ca directori pe Paul Nicorescu si peStefan Barsanescu. Ca director-adjunct, Emil Briul a colaborat excelent cu fiecaredintre acestia trei. De aceea la consiliul din 17 ianuarie 1939, spune: ”Am trait subtrei regi si la seminar sub trei directori”, apoi, trecand ın revista istoricul Liceului deaplicatie, subliniaza: ”In traditia acestei scoli este abnegatia. Chiar si cand institutiaa fost scoasa de la buget, am functionat fara salar, pana ce a fost pusa din nou ındrepturile sale”.

In anul 1940, Emil Briul iese la pensie, dar continua sa activeze ın cadrul Comite-tului scolar al Liceului de Aplicatie. Cu toate ca ıncepuse razboiul, Comitetul scolara reusit sa stranga banii necesari pentru a construi aripa de vest a scolii, pentru areface cladirea din mijloc si pentru a turna fundatia la aripa de est. (Din pacate,si azi cladirea de pe strada Toma Cosma nr. 2 are aripa de est neterminata.) Deasemenea, tot ın perioada aceasta, scoala a fost ınzestrata cu mobilier nou, dula-puri pentru biblioteca si chiar un aparat de proiectie epidiascop ”Magister”. Intoate aceste activitati Emil Briul s-a implicat, facand o echipa foarte buna cu di-rectorul St. Barsanescu si cu directorul adjunct D. Gafitanu. Dupa 1948, Liceul deaplicatie devine Liceul clasic, apoi Scoala Medie nr 4 ”Garabet Ibraileanu”, actual-mente Colegiul National ”Garabet Ibraileanu”.

Omagiem ın persoana lui Emil Briul un remarcabil matematician, un pedagogdesavarsit si un ctitor de scoala.

Prof. Magda NEGREAColegiul National ”Garabet Ibraileanu”

41

Page 46: Revista (format .pdf, 6.5 MB)

Concursul ”Recreatii Matematice”

Editia a VIII-a, Muncel, 22-29 august 2010

Clasa a V-a1. O coala de hartie dreptunghiulara este ındoita succesiv astfel ıncat sa se obtina

suprafete egale.a) Cate ındoiri se fac pentru a se obtine mai mult de 100 de suprafete egale?b) Gasiti cel mai mic numar de taieturi cu un foarfece astfel ıncat sa obtinem cel

putin 25 de suprafete egale.2. Pe malul unui rau exista o barca ce suporta maxim 32 kg peste greutatea

barcagiului. Mihaita, care cantareste 30 kg, Ana care cantareste 26 kg, patru catelusicare au fiecare cate un kilogram si 12 iepurasi, fiecare cantarind o treime de kilogram,trebuie sa traverseze raul. Cum pot traversa raul cu barca ın cat mai putine drumuriMihaita, Ana, catelusii si iepurasii?

Recreatii Matematice3. Fie a, b, c cifre nenule ın sistemul zecimal. Notam cu S suma tuturor numerelor

de doua cifre distincte care se pot forma doar cu a, b si c.a) Aflati catul si restul ımpartirii lui S la 11.b) Determinati toate numerele de forma abc, daca S = 528.

Clasele VI-VII1. Directorul Taberei Nationale de Matematica de la Muncel primeste cheile

pentru a descuia usile camerelor unde urmeaza a fi cazati participantii; nici o cheie nuse potriveste la doua usi. El efectueaza 55 de ıncercari pentru a descuia toate usile,deoarece are ghinion din plin si nu nimereste decat ın utimul moment cheia potrivita.Cate usi a descuiat directorul Taberei?

2. Determinati numerele naturale a si n pentru care a2n − 9 = 8 · (9 + 92 + 93 +. . .+ 92009).

Recreatii Matematice

3. Aratati ca oricare ar fi numarul natural nenul n, printre elementele multimii{2n, 2n+ 1, 2n+ 2, . . . , 4n} exista cel putin o putere a lui 2.

Clasa a VIII-a

1. Sa se arate ca 2 ·�

12

1 · 3+

22

3 · 5+

32

5 · 7+ . . .+

10052

2009 · 2011

�< 502

3

4.

Recreatii Matematice

2. a) Aratati ca (a− b)(a2 + ab+ b2) = a3 − b3, oricare ar fi a, b ∈ R.b) Sa se rezolve ın multimea numerelor naturale prime ecuatia x4 + y3 + 10 =

(x+ y − 2)3.

42

Page 47: Revista (format .pdf, 6.5 MB)

3. Fie triunghiul ascutitunghic ABC, A′, B′, C ′ mijloacele laturilor [BC], [CA],[AB], iar D, E, F picioarele ınaltimilor triunghiului ABC corespunzatoare varfurilorA, B si, respectiv, C. Se mai considera punctele A′′, B′′, C ′′ mijloacele ınaltimilor[AD], [BE] si [CF ]. Sa se arate ca segmentele A′A′′, B′B′′ si C ′C ′′ sunt concurente.

Clasa a IX-a1. Lungimile a, b, c ale laturilor unui triunghi satisfac relatia 2

�a8 + b8 + c8

�=�

a4 + b4 + c4�2

. Sa se demonstreze ca triunghiul este dreptunghic.

2. Determinati abcd cu a, c = 0, pentru care

√abcd√

ab+√cd

∈ Q.

Recreatii Matematice

3. Fie ABCD un tetraedru si IA, IB , IC si ID centrele cercurilor ınscrise ın feteleBCD,ACD,ABD, respectiv, ABC. Stiind ca toate muchiile au lungimi numerenaturale, dreptele AIA, BIB , CIC si DID sunt concurente si suma lungimilor tuturormuchiilor este 12, sa se demonstreze ca tetraedrul ABCD este o piramida regulata.

Clasa a X-a1. a) Fie ABCDE un pentagon convex, iar M,N,P,Q,X si Y mijloacele seg-

mentelor [BC], [CD], [DE], [EA], [NQ], respectiv, [MP ]. Sa se demonstreze ca XY ∥

AB si XY =AB

4.

b) In trapezul ABCD cu AB ∥ CD si AB > CD consideram M si N mijloacelediagonalelor [AC], respectiv, [BD]. Sa se arate ca perpendiculara din M pe AD,perpendiculara dinN pe BC si mediatoarea segmentului [AB] sunt drepte concurente.

2. Fie a, b numere reale si multimea A = {{an} − {bn}/n ∈ N}. Sa se arate camultimea A este finita daca si numai daca a−b ∈ Q ({x} reprezinta partea fractionaraa numarului x ∈ R).

Recreatii Matematice

3. Numerele ıntregi se coloreaza cu n culori astfel ıncat daca x, y ∈ Z si |x− y| ∈{2, 3, 5}, atunci x si y au culori diferite. Sa se arate ca n ≥ 4.

Clasa a XI-a1. Fie a1, a2, . . . , an ∈ (0; 1) sau a1, a2, . . . , an ∈ (1;+∞) si functiile injective

f, g : {1, 2, . . . , n} → {1, 2, . . . , n}. Sa se arate ca

�nP

k=1

logakaf(k)

ag(k)

�·�

nPk=1

ak

�≥ n2.

Recreatii Matematice

2. Fie x1, x2, . . . , xn ≥ 0 astfel ıncat x1 + x2 + . . .+ xn = a si

E =Èx1(x2 + 1) +

Èx2(x3 + 1) + . . .+

Èxn(x1 + 1).

Determinati maximul si minimul expresiei E.Recreatii Matematice

43

Page 48: Revista (format .pdf, 6.5 MB)

3. Fie triunghiul ABC cu unghiurile a, b, c astfel ıncat a < b < c. Sa se arate ca

sin a

(a− b)(a− c)+

sin b

(b− c)(b− a)+

sin c

(c− a)(c− b)< 0.

Clasa a XII-a1. Fie matricele A,B,C,D ∈ Mn(R) astfel ıncat AC + BD = In iar AD = BC.

Demonstrati ca CA+DB = In si DA = CB.Recreatii Matematice

2. Fie sirul (xn)n∈N definit prin xn+1 = xn + e−xn ,∀n ∈ N, iar x0 ∈ R. Sa se

arate ca limn→∞

xn

lnn= 1.

3. Consideram sirul (xn)n∈N∗ definit prin x1 = 1 si xn+1 = 1 +n

xn,

∀n ∈ N∗. Determinati valorile lui n pentru care xn este numar natural.

Concursul de matematica ,,Gaudeamus”

Editia a II-a, Iasi, 30 octombrie 2010

Clasa a X-aProblema 1. Daca O este centrul cercului circumscris triunghiului ABC si H

este ortocentrul sau, demonstrati ca−−→OH =

−→OA+

−−→OB +

−−→OC.

2) Se dau: un punct O, un numar strict pozitiv r si un vector u. Construiti trei

vectori−→OA,

−−→OB,

−−→OC de aceeasi marime r, stiind ca suma lor este vectorul dat u.

Problema 2. Se considera P = Z× Z multimea punctelor de coordonate ıntregidin plan. Spunem ca un punct A ∈ P este vizibil (din punctul O = (0, 0)) dacasegmentul [OA] nu are puncte interioare apartinand multimii P.

i) Sa se arate ca punctul A ∈ P este vizibil daca si numai daca abscisa si ordonatasa sunt numere prime ıntre ele.

ii) Sa se arate ca daca cercul C(O, r), cu r ∈ N∗, contine puncte vizibile, atuncir = 4k + 1, pentru un k ∈ N.

iii) Sa se arate ca daca p este numar prim, atunci cercul C(O, p) contine punctevizibile daca si numai daca p = 4k + 1, pentru un k ∈ N∗.

Problema 3. Fie n ∈ N∗ si a1, a2, . . . an ∈ [0,∞) astfel ıncat S := a1 + a2 +. . . + an < 1. Sa se arate ca 1 + S ≤ (1 + a1)(1 + a2) . . . (1 + an) ≤ 1

1−S si 1 − S ≤(1− a1)(1− a2) . . . (1− an) ≤ 1

1+S . Cand ating egalitatile?

Clasa a XI-aProblema 1. Determinati toate multimile finite A ⊂ R care satisfac proprietatea

a ∈ A =⇒ a2− | a | +1 ∈ A.

44

Page 49: Revista (format .pdf, 6.5 MB)

Problema 2. Fie triunghiul ABC, vectorii x, y, z si punctele M,N,P astfel ıncat−−→AM = λx,

−−→BN = λy si

−−→CP = λz, λ ∈ R+. Se cere locul geometric al centrului de

greutate Q al triunghiului (eventual degenerat) MNP , cand λ variaza.

Problema 3. Fie f : R →[0, 1] si a > 0 a.ı. pentru orice x ∈ R are loc relatia

(1) f(x+ a) =1

2+È(f(x)− f2(x).

Sa se arate ca f este periodica si sa se dea un exemplu de functie care verifica (1).

Clasa a XII-aProblema 1. Se noteaza cu fn derivata de ordin n (n ≥ 1) a functiei (x2 − 1)n.a) Calculati f1, f2 si f3.b) Aratati ca exista exact n valori −1 < x1 < x2 < . . . < xn < 1 pentru care

fn(xk) = 0, k = 1, 2, . . . , n.c) Acceptand ca are loc relatia de recurenta fn+1(x) = (anx+bn)fn(x)+cnfn−1(x),

determinati coeficientii an, bn cn.

Problema 2. Pe o insula traieste un grup de cameleoni de trei culori: 17 suntrosii, 15 sunt verzi, iar 13 sunt galbeni. Daca doi cameleoni de culori diferite seıntalnesc, atunci fiecare ısi schimba culoarea ın cea de a treia. Schimbarea culoriiunui cameleon se petrece doar ın aceasta situatie.

Se poate sa se ajunga la situatia ca toti cameleonii au aceeasi culoare?

Problema 3. Fie M multimea matricelor din Mn(C) de forma

A(a1, a2, . . . , an) =

�a1 a2 · · · anan a1 · · · an−1

......

...a2 a3 · · · a1

�.

a) Aratati ca B ∈ Mn(C) comuta cu toate matricele din M daca si numai dacaB ∈ M.

b) Calculati A(1, 1, 0, . . . , 0)n.

Concursul ,,Sperante Olimpice”

Editia a X-a, Pascani, 6 septembrie 2010

Clasa a III-a1. a) Aranjati numerele 10, 20, 30, 40, 50, 60 ın triunghiul alaturat, astfel ıncat

suma de pe fiecare latura sa fie 100. Gasiti cel putin doua posi-bilitati.

b) Stiind ca ∆ + ∆ + 6 = ∆ + ∆ + ∆ + ∆, ce cifra ascundesimbolul ∆?

2. a) Copiii au masurat lungimea terenului de nisip cu pasii.

45

Page 50: Revista (format .pdf, 6.5 MB)

Ana a facut 15 pasi egali, Anca 17, Dana 12 si Ionut 14. Care dintre copii are pasulmai mare?

b) In parc sunt trei adulti, fiecare ınsotit de cate trei copii. Fiecare copil plimbacate un caine si fiecare caine este ınconjurat de cate trei catelusi. Cate picioare suntın parc?

3. Radu, Nicu si Mihaela s-au pregatit pentru concursul Sperante Olimpice. Radusi Nicu au rezolvat 63 probleme, Nicu si Mihaela 73, iar Radu si Mihaela 64. Cateprobleme a rezolvat fiecare copil?

Clasa a IV-a1. 1. a) Folositi de 7 ori cifra 7 si diferite operatii aritmetice pentru a obtine

numarul 100.

b) Aflati m din egalitatea

3333− {aa : a+ [(n− bbbb : b) + 333 : 3] · 3 + 333 + 33 : 3} · 3 = 1269.

2. a) De cate ori se gaseste numarul 9 ın intervalul de la 5 la 100?

b) Intr-un depozit s-au adus lamai, portocale si banane. Lamai erau cat portocalesi banane la un loc, adica 37 de lazi a 15 kg fiecare. Daca ımpartim numerele carereprezinta cantitatea (ın kilograme) de portocale si cea de banane, obtinem catul 5 sirestul egal cu ıntreitul catului. Aflati cele trei cantitati de fructe.

3. a) In 5 vase se aseaza 31 de lalele. In fiecare vas se gaseste un numar impar delalele si nu exista vase cu acelasi numar de flori. Gasiti cel putin doua modalitati deasezare a lalelelor.

b) Suma a sase numere este 1433. Primele doua numere sunt pare consecu-tive. Suma dintre al treilea si al patrulea este 360, iar unul este a cincea partedin celalalt. Diferenta ultimelor doua numere este 321, iar unul din ele este sfertulceluilalt. Determinati cele sase numere.

Clasa a V-a1. a) Determinati numerele naturale a, b, c, stiind ca 2a + 2b−a + 2a+b+c = 1153.

b) Daca x, y, z sunt numere naturale astfel ıncat numerele 2x, 2y, 2z dau resturidistincte la ımpartirea cu 7, aratati ca 2x + 2y + 2z se divide cu 7.

2. Fie a = 2n+1 · 2n+2 · 3 si b = 2n+2 · 5n+2 · 7.a) Comparati numerele a si b.

b) Aflati suma cifrelor numarului a+ b.

c) Aratati ca 2a+ 13 nu poate fi patrat perfect.

3. a) Demonstrati ca numarul N = 2010 + 2(1 + 2 + 3 + . . . + 2009) este patratperfect.

46

Page 51: Revista (format .pdf, 6.5 MB)

b) Reconstituiti adunarea:

M A T E M A T I C A +M A T E M A T I C

M A T E M A T IM A T E M A T

M A T E M AM A T E M

M A T EM A T

M AM

1 0 0 9 7 6 7 6 1 1 7

Clasa a VI-a1. a) Aratati ca (a− 1)(1 + a+ a2 + . . .+ an) = an+1 − 1 pentru a ∈ N, a ≥ 2 si

n ∈ N.b) Rezolvati ecuatia 5 + 5 · 6 + 5 · 62 + . . .+ 5 · 62008 = x2009 − 1.

c) Aratati ca62009 − 6

35∈ N.

2. Bananele dintr-o cutie, pentru a fi comercializate, sunt puse ın pungi. Dacase pun cate 8 banane, ıntr-o punga raman 5 banane, daca se pun cate 10 banane,ıntr-o punga raman 3 banane, iar daca se pun cate 18 banane, ıntr-o punga raman 15banane. Sa se afle numarul maxim de banane din cutie, stiind ca acesta nu depaseste2010.

3. a) Demonstrati ca numarul N = x2+ y+x+ y2 este par, oricare ar fi numerelenaturale x si y.

b) Determinati numerele naturale x si y pentru care x2+y+x+y2−13 = 4(x−3y)2 .

Clasa a VII-a

1. Se dau numerele naturale nenule a si b si numerele rationale x =2a+ b

3b+ 2,

y =3b+ 2

8si z =

8

2a+ b.

a) Determinati a si b, daca x, y, z sunt simultan numere naturale.

b) Demonstrati cax

4,y

2si 4z nu pot fi simultan numere naturale.

2. a) Demonstrati ca n2 − (n− 1)(n+ 1) > 0, oricare ar fi numarul n ∈ N∗.

b) Aratati ca1

52+

1

72+ . . .+

1

20112< 0, 125.

c) Pe fiecare patratel al unei table 7 × 7 (asemanatoare celei de sah) se afla cateo broscuta. La o bataie din palme, fiecare broscuta sare pe un patratel care are olatura comuna cu patratelul pe care se afla ea initial. Aratati ca dupa o bataie dinpalme, dupa ce broscutele se aseaza, ramane cel putin un patratel neocupat de vreobroscuta.

3. Fie △ABC un triunghi echilateral, iar P un punct pe latura AC. Bisectoareaunghiului ]ABP taie paralela prin A la BC in Q. Demonstrati ca BP = CP +AQ.

47

Page 52: Revista (format .pdf, 6.5 MB)

Clasa a VIII-a1. a) Aratati ca 98|52010 − 32010.b) Fie a, b, c numere ıntregi cu proprietatea ca ab+ac+bc = 0. Aratati ca numarul

3abc se poate scrie ca suma a patru cuburi perfecte.

2. a) Demonstrati ca ecuatia 3x2 + 2x = 12y2 − 20y nu are solutii ın N∗.b) Aratati ca ecuatia de mai sus are o infinitate de solutii ın Q.

3. a) Fie ABC un triunghi de laturi a, b, c astfel ıncat b+ c = a√2. Demonstrati

ca triunghiul este ascutitunghic daca si numai daca b si c sunt distincte si se afla ın

intervalul

�a√2

4,3a

√2

4

�.

b) Se considera cubul ABCDA′B′C ′D′ si fie M mijlocul muchiei BB′. Daca{E} = AC ∩BD, demonstrati ca EM⊥(ACD′).

Cam cu 300 ani ınainte de Hristos matematicianul alexandrin Euclid a scris ocarte monumentala intitulata Elemente. Aceasta carte, una din cele mai grandioasemanifestari ale culturii eline, prin continutul riguros si forma perfecta s-a impus ıntr-atat generatiilor urmatoare ıncat si astazi formeaza scheletul geometriei elementarescolare. In unele tari geometria s-a predat chiar dupa textul lui Euclid si ın mai toatetarile cartile de geometrie scolara sunt compilari, mai mult sau mai putin fidele, aleElementelor. Poetul francez Sully Prudhomme, laureat al premiului Nobel ın anul1901, a cantat astfel aceasta opera:

J′ouvre un Euclide avec amour*,Il propose, il prouve et j′ecoute,Et je suis inonde de jour.

L′evidence, eclair de l′etude,Jaillit et me laisse enchante;Je savoure la certitude,Mon seul vrai bonheur, ma sante!

Doua mii de ani de framantari stiintifice n-au putut sa clatine ıntru nimic operageometrica a lui Euclid, care a continuat sa joace un rol unic, exclusiv, pana laınceputul secolului al XIX-lea, cand matematicienii au ınceput sa scruteze mai cusucces primele principii, axiomele, ce stau la baza geometriei, si au adus la luminageometrii noi (geometriile neeuclidiene).

(dupa Al. Myller – Evolutia ideei de spatiu, RevistaStiintifica “V. Adamachi, XII (1925-1926), nr. 4, pp. 164-168)

*Deschid pe Euclid cu inima preaplina,/ El enunta si demonstreaza, iar eu ascult/ Si sunt inun-dat de lumina.// Evidenta, strafulgerare a studiului,/ Tasneste si ma-ncanta;/ Eu simt savoareacertitudinii,/ Singura si deplina mea fericire, sanatatea mea!

48

Page 53: Revista (format .pdf, 6.5 MB)

Solutiile problemelor propuse ın nr. 1/2010

Clasele primareP.184. Vreau sa pun ıntr-o cutie bile albe si verzi, ın total 10, astfel ıncat bile

albe sa fie cel mult 6. In cate moduri pot face acest lucru?(Clasa I) Inst. Maria Racu, Iasi

Solutie.Bile albe 6 5 4 3 2 1Bile verzi 4 5 6 7 8 9

P.185. Ce litera urmeaza ın ınsiruirea logica VKUJT...?(Clasa I) Andreea Amarandei, studenta, Iasi

Solutie. In alfabetul limbii romane U este ın fata lui V , iar T ın fata lui U . Inscrierea V KUJT, literele V,U si T sunt scrise ın ordine invers alfabetica. Urmatoarealitera este I, deoarece sirul K,J, I trebuie sa respecte regula sirului V,U, T .

P.186. Completati dreptunghiurile de mai jos cu numere asa ıncat suma nu-merelor scrise ın oricare trei dreptunghiuri alaturate sa fie aceeasi. Ce observati?

35 65 35

(Clasa a II-a) Alexandru Chiriac, student, IasiSolutie. 35 a 65 b c d 35Scriind proprietatile numerelor din casete, avem 35 + a + 65 = a + 65 + b =

65+b+c = b+c+d = c+d+35, de unde deducem b = 35, d = 65 si a = c. Observamca ın locul lui a putem sa punem orice numar natural.

P.187. Soricelul Chit a primit un zar de la matusa Mit. El a aruncat zarul depatru ori, obtinand ın total 21 de puncte. Stiind ca la primele doua aruncari a obtinutın total 9 puncte, aflati cat a obtinut la fiecare aruncare. (Gasiti toate posibilitatile!)(Clasa a II-a) Ioana Maria Popa, eleva, Iasi

Solutie. La ultimele doua aruncari a obtinut, ın total, 21− 9 = 12 puncte, decia dat de fiecare data 6. La primele doua aruncari a obtinut (6, 3) sau (5, 4) sau (4, 5)sau (3, 6).

P.188. In exercitiul a+ a : a = . . ., exista o valoare a lui a pentru care putem saefectuam operatiile ın ordinea scrisa, fara a modifica rezultatul corect?(Clasa a III-a) Ionela Baragan, studenta, Iasi

Solutie. Trebuie sa-l aflam pe a astfel ıncat a + a : a = (a + a) : a, ceea ceınseamna ca a+ 1 = 2, de unde a = 1.

P.189. Aflati numarul natural a stiind ca, daca se ımparte 25 la 8 − 3 × a, seobtine restul 1.(Clasa a III-a) Mariana Nastasia, eleva, Iasi

Solutie. Avem ca 25 = (8−3×a)× c+1, adica 24 = (8−3×a)× c. Tinand contde conditia restului, sunt posibile situatiile: 8−3×a = 2, 8−3×a = 3, 8−3×a = 4,8− 3× a = 6 si 8− 3× a = 8. Obtinem solutiile a = 0 sau a = 2.

49

Page 54: Revista (format .pdf, 6.5 MB)

P.190. In gradina casei mele sunt cativa pomi. Daca ar fi de patru ori mai multidecat sunt, atunci ar depasi numarul 20 cu atat cat lipseste, de fapt, pentru a fi 20.Cati pomi sunt ın gradina?(Clasa a III-a) Inst. Dumitru Paraiala, Iasi

Solutie. Daca a este numarul pomilor, atunci 4a − 20 = 20 − a. Deducem caa = 8.

?

a - ?

a - ?

a - b

P.191. Compuneti o problema care sa se rezolve dupa schemaalaturata, cu numerele a si b convenabil alese.(Clasa a III-a) Amalia Cantemir, eleva, Iasi

Solutie. ,,Trei copii aleg trei numere mai mici decat un numar initial,a. Primul copil alege numarul b, al doilea copil alege un numar egalcu diferenta dintre numarul initial si diferenta dintre numarul initial sinumarul ales de primul copil, iar al treilea copil alege numarul egal cu diferenta dintrenumarul initial si numarul ales de al doilea copil. Ce numar a ales al treilea copil?”

P.192. Bunica are mere si pere. Daca mi-ar da un sfert din numarul merelorsi o optime din numarul perelor, as avea 35 de fructe. Daca mi-ar da o optime dinnumarul merelor si o patrime din numarul perelor, as avea 40 fructe. Cate fructe arebunica?(Clasa a IV-a) Mihaela Galca, eleva, Iasi

Solutie. Notam cu 8a numarul merelor si cu 8b numarul perelor bunicii. Primadata, bunica da 2a+ b = 35 fructe, iar a doua oara da a+ 2b = 40 fructe. Adunandcele doua egalitati, obtinem ca 3a+3b = 75, deci a+b = 25. Atunci a = 35−25 = 10,iar b = 40 − 25 = 15. Bunica are 8 · 10 = 80 mere si 8 · 15 = 120 pere, ın total 200fructe.

P.193. Mai multe perechi, formate din cate o fata si cate un baiat, culeg alune.In fiecare pereche, alunele culese de baiat sunt fie de patru ori mai multe, fie de patruori mai putine decat cele culese de fata. Numarul alunelor culese ımpreuna de fete side baieti poate fi 2009? Dar 2010?(Clasa a IV-a) Mihaela Obreja si Ioan Lungu, Vaslui

Solutie. Numarul de alune culese de fiecare pereche se ımparte exact la 5, decisi numarul total de alune trebuie sa se ımparta exact la 5. Elevii nu pot culegeımpreuna 2009 alune. Ei pot culege ınsa ımpreuna 2010 alune: daca, de exemplu,avem 6 perechi, iar ın fiecare pereche fata culegere 268 alune si baiatul 67 alune (sauinvers), obtinem o situatie favorabila.

P.194. Aratati ca exista un singur sir format din zece numere naturale consecutiveastfel ıncat suma a opt dintre numere sa fie egala cu dublul sumei celorlaltor doua.(Clasa a IV-a) Petru Asaftei, Iasi

Solutie. Sirul de numere 0, 1, 2, . . . , 9 ındeplineste conditia, deoarece 0 + 1 + 2 +3+4+5+7+8 = 2(6+9). Fie sirul a+1, a+2, . . . , a+10, cu a ≥ 0. Suma primelor 8numere este 8a+36, iar dublul sumei ultimelor doua este 4a+38. Daca a = 0 atunci8a + 36 > 4a + 38, iar daca ınlocuim unul din ultimele doua numere cu unul maimic, suma celorlaltor 8 numere se mareste. Pentru a = 0 avem sirul 1, 2, 3, . . . , 10 cusuma 55. In acest caz nu este posibila partitia ceruta de problema, deoarece 55 nu seımaprte exact la 3.

50

Page 55: Revista (format .pdf, 6.5 MB)

P.195. Trei frati, Ionut, Andrei si Mihai, primesc lunar cate o aceeasi sumade bani de la bunicul lor. Pe ascuns, bunica le da si ea aceeasi suma. Odata, unuldintre cei trei a spart un geam. Bunicul, crezandu-l vinovat pe Ionut, a ımpartit baniicuveniti lui celorlalti doi. Bunica a procedat la fel, ınsa a crezut ca cel vinovat esteMihai. Andrei, stiind ca el a spart geamul, a ımpartit jumatate din suma sa celor doifrati si a constatat ca ramane cu 60 de lei mai putin decat Ionut si Mihai la un loc.Ce suma de bani primea fiecare nepot de la bunicul?(Clasa a IV-a) Cosmin Serbanescu, student, Iasi

Solutie. Fie 4a suma pe care fiecare nepot o primea de la bunicul, ın conditiinormale. Andrei primeste 4a + 20 = 6a lei de la bunicul si ınca 6a lei de la bunica,ın total 12a lei, din care da cate 3a lei celor doi frati, ramanand ın final cu 6a lei.Ionut primeste 4a + 2a = 6a lei de la bunica si 3a lei de la Andrei, deci are ın total9a lei. Mihai primeste 4a + 2a = 6a lei de la bunicul si 3a lei de la Andrei, deci areın total 9a lei. Avem ca 6a = 9a+9a− 60, de unde a = 5. In concluzie, fiecare nepotprimeste lunar de la bunicul cate 20 lei.

Clasa a V-aV.116. Se considera numarul a = (2n · 5n+1 + 4) : 36, n ∈ N∗. Determinati

valorile lui n pentru care a este numar natural, a carui scriere ın baza 10 are toatecifrele distincte.

Andrei Nedelcu, IasiSolutie. Observam ca a = 5 00 . . . 00| {z }

n−1 zerouri

4 : 36. Daca n = 1, atunci a /∈ N. Daca

n = 2, atunci a = 504 : 36 = 14, numar care are cifrele distincte. Daca n = 3, avemca a = 5004 : 36 = 139, situatie care din nou este convenabila. Pentru n ≥ 4, obtinemca a = 13 8 . . . 88| {z }

n−3 de 8

9, iar acest numar are cifrele distincte daca si numai daca n = 4.

In concluzie, valorile cautate ale lui n sunt 2, 3 si 4.

V.117. Aratati ca A = 61001 se poate scrie ca diferenta a doua patrate perfecte.Damian Marinescu, Targoviste

Solutie. Avem: A = 6998 · 63 = 6998(225− 9) = (6499 · 15)2 − (6499 · 3)2.V.118. Determinati numerele naturale a si n pentru care a2n − 9 = 8(9 + 92 +

93 + . . .+ 92009).Gabriela Popa, Iasi

Solutie. Daca S = 9+92+93+. . .+92009, atunci 9S = 92+93+. . .+92009+92010,de unde 8S = 9S − S = 92010 − 9. Rezulta ca a2n − 9 = 92010 − 9, deci a2n = 92010,adica an = 32010. Obtinem ca a = 3k, n = 2010

k , unde k este un divizor al lui 2010.Cum 2010 = 2 · 3 · 5 · 67 are 16 divizori, problema are 16 solutii.

V.119. Fie n ∈ N un numar a carui scriere ın baza 10 este de forma . . . 55.a) Aratati ca (n− 5)(n+ 5) se divide cu 1000.b) Aflati ultimele trei cifre ale lui n2.

Mihai Craciun, PascaniSolutie. a) Fie n = 100x + 55, cu x ∈ N; atunci (n − 5)(n + 5) = (100x +

50)(100x+ 60) = 50(2x+ 1) · 20(5x+ 3) = 1000(2x+ 1)(5x+ 3).

51

Page 56: Revista (format .pdf, 6.5 MB)

b) Folosind a), deducem ca n2 − 25 = . . . 000, prin urmare n2 = . . . 025.

V.120. Consideram numarul natural a = 12345 . . . 9899. Aflati restul ımpartiriilui a prin 45.

Elena Iurea, IasiSolutie. Restul ımpartirii lui a la 5 este 4, deci a = 5k + 4, k ∈ N∗. Deoarece

suma cifrelor lui a este 20(1 + 2 + . . . + 9) = 900, rezulta ca a se divide cu 9, adicaa = 9n, n ∈ N∗. Din 9n = 5k + 4, deducem ca 4(n − 1) = 5(k − n), prin urmaren − 1 = 5t, iar k − n = 4t, t ∈ N∗. Astfel, a = 45t + 9, t ∈ N∗, deci restul ımpartiriilui a prin 45 este 9.

V.121. Aratati ca1

2 · 3 · 4+

2

3 · 4 · 5+ . . .+

2010

2011 · 2012 · 2013<

1

3.

Tinuta Bejan, IasiSolutie. Daca S este suma din membrul stang al inegalitatii din enunt, atunci

S <1

3 · 4+

1

4 · 5+ . . .+

1

2012 · 2013=

�1

3− 1

4

�+

�1

4− 1

5

�+ . . .+

�1

2012− 1

2013

�=

1

3− 1

2013<

1

3.

V.122. Cate fractii ireductibile de forma20xy

2yexista?

Diana Gregoretti, GalatiSolutie. Daca y este o cifra para, fractia se va simplifica prin 2. Daca y = 5,

fractia se va simplifica prin 5. Daca y = 1, fractia20x1

21este reductibila daca se

simplifica prin 3 (deci cand x ∈ {0, 3, 6, 9}) sau prin 7 (adica pentru x = 5); ın acestcaz, raman cinci valori ale lui x pentru care fractia este ireductibila. Daca y = 3,

fractia20x3

23este reductibila doar pentru x = 9, deci este ireductibila pentru noua

valori ale lui x. Daca y = 7, fractia20x7

27este reductibila daca si numai daca se

simplifica prin 3, deci cand x ∈ {0, 3, 6, 9}; raman sase valori ale lui x pentru care este

ireductibila. In sfarsit, daca y = 9, cum 29 este prim, fractia20x9

29este reductibila

doar cand 29��20x9, fapt care se petrece numai pentru x = 5; raman noua fractii

ireductibile ın acest caz. In total, vom avea 5 + 9 + 6 + 9 = 29 fractii ireductibile caın enunt.

Clasa a VI-aVI.116. Se considera unghiul ÕAOB cu masura de 126◦ si semidreptele (OM1, (OM2,

. . . , (OMn−1 interioare lui, astfel ıncat interioarele unghiurilor ÖAOM1, ØM1OM2, . . . ,ÙMn−1OB sunt disjuncte doua cate doua, iar m(ÖAOM1) = 2◦, m(ØM1OM2) = (22)◦,

. . . ,m(ÙMn−1OB) = (2n)◦. Daca (OM este bisectoarea unghiului ÖAOM4, determinati

masura complementului lui ÖAOM .Catalina Dragan, Galati

52

Page 57: Revista (format .pdf, 6.5 MB)

Solutie. Cum m(ÕAOB) = m(ÖAOM1)+m(ØM1OM2)+ . . .+m(ÙMn−1OB), rezultaca 126 = 2 + 22 + . . . + 2n, deci 2n+1 − 2 = 126, de unde n = 6. Deducem ca

m(ÖAOM4) = 30◦, deci m(ÖAOM) = 15◦, iar complementul unghiului de 15◦ aremasura de 75◦.

VI.117. Fie I centrul cercului ınscris ın triunghiul ABC, iar x = m(ÕBIC) −m( bA), y = m(ÔAIC) − m(ÒB), z = m(ÔAIB) − m(ÒC). Aratati ca numerele x, y si zsunt masurile unghiurilor unui triunghi ascutitunghic.

Constantin Apostol, Rm. Sarat

Solutie. Observam ca m(ÕBIC) = 180◦ − 12 [m(ÒB) + m(ÒC)] = 180◦ − 1

2 [180◦ −

m( bA)] = 90◦ + 12 · m( bA), prin urmare x = 90◦ − 1

2 · m( bA). Analog obtinem ca

y = 90◦ − 12 ·m(ÒB), iar z = 90◦ − 1

2m(ÒC). Este evident atunci ca x, y, z sunt masuri

ale unor unghiuri ascutite si, cum x+y+z = 3 ·90◦− 12 [m( bA)+m(ÒB)+m(ÒC)] = 180◦,

concluzia se impune.

VI.118. In triunghiul isoscel ABC, cu m( bA) = 120◦, se noteaza cu D mijlocullaturii [AB]. Perpendiculara din D pe BC intersecteaza dreptele AC si BC ın E,

respectiv F. Bisectoarea unghiului ÕDEA taie BC ın G. Aratati ca BC = 4 · FG.

Catalin Budeanu, Iasi

Solutie. Observam ca m(ÒB) = m(ÒC) = 30◦, m(ÕEAD) = 180◦ − 120◦ = 60◦,

A

B C

D

E

F G

m(ÕFEC) = 180◦ − 90◦ − 30◦ = 60◦, m(ÕEDA) = 180◦ −m(ÕDEA) − m(ÕEAD) = 60◦, iar m(ÕEDB) = 180◦ −m(ÕEDA) = 120◦. Atunci triunghiul ADE este echilateral;rezulta ca ED = DA = DB, prin urmare △DEB este isos-

cel. Obtinem ca m(ÕDBE) = m(ÕDEB) = 30◦, de unde

m(ÕBEG) = 30◦+30◦ = 60◦, iarm(ÕEBC) = 30◦+30◦ = 60◦.

Astfel, △BEG este echilateral, iar bisectoarea EF a unghiului ÕBEG va fi mediana

laturii BG, deci FG = 12BG. Cum m(ÕGEC) = m(ÕGCE) = 30◦, triunghiul GEC este

isoscel, cu GE = GC. Obtinem ca CG = EG = BG, adica BG = 12BC. In concluzie,

FG = 14BC.

VI.119. Un numar natural N se termina ın 0 si are exact 323 de divizori. Aflatiultimele 16 cifre ale numarului N .

Mirela Obreja, Vaslui

Solutie. Descompunerea ın factori primi a numarului 323 este 17 · 19; rezulta caN = p322 sau N = p16 · q18, cu p, q prime. Cum N se termina ın 0, atunci 2|N si5|N , deci prima situatie nu este posibila. Ramane ca N = 216 · 518 = 25 00 . . . 00| {z }

16 zerouri

sau

N = 218 · 516 = 400 . . . 00| {z }16 zerouri

. In ambele cazuri, ultimele 16 cifre ale lui N sunt zerouri.

VI.120. Demonstrati ca numarul A = 12 +22 +32 + . . .+20102 − 2000 se dividecu 3.

Nicolae Ivaschescu, Craiova

53

Page 58: Revista (format .pdf, 6.5 MB)

Solutia 1. Numerele divizibile cu 3, ridicate la patrat, raman divizibile cu 3, iarcele care nu sunt divizibile cu 3, prin ridicare la patrat, vor fi de forma M3 + 1. Cum2000 = M3 + 2, avem ca A = (M3 + 1) + (M3 + 1) +M3 + . . . + (M3 + 1) + (M3 +1) +M3 − (M3 + 2) = M3 + 1340− 2 = M3 + 1338 = M3.

Solutia 2. Se stie ca 12 + 22 + 32 + . . . + n2 = n(n+1)(2n+1)6 . Rezulta ca A =

2010·2011·40216 − 2000 = 2 708 885 385, iar acest numar are suma cifrelor 54, deci este

divizibil cu 3.

VI.121. Fie n ∈ N∗ si a1, a2, . . . , an numere naturale consecutive. Aratati casuma S = a1 + a2 + . . .+ an se divide cu n daca si numai daca n este numar impar.

Andrei Pasa, elev, IasiSolutie. Daca n = 2k + 1 este impar, se arata ca S = n(a1 + k), numar care se

divide cu n. Reciproc, daca am presupune prin absurd ca n ar fi par, n = 2k, amobtine ca S = k(2a1 +2k− 1), unde paranteza este numar impar. Rezulta ca S nu sedivide cu 2k, contradictie.

VI.122. Vom spune ca un numar natural este anti-Goldbach daca poate fi scrisca suma de doua numere compuse. Determinati toate numerele anti-Goldbach.

Ionel Nechifor, IasiSolutie. Prin verificari directe, se observa ca numerele 0, 1, 2, 3, 4, 5, 6, 7, 9 si 11

nu pot fi scrise ca suma de doua numere compuse. Avem ca 8 = 4 + 4, 10 = 6 + 4,2k = 6 + 2(k − 3), ∀k ≥ 6 si 2k + 1 = 9 + 2(k − 4), ∀k ≥ 6, prin urmare toatenumerele naturale, ın afara celor enumerate initial, pot fi scrise ca suma de douanumere compuse.

Clasa a VII-a

VII.116. Calculati suma S =

����12 − 2

3

����+ ����23 − 3

4

����+ . . .+

����20082009− 2009

2010

����.Daniela Munteanu, Iasi

Solutia 1. Se arata usor can

n+ 1<

n+ 1

n+ 2, ∀n ∈ N∗. Atunci diferentele din

interiorul fiecarui modul sunt negative, prin urmare S =

�2

3− 1

2

�+

�3

4− 2

3

�+ . . .+�

2009

2010− 2008

2009

�=

2009

2010− 1

2=

502

1005.

Solutia 2. Observam ca

���� n

n+ 1− n+ 1

n+ 2

���� = ���� −1

(n+ 1)(n+ 2)

���� = (n+ 2)− (n+ 1)

(n+ 1)(n+ 2)=

1

n+ 1− 1

n+ 2, ∀n ∈ N∗. Dam lui n valori de la 1 la 2008 si, prin sumarea egalitatilor

obtinute si dupa reducerea termenilor, obtinem ca S =1

2− 1

2010=

502

1005.

VII.117. Fie x, y numere reale astfel ıncat x > 2011, iar xy = x+ y. Aratati ca

partea fractionara a lui y este mai mica decat1

2010.

Claudiu Stefan Popa, IasiSolutie. Din xy − x− y + 1 = 1, deducem ca (x− 1)(y − 1) = 1. Cum x > 2011,

atunci x−1 > 2010. Produsul (x−1)(y−1) fiind pozitiv si factorul x−1 de asemenea,

54

Page 59: Revista (format .pdf, 6.5 MB)

rezulta ca y − 1 > 0. Deducem ca 0 < y − 1 <1

2010, de unde 1 < y < 1

1

2010, prin

urmare [y] = 1, iar {y} <1

2010.

VII.118. Aratati ca x2010+1 ≥ x1010+x1000,∀x ∈ R. Cand se atinge egalitatea?Catalin Melinte, student, Iasi

Solutie. Inegalitatea din enunt se scrie echivalent sub forma (x1010 − 1)(x1000 −1) ≥ 0. Daca −1 < x < 1, ambele paranteze sunt negative, deci produsul este pozitiv.Daca x > 1 sau x < −1, ambele paranteze sunt pozitive, deci produsul este pozitiv.Daca x ∈ {−1, 1}, produsul este nul, prin urmare se atinge egalitatea ın inegalitateadin enunt.

VII.119. Consideram numarul natural A = 5n + 2 · 3n−1 + 1, n ∈ N∗.a) Demonstrati ca A se divide cu 8, oricare ar fi n ∈ N∗.b) Determinati n ∈ N∗ pentru care A se divide cu 40.

Ciprian Baghiu, IasiSolutie. a) Daca n = 2p, p ∈ N∗, atunci A = 25p + 6 · 9p−1 + 1 = (M8 + 1) +

6(M8 + 1) + 1 = M8. Daca n = 2p + 1, p ∈ N, atunci A = 5 · 25p + 2 · 9p + 1 =5(M8 + 1) + 2(M8 + 1) + 1 = M8.

b) Cum A...8, rezulta ca A

...40 daca si numai daca A...5. Insa 5n

...5, ∀n ∈ N∗ si

2 · 3n−1 + 1 este numar impar; atunci A...5 ⇔ U(2 · 3n−1 + 1) = 5 ⇔ U(3n−1) ∈

{2, 7} ⇔ n = 4p, n ∈ N∗.

VII.120. Un poligon convex are 170 de diagonale. Masurile unghiurilor sale seexprima, ın grade, prin numere naturale impare. Demonstrati ca poligonul are celputin doua unghiuri congruente.

Catalin Budeanu, Iasi

Solutie. Un poligon convex cu n laturi aren(n− 3)

2diagonale. Din

n(n− 3)

2=

170, obtinem ca n = 20. Suma masurilor unghiurilor poligonului va fi 180◦(n − 2) =3240◦. Fie x masura, ın grade, a celui mai mare dintre unghiuri. Daca am presupune,prin absurd, ca poligonul nu are unghiuri congruente, cele 19 unghiuri ramase ar aveamasurile cel mult egale cu x− 2, x− 4, . . . , x− 38. Suma lor ar fi cel mult 20x− 2(1+2 + . . . + 19) = 20(x − 19) ≤ 20(179 − 19) = 20 · 160 = 3200 < 3240, contradictie.Astfel, ramane adevarat ca poligonul are cel putin doua unghiuri congruente.

AB C

A

B

C

M

NP

ZY

X

VII.121. In triunghiul ascutitunghic ABC, notam cuX,Y si Z mijloacele ınaltimilor [AA′], [BB′], respectiv [CC ′]si cu M,N si P mijloacele laturilor [BC], [CA], respectiv[AB]. Demonstrati ca dreptele XM,Y N si ZP sunt con-curente.

Doru Buzac, IasiSolutie. Cum linia mijlocie [PN ] contine mijloacele tu-

turor cevienelor care pleaca din A, rezulta ca X ∈ [PN ],iar PX si XN sunt linii mijlocii ın triunghiurile ABA′, respectiv ACA′. Astfel,PX

XN=

BA′

2· 2

A′C=

BA′

A′C. Analog se arata ca Y ∈ [PM ], Z ∈ [MN ], iar

MY

Y P=

CB′

B′A

55

Page 60: Revista (format .pdf, 6.5 MB)

siNZ

ZM=

AC ′

C ′B. Atunci

PX

XN· NZ

ZM· MY

Y P=

BA′

A′C· CB′

B′A· AC ′

C ′B= 1 (deoarece AA′,

BB′, CC ′ sunt concurente). Aplicand reciproca teoremei lui Ceva, urmeaza concluziaproblemei.

Nota (Titu Zvonaru). Proprietatea este cunoscuta ın cazul triunghiului oare-care; a se vedea, de exemplu, Gh. Mihalescu - Geometria elementelor remarcabile, Ed.Tehnica, Bucuresti, 1957, pg. 379. Punctul de concurenta este punctul lui Lemoine.Mai mult, are loc proprietatea mai generala

In triunghiul ABC, ın care A′, B′, C ′ sunt mijloacele laturilor BC,CA,AB, iarX,Y, Z mijloacele cevienelor concurente AA1, BB1, CC1, dreptele A′X,B′Y , C ′Zsunt concurente (sursa citata, pg. 494).

VII.122. Se considera dreptunghiul ABCD, cu AB = 1 si AD = 1 +√3, iar

M este un punct interior dreptunghiului, astfel ıncat m(ÖMDC) = m(ÖMCD) = 75◦.Aratati ca triunghiul MAB este echilateral.

Dumitru Mihalache, BarladSolutie. Fie N un punct interior dreptunghiului astfel ıncat triunghiul CDN

A

B C

D

E FM N

sa fie echilateral, iar E si F intersectiile dreptei MN cuAB, respectiv CD. Punctele M si N fiind egal departatede capetele segmentului [CD], rezulta ca MN este medi-atoarea (comuna) a segmentelor [CD] si [AB], deci E siF sunt mijloace pentru [AB], respectiv [CD]. Apoi, cum

m(ÖNMD) = m(ÖNDM) = 15◦, deducem ca △NMD este isoscel, cu NM = ND = 1.

Insa NF =

√3

2, prin urmare EM = (1 +

√3) − 1 −

√3

2=

√3

2. Cu teorema lui

Pitagora, gasim ca AM = BM = 1, deci △ABM este echilateral.

Clasa a VIII-aVIII.116. Raportam planul la un reper cartezian xOy. Determinati multimea

punctelor M(x, y) din plan pentru care 2px2 − y2 = x+ y.Romanta Ghita si Ioan Ghita, Blaj

Solutie. Prin ridicare la patrat, relatia din ipoteza conduce la 4(x2 − y2) =

(x + y)2, adica (x + y)(3x − 5y) = 0, deci y = −x sau y =3x

5. Pe de alta parte,

pentru a asigura existenta radicalului, se impune ca |x| ≥ |y|. In cazul ın care y = −x,

aceasta conditie se verfica pentru orice x ∈ R; daca y =3x

5, conditia se verfica numai

daca x ≥ 0. In concluzie, multimea cautata este reuniunea dintre dreapta y = −x si

semidreapta y =3x

5, x ≥ 0.

VIII.117. Numerele reale pozitive x, y, z sunt astfel ıncat xy = z+1 six

z + 1+y =

2. Demonstrati cax

y+

y

z≥ 3. Cand se atinge egalitatea?

Gheorghe Molea, Curtea de Arges

Solutie. Din prima relatie, obtinem cax

z + 1=

1

y. Inlocuind ın a doua, deducem

56

Page 61: Revista (format .pdf, 6.5 MB)

ca1

y+ y = 2, adica (y − 1)2 = 0. Prin urmare, y = 1 si x = z + 1, cu z ∈ (0,∞).

Astfel,x

y+

y

z= z + 1 +

1

z≥ 2 + 1 = 3, cu egalitate pentru z = 1, x = 2, y = 1.

VIII.118. Fie x, y, z numere reale astfel ıncat x2 + 5yz ≥ 6, y2 + 5zx ≥ 6 siz2 + 5xy ≥ 6. Aflati valoarea minima a lui |x+ y + z|.

Dan Nedeianu, Drobeta-Tr. SeverinSolutie. Adunand inegalitatile din ipoteza, obtinem ca (x+y+ z)2+3(xy+yz+

zx) ≥ 18 si, cum 3(xy + yz + zx) ≤ (x + y + z)2, rezulta ca 2(x + y + z)2 ≥ 18, deunde |x+ y + z| ≥ 3. Deoarece x = y = z = 1 verifica ipotezele problemei si, pentruaceste valori, |x+ y + z| = 3, ınseamna ca valoarea minima a lui |x+ y + z| este 3.

VIII.119. Determinati x ∈�0,

5

4

�si y ∈

�−5x

2,x

2

�pentru careÈ

(7− 2x)(5x+ 2y) +È

(x− 2y)(5− 4x) = 6.

Liviu Smarandache si Lucian Tutescu, CraiovaSolutie. Intervalele ın care se afla numerele x si y ne garanteaza ca 7 − 2x,

5x + 2y, x − 2y si 5 − 4x sunt numere reale nenegative; fie atunci a =√7− 2x,

b =√5x+ 2y, c =

√x− 2y si d =

√5− 4x. Observam ca a2 + b2 + c2 + d2 =

7− 2x+5x+2y+2y+x− 2y+5− 4x = 12. Pe de alta parte, din ipoteza problemei,ab+ cd = 6, deci 2ab+2cd = 12. Deducem ca a2 + b2 + c2 + d2 − 2ab− 2cd = 0, adica(a− b)2 + (c− d)2 = 0, de unde a = b si c = d. Rezolvand sistemul 7− 2x = 5x+ 2y,x− 2y = 5− 4x, gasim x = 1, y = 0.

VIII.120. Rezolvati ın numere naturale ecuatia x+ 2y + 3z = 4xyz − 5.Titu Zvonaru, Comanesti

Solutie. Avem ca z =x+ 2y + 5

4xy − 3. Nu putem avea simultan x ≥ 2 si y ≥ 2

deoarece, ın caz contrar, (x− 2)(y− 2) ≥ 0, deci xy ≥ 2x+2y− 4 si atunci 4xy− 3 ≥8x + 8y − 16 − 3 = x + 2y + 7x + 6y − 19 ≥ x + 2y + 14 + 12 − 19 > x + 2y + 5,adica fractia care-l da pe z ar fi subunitara. Rezulta ca x ∈ {1, 2} sau y ∈ {1, 2}.Daca x = 1, atunci z =

2y + 6

4y − 3. Eventualele solutii y trebuie sa verifice conditia

4y − 3 ≤ 2y + 6, deci y ∈ {1, 2, 3, 4}. Verificand fiecare situatie, gasim solutiile(x, y, z) ∈ {(1, 1, 8); (1, 2, 2)}. Tratand ın aceeasi maniera cazurile x = 2, y = 1 siy = 2, nu obtinem noi solutii.

VIII.121. Demonstrati ca nu putem alege trei puncte necoliniare A,B,C, deaceeasi parte a unui plan α, astfel ıncat dreptele AB,BC si CA sa formeze cu planulα unghiuri de aceeasi masura nenula.

Petru Asaftei, IasiSolutie. Presupunem contrariul si fie A,B,C necoliniare, astfel ıncat (ABC) ∦ α

si m(ÖAB,α) = m(ÖAC,α) = m(ÖBC,α) = θ. Consideram ca d(A,α) > d(B,α) >

d(C,α). Ducand prin C planul β paralel cu α, rezulta ca m(ÖAB, β) = m(ÕAC, β) =

m(ÖBC, β) = θ. Fie {M} = AB ∩ β si P,N proiectiile punctelor A si B pe β; atunci

57

Page 62: Revista (format .pdf, 6.5 MB)

m(ÕACP ) = m(ÕBCN) = m(ÖAMP ) = θ si N ∈ (PM). Din congruenta triunghiurilorAPC si APM deducem (PC) ≡ (PM), iar din congruenta triunghiurilor BNC siBNM rezulta ca (NC) ≡ (NM). Folosind triunghiurile isoscele PCM si NCM ,

obtinem ca ÖNCM ≡ ÖPCM , fals. Ramane atunci adevarata concluzia problemei.

VIII.122. In fiecare patratel al unei table de sah este scris cate un numar natural.Fiecare numar n de pe tabla apare de cate n ori, iar pe primul rand al tablei apar,ordonat strict crescator, toate numerele folosite.

a) Aratati ca, dintre cele opt numere de pe primul rand, cel mult sase sunt pare.b) Determinati cel mai mare numar care poate aparea pe tabla.c) Aratati ca exista o singura modalitate de alegere a numerelor care apar pe tabla,

pentru care produsul numerelor de pe primul rand este impar.Silviu Boga, Iasi

Solutie. a) Suma numerelor de pe primul rand trebuie sa fie 64. Numarul 0 nupoate aparea pe tabla. Daca toate numerele de pe primul rand ar fi pare, suma lorar fi cel putin egala cu 2 + 4 + 6 + 8 + 10 + 12 + 14 + 16 = 72 > 64, fals. Dacaun singur numar de pe primul rand ar fi par, suma de pe primul rand ar fi impara,contradictie. Ramane ca pe primul rand sunt cel mult sase numere pare, situatieposibila; un exemplu ar fi 2, 4, 6, 8, 9, 10, 12, 13.

b) Cele mai mici sapte numere naturale nenule au suma 1+2+ . . .+7 = 28; raman64− 28 = 36 de patratele, deci cel mai mare numar care poate fi ales este 36.

c) Daca produsul numerelor utilizate este impar, toate numerele sunt impare. Cum1+3+5+7+9+11+13+15 = 64 si orice alta alegere de numere impare distincte vaavea suma mai mare, rezulta ca singura posibilitate de alegere a unor numere imparepe tabla este 1, 3, 5, 7, 9, 11, 13, 15.

Clasa a IX-aIX.106. Stabiliti valoarea de adevar a propozitiilor p si q, unde

p : (∃a ∈ Z)(∃n ∈ N∗)(a4 + 1 = 3n); q : (∃a ∈ Z)(∃n ∈ N)(a4 − 1 = 3n).Dan Popescu, Suceava

Solutie. Cum orice patrat perfect este fie M3, fie M3 + 1, a4 + 1 este fie M3 + 1,fie M3 + 2, deci nu poate fi putere a lui 3 cu exponent nenul. Rezulta ca p este opropozitie falsa.

Vom arata ca si q este tot falsa. Pentru aceasta sa presupunem, prin absurd, caexista a ∈ Z si n ∈ N pentru care a4 − 1 = (a − 1)(a + 1)(a2 + 1) = 3n. Atuncia− 1 = 3α, a+ 1 = 3β si a2 + 1 = 3γ , unde 0 ≤ α < β ≤ γ si α+ β + γ = n. Rezultaca 2 = (a + 1) − (a − 1) = 3β − 3α = 3α(3β−α − 1), deci α = 0, β = 1. Deducem caa = 2 si, cum 22 + 1 = 3γ , ajungem la o contradictie.

IX.107. Daca a, b ∈ N∗, atunci

����3a+ 4b

2a+ 3b−

√2

���� < 1

4

����a+ 2b

a+ b−

√2

���� < 1

16

���ab−√2���.

Mihail Bencze, Brasov

Solutie. Consideram functia f : [1,∞) → [1,∞), f(x) =x+ 2

x+ 1. Deoarece

|f(x) − f(y)| < 1

4|x − y|, ∀x, y ∈ [0,∞), x = y, (f ◦ f)(x) = 3x+ 4

2x+ 3si f(

√2) =

√2,

58

Page 63: Revista (format .pdf, 6.5 MB)

rezulta ca |(f ◦ f)(x)− (f ◦ f)(y)| < 1

4|f(x)− f(y)| < 1

16|x− y|, pentru orice x, y ∈

[1,∞), x = y. Considerand x =a

b, a > b si y =

√2, obtinem inegalitatea ceruta.

IX.108. Daca a ∈ (0, 1), demonstrati ca (a+ b)

�1

a+

1

b− 4

(a+ 1)2

�≥ 4

(a+ 1)2,

pentru orice b ∈ (0,∞).Ovidiu Pop, Satu Mare

Solutie. Inegalitatea din enunt este echivalenta cua

b+

b(a− 1)2

a(a+ 1)2≥ 2

1− a

1 + a, care

rezulta din inegalitatea mediilor x+ y ≥ 2√xy, ∀x, y ∈ (0,∞).

IX.109. Demonstrati ca tg x > 4 sinx− 2, oricare ar fi x ∈h0,

π

2

i.

Ionut Ivanescu, Craiova

Solutie. Pentru x = 0, inegalitatea se verifica. Fie x ∈ (0,π

2) si triunghiul ABC

cu m( bA) = 90◦, m(ÒB) = x, iar AD ⊥ BC, D ∈ BC. Cu notatiile uzuale, avem:

2 + tg x > 4 sinx ⇔ 2 +b

c> 4 · ha

c⇔ 2c + b > 4

bc

a⇔ a2(2c + b)2 > 16b2c2 ⇔

(b2 + c2)(2c+ b)2 > 2bc · 8bc.Insa b2 + c2 ≥ 2bc (cu egalitate cand b = c), iar (2c + b)2 ≥ (2

√2cb)2 = 8bc

(cu egalitate cand b = 2c). Rezulta astfel inegalitatea stricta din enunt, ıntrucat nuputem avea simultan b = c si b = 2c.

IX.110. In △ABC, cu notatiile uzuale, aratati ca OI ⊥ OIa ⇔ m( bA) = 60◦.Temistocle Bırsan, Iasi

Solutie. Utilizam relatiile lui Euler OI2 = R2−2Rr si OI2a = R2+2Rra si faptul

ca IIa = AIa −AI =p

cos A2

− p− a

cos A2

=a

cos A2

. Avem succesiv:

OI ⊥ OIa ⇔ OI2 + OI2a = II2a ⇔ (R2 − 2Rr) + (R2 + 2Rra) =a2

cos2 A2

R2 −Rr +Rra =a2

1 + cosA⇔ 1 +

ra − r

R=

4 sin2 A

1 + cosA⇔ 1 +

ra − r

R= 4(1− cosA).

Cumra − r

R=

1

R

�S

p− a− S

p

�=

aS

Rp(p− a)=

abcS

Rbcp(p− a)=

4RS2

Rbcp(p− a)=

4(p− b)(p− c)

bc=

a2 − (b− c)2

bc=

a2 − b2 − c2 + 2bc

bc= −2 cosA + 2, rezulta ca

OI ⊥ OIa ⇔ 1− 2 cosA+ 2 = 4(1− cosA) ⇔ cosA =1

2⇔ m( bA) = 60◦.

Clasa a X-aX.106. Cele m × n patratele ale unui dreptunghi cu m linii si n coloane se

coloreaza cu p culori, unde m < p < n. Spunem ca o colorare are o taietura daca,pe una dintre cele n coloane, toate cele m patratele au aceeasi culoare. Determinatinumarul colorarilor care au k taieturi, unde 0 ≤ k ≤ n. (In legatura cu problema 2,OJM 2006.)

Cecilia Deaconescu, Pitesti

59

Page 64: Revista (format .pdf, 6.5 MB)

Solutie. O coloana care este taietura poate fi colorata ın p moduri, iar o coloanacare nu este taietura poate fi colorata ın pm−pmoduri. Cum cele k taieturi pot fi aleseın Ck

n moduri, rezulta ca numarul colorarilor cu k taieturi este Ckn · pk · (pm − p)n−k,

∀0 ≤ k ≤ n.

X.107. Se considera variabila aleatoare X :

�1 0p q

�, unde p, q ∈ (0, 1), p = q.

Aratati ca variabilele aleatoare |X −M(X)| si (X −M(X))2 sunt dependente.Laurentiu Modan, Bucuresti

Solutie. Cum M(X) = p, atunci X−M(X) :

�q −pp q

�. Rezulta ca |X−M(X)| :�

q pp q

�, (X−M(X))2 :

�q2 p2

p q

�, iar |X−M(X)|3 :

�q3 p3

p q

�. Deducem caM(|X−

M(X)|) = 2pq, M((X−M(X))2) = pq(p+q) = pq si M(|X−M(X)|3) = pq(p2+q2).Daca, prin absurd, variabilele |X−M(X)| si (X−M(X))2 ar fi independente, atunciM(|X −M(X)|3) = M(|X −M(X)|)M((X −M(X))2), adica pq(p2 + q2) = 2p2q2.Cum p = 0 si q = 0, ar rezulta ca p2 + q2 = 2pq, deci p = q, contradictie. Ramane cacele doua varibile aleatoare sunt dependente.

X.108. Daca a, b, c ∈ (1,∞), demonstrati ca are loc inegalitatea

(logb a+ logc a) · (loga b+ logc b)(loga c+ logb c) ≥ 8 log b+c2

a · log c+a2

b · log a+b2

c.

Lucian Tutescu, CraiovaSolutie. Vom arata ca logb a+logc a ≥ 2 log b+c

2a; scriind celelalte doua inegalitati

similare si ınmultindu-le, vom obtine inegalitatea dorita. Cumb+ c

2≥

√bc > 1,

rezulta ca log b+c2

a ≤ log√bc a = 2 logbc a. Notand x = loga b, y = loga c, ar fi

suficient sa demonstram ca1

x+

1

y≥ 4

x+ y, fapt care rezulta din inegalitatea mediilor

MH ≤ MA.

X.109. Se considera multimile nedisjuncte A si B si functiile f : A → (0,∞),g : B → (0, 1). Determinati numarul a ∈ (0, 1) ∪ (1,∞) pentru care af(x)·g(x) +loga g(x) ≥ af(x), ∀x ∈ A ∩B.

Mihai Haivas, IasiSolutie. Consideram functia p : A ∩ B → (0,∞), p(x) = f(x) · g(x). Adunand

ın ambii membri ai inegalitatii din enunt termenul loga f(x), obtinem ca ap(x) +loga p(x) ≥ af(x) + loga f(x), ∀x ∈ A ∩B. Functia h : (0,∞) → R, h(x) = at + loga teste strict descrescatoare daca a ∈ (0, 1) si strict crescatoare daca a ∈ (1,∞). Cumf(x)(g(x) − 1) ≤ 0, ∀x ∈ A ∩ B, rezulta ca p(x) ≤ f(x), ∀x ∈ A ∩ B. Avem si cah(p(x)) ≥ h(f(x)), prin urmare valorile cautate ale lui a sunt elementele intervalului(0, 1).

X.110. Fie D = {z = x+ iy|y > 0} multimea numerelor complexe din semiplanulsuperior, iar D′ = {z = x + iy|x2 + y2 < 1} discul unitate (fara frontiera). Daca

z0 ∈ D este fixat, aratati ca functia f : D → D′, f(z) =z − z0z − z0

este bijectiva.

Adrian Corduneanu, Iasi

60

Page 65: Revista (format .pdf, 6.5 MB)

Solutie. Demonstram mai ıntai ca f este bine definita. Pentru aceasta, observamca f(z) ∈ D′ ⇔ |f(z)| < 1 ⇔ |z − z0| < |z − z0| ⇔ (x − x0)

2 + (y − y0)2 <

(x − x0)2 + (y + y0)

2 ⇔ yy0 > 0, fapt adevarat cand z, z0 ∈ D. Daca presupunemca f(z1) = f(z2) obtinem, dupa calcule, ca (z1 − z2)(z0 − z0) = 0; cum z0 = z0,ıntrucat y0 > 0, rezulta ca z1 = z2, deci f este functie injectiva. Sa demonstram sica f este surjectiva; fie w ∈ D′, adica |w| < 1. Din egalitatea f(z) = w gasim ca

z =z0 · w − z0

w − 1=

(z0 · w − z0)(w − 1)

|w − 1|2=

z0|w|2 − (z0w + z0w) + z0|w − 1|2

, unde Im z =

y0(1− |w|2)|w − 1|2

> 0, prin urmare z ∈ D. Cu aceasta, solutia problemei este completa.

Clasa a XI-a

XI.106. Daca A ∈ M2(R), aratati ca det(A2 +A+ I2) + det(A2 −A+ I2) ≥3

2.

Dan Nedeianu, Drobeta Tr. Severin

Solutia 1. Fie a = trA, b = detA, iar f(X) = det(A − XI2) = X2 − aX + bpolinomul caracteristic al matricei A. Atunci det(A2 + A + I2) = det(A − εI2)(A −ε2I2) = f(ε) · f(ε2) = (ε2 − aε+ b)(ε− aε2 + b) = a2 + b2 + ab+ a− b+1, unde ε esteradacina primitiva de ordin trei a unitatii. Analog, det(A2 −A+ I2) = a2 + b2 − ab−a − b + 1, prin urmare det(A2 + A + I2) + det(A2 − A + I2) = 2a2 + 2b2 − 2b + 2 =

2a2 + 2 + 2(b2 − b) ≥ 0 + 2 + 2

�−1

4

�=

3

2. Egalitatea se atinge cand trA = 0 si

detA =1

2.

Solutia 2. Cu notatiile anterioare, f(A2 + I2) = f(A + iI2) · f(A − iI2) =f(i) ·f(−i) = (i2−ai+b)(i2+ai+b) = (b−1)2+a2. Se stie ca det(X+Y )+det(X−Y ) = 2detX + 2detY, ∀X,Y ∈ M2(R). Pentru X = A2 + I2, Y = A, obtinem cadet(A2+A+ I2)+det(A2−A+ I2) = 2det(A2+ I2)+2detA = 2(b− 1)2+2a2+2b =

2(b2 − b+ 1) + 2a2 ≥ 2 · 34+ 0 =

3

2, cu egalitate cand a = 0, b =

1

2.

XI.107. Se da parabola y = ax2(a > 0). In fiecare punct P (x, y) al parabolei, se

considera vectorul tangent−−→PP ′ =

−→i + 2a

−→xj si vectorul normal

−−→PQ′, orientat spre

exterior, astfel ıncat |−−→PQ′| = |

−−→PP ′|. Apoi, se considera vectorul

−−→PQ = α(x) ·

−−→PQ′,

unde α(x) este o functie data. Determinati locul geometric al punctului Q, atuncicand P descrie parabola, ın fiecare din cazurile:

a) α(x) = a,∀x ∈ R; b) α(x) = −1

a,∀x ∈ R; c) α(x) =

1

2ax,∀x ∈ R∗.

Adrian Corduneanu, Iasi

Solutie. Intrucat−−→OP = x

−→i +ax2−→j ,

−−→PP ′ =

−→i +2ax

−→j ,

−−→PQ′ = 2ax

−→i −−→

j ,−−→PQ =

α(x)(2ax−→i −−→

j ), rezulta ca−−→OQ =

−−→OP +

−−→PQ = (x+ 2ax · α(x))−→i + (ax2 − α(x))

−→j ,

prin urmare xQ = x+ 2ax · α(x), iar yQ = ax2 − α(x).

a), b) Daca α(x) = c, c ∈ R\§− 1

2a

ª, atunci xQ = (1+2ac)x, cu 1+2ac = 0, deci

x =xQ

1 + 2ac, iar yQ =

a

(1 + 2ac)2x2Q − c. In cazul ın care c = a, locul geometric al

61

Page 66: Revista (format .pdf, 6.5 MB)

lui Q este parabola y =a

(1 + 2a2)2x2 − a, iar cand c = −1

a, locul lui Q este parabola

y = ax2 − 1

a.

c) Daca α(x) =1

2ax, ∀x ∈ R∗, atunci xQ = x+2ax · 1

2ax= x+1, deci x = xQ−1,

iar yQ = ax2 − 1

2ax= a(xQ − 1)2 − 1

2a(xQ − 1). Ecuatia locului geometric, ın acest

caz, este y = a(x− 1)2− 1

2a(x− 1), x = 1; invitam cititorul sa traseze efectiv aceasta

curba.

XI.108. Calculati limn→∞

( n√n)1+

1ln 2+

1ln 3+...+ 1

lnn .

Cezar Lupu, student, Bucuresti

Solutie. Fie xn = 1 +1

ln 2+

1

ln 3+ . . . +

1

lnn, n ∈ N, n ≥ 2; atunci ( n

√n)xn =

elnnn xn . Pentru a calcula limita sirului de la exponent, folosim lema Stolz-Cesaro:

limn→∞

lnn

nxn = lim

n→∞

xnn

lnn

= limn→∞

xn+1 − xnn+1

ln(n+1) −n

lnn

=

= limn→∞

lnn

(n+ 1) lnn− n ln(n+ 1)= lim

n→∞

lnn

lnn− ln(1 + 1n )

n= 1.

Prin urmare, limita ceruta este egala cu e.

XI.109. Determinati a, b, c ∈ (0,∞) pentru care limita limn→∞

�a

n√b+ c

�n

exista

si este finita.Constantin Chirila, Iasi

Solutie. Daca a < c + 1, atunci exista ε > 0 pentru care a < c + 1 − ε. Cumlim

n→∞n√b = 1, pentru acest ε, exista n0(ε) astfel ıncat

n√b > 1− ε, ∀n ≥ n0(ε). Cand

n ≥ n0(ε), avem caa

n√b+ c

<a

c+ 1− ε< 1 si atunci lim

n→∞

�a

n√b+ c

�n

= 0. Cu un

rationament analog, obtinem ca limn→∞

�a

n√b+ c

�n

= +∞, daca a > c+ 1. Vom arata

ca, pentru a = c + 1, limita din enunt este b−1a . Fie yn =

n√b+ c

a=

n√b+ a− 1

a=

1 +n( n

√b− 1)

na= 1 +

xn

n, unde xn =

n( n√b− 1)

aeste un sir cu limita

ln b

a. Rezulta

ca limn→∞

�a

n√b+ c

�n

= limn→∞

1

(1 + xn

n )n=

1

eln ba

= b−1a . In concluzie, limita exista si

este finita daca si numai daca a ≤ c+ 1.

XI.110. Date functiile continue f, g : R → R, sunt echivalente afirmatiile:i) f = g;ii) daca h : R → R este continua, ecuatia f(x) = h(x) are solutii reale atunci si

numai atunci cand ecuatia g(x) = h(x) are solutii reale.Marian Tetiva, Barlad

62

Page 67: Revista (format .pdf, 6.5 MB)

Solutie. Implicatia i)⇒ii) este evidenta. Pentru a demonstra reciproca, estesuficient sa aratam ca: daca f, g sunt functii continue distincte, exista h continuaastfel ıncat una dintre ecuatiile f(x) = h(x) si g(x) = h(x) are solutii reale, iarcealalta nu. Sa justificam aceasta afirmatie.

In ipoteza ca f = g, putem presupune ca f(p) > g(p), pentru un numar real p; fie[a, b] un interval care contine p ın interior, iar h : R → R,

h(x) =

8>>>><>>>>:max{f(x), g(x)}+ 1, x < a

max{f(x), g(x)}+ p− x

p− a, a ≤ x < p

max{f(x), g(x)}+ x− p

b− p, p ≤ x < b

max{f(x), g(x)}+ 1, b ≤ x

.

Se verifica faptul ca h este continua. Cum h(x) > max{f(x), g(x)}, ∀x = p, ınseamnaca niciuna dintre ecuatiile f(x) = h(x) si g(x) = h(x) nu poate avea solutii diferitede p. Pe de alta parte, h(p) = f(p) > g(p), deci prima ecuatie are solutia p, iar ceade-a doua nu are solutii, ceea ce trebuia demonstrat.

Nota autorului. Afirmatia problemei ramane adevarata pentru functii polino-miale pe portiuni.

Clasa a XII-a

XII.106. Pentru a > 0 dat, calculati

Zax+ 2

x(a+ x2eax)dx, unde x ∈ (0,+∞)

I.V. Maftei, Bucuresti si Mihai Haivas, Iasi

Solutie. Daca φ(x) = x2eax, atunci φ′(x) = xeax(ax+ 2), deciZf(x)dx =

Zφ′(x)

x2eax(a+ φ(x))dx =

Zφ′(x)

φ(x)(a+ φ(x))dx =

=1

a

�Zφ′(x)

φ(x)dx−

Zφ′(x)

a+ φ(x)dx

�=

1

aln

φ(x)

a+ φ(x)+ C =

=1

aln

x2eax

a+ x2eax+ C.

XII.107. Fie Un =

Z π2

0

sin(2n+ 1)x

sinxdx si Vn =

Z π2

0

cos(2n+ 1)x

cosxdx, unde n ∈

N∗. Aratati ca Un = (−1)nVn =π

2, ∀n ∈ N∗.

Gheorghe Costovici, Iasi

Solutie. Demonstram prin inductie matematica faptul ca Un =π

2, ∀n ∈ N∗.

Pentru n = 1, avem:

U1 =

Z π2

0

sin 3x

sinxdx =

Z π2

0(3− 4 sin2 x)dx =

Z π2

0(1 + 2 cos 2x)dx =

π

2.

63

Page 68: Revista (format .pdf, 6.5 MB)

Apoi, presupunem ca Un =π

2si demonstram ca Un+1 =

π

2:

Un+1 =

Z π2

0

sin((2n+ 1)x+ 2x)

sinxdx =

=

Z π2

0

sin(2n+ 1)x · (1− 2 sin2 x) + cos(2n+ 1)x · 2 sinx cosxsinx

dx =

= Un + 2

Z π2

0(cos(2n+ 1)x · cosx− sin(2n+ 1)x sinx)dx =

= Un + 2

Z π2

0cos(2n+ 2)xdx = Un =

π

2.

Daca ın Un facem schimbarea de variabila x =π

2− t, obtinem ca Un = (−1)nVn si

astfel solutia problemei este completa.

XII.108. Demonstrati ca sirul (an)n≥1 definit prin an =

Z n

1

lnx

xp + 1dx, ∀n ≥ 1,

unde p ∈ (1,∞) este fixat, este convergent.Rodica Luca Tudorache, Iasi

Solutie. Deoarece an+1 − an =R n+1n

lnx

xp + 1dx > 0, ∀n ∈ N∗, rezulta ca sirul

este monoton crescator. Considerand functia g : [1,+∞) → R, g(x) = lnx− xp−12 si

studiindu-i variatia, observam ca exista x0 ≥ 1 astfel ıncat g(x) ≤ 0, ∀x ∈ [x0,+∞)

(unde x0 = 1 cand p ≥ 1 +2

esi x0 > 1 daca p ∈ (1, 1 +

2

e)). Atunci:

an =

Z x0

1

lnx

xp + 1dx+

Z n

x0

lnx

xp + 1dx ≤ A+

Z n

x0

xp−12

xp + 1dx <

< A+

Z n

x0

xp−12

xpdx = A+

2

1− px

1−p2

����nx0

< A+2

p− 1x

1−p2

0 = B,

unde A =

Z x0

1

lnx

xp + 1dx, pentru n ≥ [x0]+1. Rezulta astfel ca sirul dat este marginit

superior, iar concluzia urmeaza din teorema lui Weierstrass.Nota (Gheorghe Costovici). Se arata, cu metode ce depasesc putin nivelul

matematicii de liceu, ca daca c > 0, p > 1, atunci exista limy→∞

Z y

1

lnx

xp + c∈ [0,+∞).

De aici, problema rezulta ca un caz particular.

XII.109. Fie (G, ·) un grup comuativ, cu proprietatea ca exista n ∈ N∗ astfelıncat din xn = yn, x, y ∈ G, rezulta ca x = y. Daca f, g sunt endomorfisme ale lui G,aratati ca ecuatia f(x) = g(x−1) are solutie unica ın G, daca si numai daca functiah : G → G, h(x) = f(xn) · g(xn) este injectiva.

D.M. Batinetu-Giurgiu, BucurestiNota. Problema a fost publicata, cu numarul XII.74, ın RecMat 2/2006, iar

solutia sa poate fi gasita ın RecMat 2/2007, pg. 136. De aceasta situatie este vinovataredactia, care ısi cere scuze.

64

Page 69: Revista (format .pdf, 6.5 MB)

XII.110. Fie P,Q ∈ C[X] polinoame neconstante, astfel ıncat P si Q au aceleasiradacini, iar P − 1 si Q− 1 au si ele aceleasi radacini. Aratati ca P = Q.

Adrian Reisner, ParisSolutie. Fie n = grP, m = grQ, α1, . . . , αp radacinile lui P , de multiplicitati

k1, . . . , kp, iar β1, . . . , βr radacinile lui P − 1, de multiplicitatile l1, . . . , lr. Notam cuD si ∆ cei mai mari divizori comuni ai perechilor de polinoame (P, P ′), respectiv(P − 1, P ′). Cum D si ∆ divid P si P − 1, iar (P, P − 1) = 1, atunci (D,∆) = 1.Insa D|P ′,∆|P ′, prin urmare D · ∆|P ′, deci gr(D · ∆) ≤ n − 1. Pe de alta parte,

gr D =pP

i=1

(ki − 1) = n − p, gr∆ =rP

i=1

(li − 1) = n − r, asadar 2n − p − r ≤ n − 1,

adica p+ r ≥ n+1. Procedand asemanator ın cazul lui Q, obtinem ca p+ r ≥ m+1.Observam ca P − Q = (P − 1) − (Q − 1) este un polinom care se anuleaza ın

α1, . . . , αp, β1, . . . , βr, iar aceste numere sunt distincte, ıntrucat (P, P−1) = 1. Rezulta

ca P −Q are cel putin p+ r radacini, cu p+ r ≥ max{m,n}+ 1. Insa gr(P −Q) ≤max{m,n}, prin urmare P −Q = 0, deci P = Q.

Solutiile problemelor pentru pregatireaconcursurilor propuse ın nr. 1/2010

A. Nivel gimnazial

G156. Fie a1, a2, . . . , an ∈ R∗+, cu

1

a1+

1

a2+ . . .+

1

an= 1. Aratati ca

1

a31 + a22+

1

a32 + a23+ . . .+

1

a3n + a21<

1

2.

Angela Tigaeru, SuceavaSolutie. Din datele problemei rezulta ca ai ∈ (1,∞), ∀i = 1, n, prin urmare1

a31 + a22<

1

1 + a22≤ 1

2a2. Procedand analog cu ceilalti termeni si sumand inegalitatile

obtinute, rezulta cerinta problemei.

G177. Fie k > 0 si a, b, c ∈ [0,+∞) astfel ıncat a + b + c = 1. Demonstrati caa

a2 + a+ k+

b

b2 + b+ k+

c

c2 + c+ k≤ 9

9k + 4.

Titu Zvonaru, ComanestiSolutie (Gheorghe Iurea). Sensul inegalitatii din enunt, sa o numim (I), de-

pinde de parametrul k > 0; pentru k = 0, 01, a = 0, 1, b = 0, 2, c = 0, 7 inegali-

tatea este falsa. Cu notatia f(x) =x

x2 + x+ k, x > 0, si avand ın vedere conditia

a+ b+ c = 1, (I) se scrie f(a) + f(b) + f(c) ≤ 3f

�a+ b+ c

3

�si va fi adevarata pen-

tru valorile lui k pentru care functia f este concava (inegalitatea lui Jensen). Cum

f ′′(x) =2(x3 − 3kx− k)

(x2 + x+ k)3, x > 0, problema revine la a gasi valorile lui k pentru care

g(x) = x3−3kx−k ≤ 0, x ∈ (0, 1). Ecuatia de gradul trei g(x) = 0 are discriminantul

65

Page 70: Revista (format .pdf, 6.5 MB)

∆ = k2�1

4− k

�(a vedea, de exemplu, Viorel Gh. Voda - Miraculoasele ecuatii,

1987; pp. 39-43). Daca k =1

4(i.e. ∆ = 0), avem g(x) =

�x+

1

2

�2

(x− 1) si rezulta

ca f ′′(x) < 0, x ∈ (0, 1), adica f este concava pe intervalul (0, 1); ın acest caz, are

loc (I). Daca k >1

4(i.e. ∆ > 0), ecuatia g(x) = 0 are o singura radacina reala, care

este pozitiva; mai mult, ea este > 1, deoarece g(1) = 1 − 4k < 0 si g(+∞) > 0. In

acest caz, f va fi concava si are loc (I). Daca k <1

4(i.e. ∆ < 0), ecuatia g(x) = 0

are trei radacini reale, dar numai una pozitiva, s-o numim x1; x1 ∈ (0, 1), deoarece

g(0) = −k < 0 si g(1) = 1 − 4k > 0. Cum g

�1

3

�=

1

27− 2k, deducem ca: pentru

k <1

54avem g

�1

3

�> 0 si, deci, x1 <

1

3; pentru k >

1

54avem g

�1

3

�< 0 si x1 >

1

3.

Ca urmare, daca k >1

54inegalitatea (I) are loc pe intervalul (0, x1), iar daca k <

1

54are loc pe (x1, 1) inegalitatea inversata.

G178. Daca n ∈ N\{0, 1}, aratati ca 1− n

n≤ {nx} − {x} <

n− 1

n, ∀x ∈ R.

Gheorghe Iurea, Iasi

Solutie. Cum inegalitatea se pastreaza daca ınlocuim pe x cu x + 1, este su-ficient sa o demonstram pentru x ∈ [0, 1). In acest caz, exista k ∈ {1, 2, . . . , n}

pentru care x ∈�k − 1

n,k

n

�, deci {nx} = nx − (k − 1), iar {x} = x. Inegalitatea

este echivalenta cunk − 2n+ 1

n(n− 1)≤ x <

nk − 1

n(n− 1), ceea ce este adevarat, ıntrucat�

k − 1

n,k

n

�⊂�nk − 2n+ 1

n(n− 1),nk − 1

n(n− 1)

�. In partea stanga, egalitatea se realizeaza

pentru x ∈§p− 1

n

����p ∈ Zª.

G179. Determinati numerele prime a, b, c, d si numarul p ∈ N∗, astfel ıncata2

p

+ b2p

+ c2p

= d2p

+ 3.

Cosmin Manea si Dragos Petrica, Pitesti

Solutie. Nu putem avea d = 2 deoarece, ın caz contrar, a, b si c ar fi cel putinegale cu d si membrul stang ar fi strict mai mare decat cel drept. Rezulta ca d esteun numar prim impar, deci suma din stanga va fi para. Atunci un termen va fi sigurpar (si atunci va fi egal cu 2; sa spunem ca a = 2), iar ceilalti doi vor avea aceeasiparitate.

Daca b, c sunt pare, atunci b = c = 2, iar ecuatia devine 3 ·22p = d2p

+3. Deducemca d se divide cu 3 si fiind, prim, obtinem ca d = 3, iar 22

p

= 32p−1 + 1. Aceasta

egalitate se realizeaza pentru p = 1, iar daca p ≥ 2, membrul drept este strict maimare decat cel stang.

Daca b, c sunt impare, atunci b2p

, c2p

si d2p

vor fi de forma M4 + 1, prin urmaremembrul stang al egalitatii din enunt va fi de forma M4+3, iar cel drept se divide cu4, contradictie. Ramane ca singura solutie este (a, b, c, d, p) = (2, 2, 2, 3, 1).

66

Page 71: Revista (format .pdf, 6.5 MB)

G180. Aflati restul ımpartirii numarului S = 20102009!+20092008!+ . . .+21!+10!

prin 41.Razvan Ceuca, elev, Iasi

Solutie. Daca a este un numar care nu se divide cu 41 (care este prim), dinmica teorema a lui Fermat obtinem ca a40 ≡ 1(mod 41). Daca a ≥ 6, atunci (a −1)! se divide cu 40, prin urmare a(a−1)! ≡ 1(mod 41) pentru orice a ≥ 6 care nuse divide cu 41. Daca b se divide cu 41, evident ca b(b−1)! ≡ 0(mod 41). Atuncisuma A = 20102009! + 20092008! + . . . + 65! este, modulo 41, congruenta cu 1 + 0 +(1 + 1 + . . .+ 1| {z }

40 de 1

+0) · 48 + (1 + 1 + . . .+ 1| {z }35 de 1

) = 1956, deci A ≡ 29(mod 41). Ramane

sa ımpartim la 41 primii cinci termeni ai sumei, afland fiecare rest. Avem ca 10! ≡1(mod 41), 21! ≡ 2(mod 41), 32! ≡ 9(mod 41), 43! = 4096 ≡ 37(mod 41), iar 54! =1258 = (M41 + 2)8 = M41 + 28 = M41 + 256 = M41 + 10, deci 541 ≡ 10(mod 41). Inconcluzie, S ≡ 1 + 2 + 9 + 37 + 10 + 29 ≡ 6(mod 41).

G181. Fie k ≥ 1 un numar natural dat. Aratati ca exista o infinitate de numerenaturale n astfel ıncat nk divide n!.

Marian Tetiva, BarladSolutie. Alegem n = pq, unde p si q sunt numere prime distincte, mai mari

decat k. Cum pq > kp si pq > kq, printre numerele 1, 2, . . . , pq apar toate numerelep, 2p, . . . , kp, q, 2q, . . . , kq, iar acestea sunt distincte, deoarece (p, q) = 1. Rezulta ca(pq)! contine factorul p de cel putin k ori si, de asemenea, contine factorul q de celputin k ori, prin urmare (pq)! se divide cu (pq)k, ceea ce trebuia demonstrat.

A

CB

QM

S

T

N

P

G182. Se considera triunghiul ABC si punctele M ∈ [AB],N ∈ [BC], P ∈ [CA] astfel ıncat MP∥BC, iar MN∥AC. Fie{Q} = AN ∩MP si {T} = BP ∩MN . Demonstrati ca AAMN =APTN +AQPC .

Andrei Razvan Baleanu, elev, MotruSolutie (Gabriel Popa). Vom arata, mai ıntai, ca QT∥AB.

Intr-adevar, folosind asemanari evidente si faptul ca CPMN este

paralelogram, obtinem caMQ

QP=

BN

NC=

BN

MP=

BT

TPsi, conform reciprocei teoremei

lui Thales, urmeaza ca QT∥MB.Din AC∥MN, rezulta ca AAMN = APMN = APTN +APTM . Cum BC∥MP, de-

ducem ca AQPC = AQPB . Ramane sa mai aratam ca APTM = AQPB , ceea ce revineA

B C

D

E

F

la AMQS = ABTS , unde {S} = BQ ∩ MT. Insa QT∥MB, prin urmareAQBM = ATBM si, scazand din ambii membri ASBM , obtinem ceea cedorim.

G183. Se considera triunghiul isoscel ABC, cu AB = AC sim( bA) < 30◦. Stiind ca exista D ∈ [AB] si E ∈ [AC] astfel ıncat

AD = DE = EC = BC, determinati masura unghiului bA.Vasile Chiriac, Bacau

Solutie. Consideram punctul F astfel ıncat BDEF sa fie paralel-ogram; atunci BF = DE = AD = EC = BC, iar EF = BD =

AB − AD = AC − EC = AE. Din AB∥EF, obtinem ca ÕDAE ≡ ÕCEF

67

Page 72: Revista (format .pdf, 6.5 MB)

(corespondente), prin urmare △ADE ≡ △ECF (L.U.L.). Rezulta ca CE = CF, deci

△BCF este echilateral. Insa m(ÕBFE) = m(ÕBDE) = 2m( bA), m(ÕEFC) = m( bA),

asadar 60◦ = m(ÕBFC) = m(ÕBFE) +m(ÕEFC) = 3m( bA), de unde m( bA) = 20◦.

G184. In planul xOy, consideram punctele Aij de coordonate (i, j), unde i, j ∈{0, 1, 2, 3, 4}. Fie P multimea patratelor care au toate varfurile printre punctele Aij

considerate. Aflati lungimea minima a unui drum care parcurge numai laturi alepatratelor din P si care uneste punctele A00 si A44.

Claudiu Stefan Popa, IasiSolutie. Cum A11 si A00 nu apartin unei aceleiasi laturi a unui patrat din P, un

drum care pleaca din A00 trece fie prin A10, fie prin A01. Din acelasi motiv, un drumcare ajunge ın A44 trece fie prin A43, fie prin A34. Drumul minim ıntre A10 si A43

este cel ın linie dreapta; el ındeplineste cerintele enuntului, ıntrucat A10A21A13A01,A21A32A24A13 si A32A43A34A23 sunt patrate din P, iar A10A21, A21A32 si A32A43

sunt laturi ale lor. Analog pentru drumul ıntre A01 si A34. Drumul ın linie dreaptaıntre A10 si A34 sau cel ıntre A01 si A43 nu contin doar laturi ale patratelor din P sisunt, oricum mai lungi decat celelalte, ıntrucat 2

√5 > 3

√2. Rezulta ca exista doua

trasee minimale, ambele de lungime 2 + 3√2.

G185. Aratati ca exista o colorare a planului cu n culori, unde n ≥ 2 este unnumar natural dat, astfel ıncat orice segment din plan sa contina puncte colorate cufiecare dintre cele n culori.

Paul Georgescu si Gabriel Popa, IasiSolutie. Consideram x1, x2, . . . , xn−1 numere irationale, astfel ıncat raportul

oricaror doua dintre aceste numere sa fie tot numar irational. Alegerea unor ast-fel de numere este posibila, cunoscut fiind faptul ca multimea numerelor irationaleeste nenumarabila. Fixam un punct ın plan O si coloram toate cercurile de centru Osi raza qx1, unde q ∈ Q∗, cu prima culoare; apoi, toate cercurile de centru O si razaqx2, q ∈ Q∗, cu a doua culoare s.a.m.d., folosind astfel primele n − 1 culori. Toatepunctele planului ramase le coloram cu a n-a culoare si, ın acest fel, obtinem o colorarecu proprietatea dorita. Intr-adevar, intervalul [a, b] - unde a = min{OP |P ∈ [AB]},b = max{OP |P ∈ [AB]}, [AB] fiind un segment oarecare, dar fixat - contine punctede forma qixi, qi ∈ Q∗, pentru fiecare i = 1, n− 1, si aceste puncte nu acopera ıntregulinterval, deci segmentul [AB] va contine puncte colorate cu fiecare dintre cele n culori.

B. Nivel licealNota. Cand numarul 2/2010 al revistei era sub tipar, am primit solutii corecte

ale problemelor L171 si L174 din partea domnului Cosmin Manea, Pitesti.A

B C

EB

A

C

F

D

1

21

2

1

2

G

L176. Fie D,E, F proiectiile centrului de greutate G altriunghiului ABC pe dreptele BC,CA, respectiv AB. Aratatica cevienele AD,BE si CF sunt concurente daca si numaidaca triunghul este isoscel.

Temistocle Bırsan, IasiSolutie. Daca △ABC este isoscel, se constata usor ca

AD,BE si CF sunt concurente. Reciproc, sa presupunem cacele trei ceviene sunt concurente; atunci, conform teoremei lui Ceva, forma trigono-

68

Page 73: Revista (format .pdf, 6.5 MB)

metrica, are loc egalitatea cosA1 · cosB1 · cosC1 = cosA2 · cosB2 · cosC2 (notatiilesunt cele din figura). Folosind teorema cosinusului si teorema medianei ın triunghiulB′BC obtinem ca

cosB2 =a2 +m2

b − b2

4

2amb=

1

2ama

�a2 +

2a2 + 2c2 − b2

4− b2

4

�=

3a2 − b2 + c2

4ama.

Scriind ınca cinci relatii analoage si ınlocuind ın teorema lui Ceva, deducem ca (3a2−b2+c2)(3b2−c2+a2)(3c2−a2+b2) = (3a2−c2+b2)(3b2−a2+c2)(3c2−b2+a2). Dupaınmultiri si reduceri de termeni, rezulta ca a2b4−a4b2+ b2c4− b4c2+ c2a4− c4a2 = 0,adica (a2 − b2)(b2 − c2)(c2 − a2) = 0, de unde a = b sau b = c sau c = a, deci △ABCeste isoscel.

In cazul ın care △ABC ar fi obtuzunghic, poate aparea cazul ın care una dintreproiectiile lui G se afla ın afara triunghiului; se utlizeaza segmente orientate si seprocedeaza analog.

L177. Consideram triunghiul ABC, G centrul sau de greutate, iar K punctulde intersectie al simedianelor. Notam cu M si N proiectiile lui G pe bisectoareainterioara si pe cea exterioara ale unghiului bA, iar P si Q sunt proectiile lui K pebisectoarele interioara, respectiv exterioara, ale unghiului bA. Demonstrati ca drepteleGK,MN si PQ sunt concurente.

Titu Zvonaru, ComanestiSolutie. Fie {S} = MN ∩ AG, iar {T} = MN ∩ KG. Cum AMGN este

N

K

G

T

S

M

A

B C

.

.

.

dreptunghi, rezulta ca ÖNMA ≡ ÖMAG. Deoarece AG si

AK sunt izogonale, avem ca ÖMAG ≡ ÖMAK. Deducem caÖNMA ≡ ÖMAK, deci dreptele MN si AK vor fi paralele.Punctul S fiind mijlocul lui AG, din reciproca teoremei lin-iei mijlocii urmeaza ca T este mijlocul lui KG. Analog sedemonstreaza ca PQ trece prin mijlocul lui KG si astfeldreptele MN,PQ si KG sunt concurente ın T .

L178. Fie ABCD un romb de latura 1 si punctele A1 ∈(AB), B1 ∈ (BC), C1 ∈ (CD), D1 ∈ (DA). Demonstrati ca A1B

21 +B1C

21 +C1D

21 +

D1A21 ≥ 2 sin2 A.

Neculai Roman, Mircesti, IasiSolutie. Fie x = AA1, y = BB1, z = CC1 si t = DD1. Cu teorema cosinusului,

obtinem ca E = A1B21+B1C

21+C1D

21+D1A

21 = [(1−x)2+y2+2(1−x)y cosA]+[(1−

y)2+z2−2(1−y)z cosA]+[(1−z)2+t2+2(1−z)t cosA]+[(1−t)2+x2−2(1−t)x cosA] =2(x2 + y2 + z2 + t2) − 2(xy − yz + zt − tx) cosA − 2(x + y + z + t) − 2(x − y + z −t) cosA + 4. Privind pe E ca trinom de gradul doi ın x, minimul se atinge cand

x =1 + cosA+ (y − t) cosA

2, deci cand 2x−y cosA+ t cosA = 1+cosA. Privind pe

E ca trinom de gradul doi ın y, z, t, minimul se va atinge cand 2y−x cosA+z cosA =1−cosA, 2z−t cosA+y cosA = 1+cosA, respectiv 2t−z cosA+x cosA = 1−cosA.

Cele patru egalitati se realizeaza simultan daca si numai daca x = z =1 + cosA

2,

y = t =1− cosA

2si, ın acest caz, Emin = 2 sin2 A, de unde cerinta problemei.

69

Page 74: Revista (format .pdf, 6.5 MB)

L179. Demonstrati ca ın orice triunghi are loc inegalitatea

2(9R2 − p2)

9Rr≥ cos2 A

sinB sinC+

cos2 B

sinC sinA+

cos2 C

sinA sinB≥ 1.

I.V. Maftei si Dorel Baitan, Bucuresti

Solutie. Pentru inegalitatea din stanga, avem ca Σcos2 A

sinB sinC=

ΣsinA cos2 A

ΠsinA=

ΣsinA− Σsin3 A

ΠsinA≤

ΣsinA− 19 (Σ sinA)3

ΠsinA=

4 ·Πcos A2 − 64

9 Πcos3 A2

8Π sin A2 cos A

2

=pR − p3

9R3

2pr4R2

=

2(9R2 − p2)

9Rr. Sa demonstram acum inegalitatea din dreapta:

Σcos2 A

sinB sinC≥ (Σ cosA)2

ΣsinB sinC=

(R+rR )2

14R2 · Σbc

=4(R+ r)2

p2 + r2 + 4Rr≥ 1,

deoarece ultima inegalitate revine la 4R2 +4Rr+3r2 ≥ p2, adica la cunoscuta 8R2 +4r2 ≥ a2+ b2+ c2. (Am folosit pe parcurs faptul ca Σbc = p2+ r2+4Rr.) Egalitatilese ating ın cazul triunghiului echilateral.

L180. Determinati numarul minim de factori din produsul P = sinπ

4n· sin 2π

4n·

sin3π

4n· . . . · sin (22n−1 − 1)π

4n, n ∈ N∗, astfel ıncat P < 10−9.

Ionel Tudor, Calugareni, Giurgiu

Solutie. Este cunoscuta modalitatea de calcul a produsului Q = sinπ

msin

sin3π

m. . . sin

(m− 1)π

m: consideram xk = cos

2kπ

2m+ i sin

2kπ

2m= cos

m+ i sin

m,

k = 1,m− 1 si xk = coskπ

m− i sin

m, k = 1,m− 1, radacinile polinomului X2m−2+

. . .+X2+1 =X2m − 1

X2 − 1, prin urmare X2m−2+ . . .+X2+1 =

m−1Qk=1

(X−xk)(X−xk) =

m−1Qk=1

(X2 − 2X coskπ

m+ 1). Dand lui X valoarea 1 si efecutand calculele, obtinem

cam−1Qk=1

sinkπ

2m=

√m

2m−1. Cum sin

2m= cos

�π

2− kπ

2m

�= cos

(m− k)π

2m, k =

1,m− 1, deducem cam−1Qk=1

coskπ

2m=

√m

2m−1. Atunci Q =

m−1Qk=1

�2 sin

2m· cos kπ

2m

�=

2m−1

� √m

2m−1

�2

=m

2m−1, m ≥ 2.

Observam ca sin(22n−1 + k)π

4n= sin

�π

2+

4n

�= cos

4n= sin

�π

2− kπ

4n

�=

sin(22n−1 − k)π

4n, ∀k = 0, 22n−1 − 1. Atunci, pentru m = 4n, produsul Q poate fi

70

Page 75: Revista (format .pdf, 6.5 MB)

scris sub forma

Q =

�22n−1−1Y

k=1

sinkπ

4n

�·

4n−1Yk=2n

sinkπ

4n

!=

�22n−1−1Y

k=1

sinkπ

4n

�2

= P 2,

prin urmare P 2 =4n

24n−1=

1

24n−2n−1. Dorim ca P 2 < 10−18, deci se impune conditia

24n−2n−1 > 1018. Obtinem ca 4n − 2n− 1 >

18

lg 2, iar valoarea minima a lui n pentru

care are loc aceasta inegalitate este n = 4. Rezulta ca numarul minim cautat defactori din P este 27 − 1 = 127.

L181. Fie P un punct de pe frontiera circulara a unui semidisc, iar d tangentaın P la aceasta frontiera. Notam cu C corpul de rotatie care se obtine prin rotireasemidiscului ın jurul dreptei d. Studiati variatia volumului lui C, functie de pozitiapunctului P .

Paul Georgescu si Gabriel Popa, IasiSolutie (Gheorghe Iurea). Consideram semidiscul de diametru MN = 2a si fie

P

T

QQ

y

M N

N

x

Q si T mijloacele segmentuluiMN, respectiv arcului øMN . Notamcu V(P ) volumul corpului obtinut prin rotirea semidiscului ın ju-

rul tangentei ın P la el. Evident ca, daca P ∈ øMT iar P ′ estesimetricul lui P fata de QT, atunci V(P ′) = V(P ); vom calcula

deci V(P ) doar pentru P ∈ øMT .Raportam spatiul la un reper Pxyz si consideram Q(0, a, 0),

unde a > 0 este raza discului din care face parte semidiscul. Con-sideram semidiscul inclus ın planul xPy si fie xN = t ∈ [0, a].

Daca P = M (deci t = 0), ecuatia arcului øMT este y = a −√a2 − x2, ecuatia

arcului ÷TN este y = a +√a2 − x2, deci V(P ) = π

Z a

0[(a +

pa2 − x2)2 − (a −p

a2 − x2)2]dx = 4aπ

Z a

0

pa2 − x2dx = a3π2.

Daca P ∈ øMT\{M,T} (deci t ∈ (0, a)), atunci, cu notatiile din figura alaturata,

N(t, a +√a2 − t2), N ′(t, a −

√a2 − t2), M(−t, a −

√a2 − t2), ecuatiile arcelor ùMQ′

si øQ′N sunt y = a−√a2 − x2, respectiv y = a+

√a2 − x2, iar ecuatia drepteiMN este

y = a+

√a2 − t2

tx. Astfel, V(P ) = π

R t−t

"�a+ x

√a2 − t2

t

�2

− (a−√a2 − x3)2

#dx+

π

Z a

t[(a+

pa2 − x3)2 − (a−

pa2 − x2)2]−tdx = a2π

�aπ − 4t

3

�.

Daca P = T (deci t = a), atunci V(P ) = πR a−a[a

2 − (a −√a2 − x2)2]dx =

a3π

�π − 4

3

�. In concluzie, V(P ) = a2π

�aπ − 4t

3

�, ∀t ∈ [0, a], iar aceasta functie

(care este liniara ın t, cu panta negativa) ısi atinge maximul ın primul caz.

L182. Fie (xn)n≥1 un sir de numere ıntregi cu proprietatea ca xn+2−5xn+1+xn =0, ∀n ∈ N∗. Aratati ca, daca un termen al sirului se divide cu 22, atunci o infinitate

71

Page 76: Revista (format .pdf, 6.5 MB)

de termeni au aceasta proprietate.

Marian Tetiva, Barlad

Solutie. Modulo 22, au loc egalitatile: xn+2 ≡ 5xn+1 − xn, xn+3 ≡ 2xn+1 − 5xn,xn+4 ≡ 5xn+1 − 2xn, xn+5 ≡ xn+1 − 5xn, xn+6 ≡ −xn. Observam astfel ca, dacaxn0 se divide cu 22, atunci se vor divide cu 22 toti termenii sirului de forma xn0+6k,k ∈ N, de unde decurge concluzia problemei.

Nota. Dl. Titu Zvonaru observa ca are loc un rezultat ceva mai general decatcel din enuntul problemei, anume:

Fie a ≥ 1 un numar natural, b = a3 − 3a, iar (xn)n≥1 un sir de numere ıntregidefinit prin xn+2 − axn+1 + xn = 0, ∀n ∈ N si x1, x2 ∈ Z. Daca un termen al siruluise divide cu b, o infinitate de termeni ai sirului au aceasta proprietate.

Aceasta generalizare a fost gasita si de catre dl. Marian Tetiva, care a trimis-oredactiei odata cu problema L182; s-a preferat publicarea cazului particular.

L183. Consideram numerele a ∈ Z, n ∈ N∗ si polinomul p(X) = X2 + aX + 1.Aratati ca exista un polinom cu coeficienti ıntregi qn si un numar ıntreg bn, astfelıncat p(X)qn(X) = X2n + bnX

n + 1.

Marian Tetiva, Barlad

Solutie. Demonstram prin inductie ca exista bn ∈ Z si qn ∈ Z[X], astfel ıncatX2n + 1 = (X2 + aX + 1)qn(X) − bnX

n, pentru orice n ∈ N∗. Putem considerab1 = a, q1(X) = 1; b2 = 2 − a2, q2(X) = X2 − aX + 1. Presupunem afirmatiaadevarata pentru k ≤ n si o demonstram pentru k = n + 1. Deoarece X2n+2 + 1 =(X2 + 1)(X2n + 1)−X2(X2n−2 + 1), rezulta ca X2n+2 + 1 = (X2 + 1)((X2 + aX +1)qn(X)− bnX

n)−X2((X2+aX+1) ·qn−1(X)− bn−1Xn−1) = (X2+aX+1)((X2+

1)qn(X)−X2qn−1(X))−bnXn(X2+1)+bn−1X

n+1 = (X2+aX+1)((X2+1)qn(X)−X2qn−1(X)−bnX

n)+Xn+1(bna+bn−1) = (X2+aX+1)qn+1(X)−bn+1Xn+1, unde

bn+1 = −abn − bn−1, iar qn+1(X) = (X2 + 1)qn(X) − X2qn−1(X) − bnXn si astfel

demonstratia este completa.

L184. Aratati ca functia f : N∗×N∗ → N∗, f(x, y) =x2 + y2 + 2xy − x− 3y + 2

2,

este bijectiva.

Silviu Boga, Iasi

Solutie. Avem ca f(x, y) =(x+ y − 2)(x+ y − 1) + 2x

2= [1 + 2 + . . . + (x +

y − 2)] + x ∈ N∗, ∀(x, y) ∈ N∗ × N∗, deci f este bine definita. Pentru orice n ∈ N∗,exista o unica pereche (xn, yn) ∈ N∗ × N∗ cu proprietatea ca f(xn, yn) = n, anume

xn = n −�p2

�, yn = 1 − p − xn, unde p =

�1 +

√8n

2

�. Rezulta astfel ca f este

bijectiva. De fapt, definirea lui f a fost inspirata de procedeul diagonal al lui Cantor;se observa ca f(1, 1) = 1, f(1, 2) = 2, f(2, 1) = 3, f(1, 3) = 4, f(2, 2) = 5, f(3, 1) = 6,f(1, 4) = 7, f(2, 3) = 8 etc.

L185. Fie f : R → R o functie cu proprietatea ca |f(x+y)−f(x)−f(y)| ≤ |x−y|,∀x, y ∈ R. Aratati ca lim

x→0f(x) = 0 daca si numai daca lim

x→0xf(x) = 0.

Adrian Zahariuc, student, Princeton

72

Page 77: Revista (format .pdf, 6.5 MB)

Solutie. O implicatie este evidenta, deci ne vom ocupa doar de cea de-a doua.Vom demonstra mai ıntai ca, daca lim

x→0xf(x) = 0, atunci lim sup

x→0|f(x)| < +∞.

Pentru aceasta, sa presupunem prin absurd ca exista un sir xn → 0 astfel ıncat

f(xn) → ±∞. Deducem ca xn +1

f(xn)→ 0; conform inegalitatii din enunt, rezulta

ca limn→∞

�f(xn +

1

f(xn)

�− f(xn)− f

�1

f(xn)

�= 0, de unde

limn→∞

�xn +

1

f(xn)

��f

�xn +

1

f(xn)

�− f(xn)− f

�1

f(xn)

��= 0.

Insa limn→∞

�xn +

1

f(xn)

�f

�xn +

1

f(xn)

�= 0 si

limn→∞

�xn +

1

f(xn)

��f(xn) + f

�1

f(xn)

��=

= 1 + limn→∞

xnf(xn) ·1

f(xn)· f�

1

f(xn)

�= 1

si astfel se ajunge la o contradictie. Ramane ca lim supx→0

|f(xn)| < ∞.

Se observa usor ca f(2x) = 2f(x), deci

lim supx→0

|f(x)| = lim supx→0

|f(2x)| = 2 lim supx→0

|f(x)|.

Deducem ca lim supx→0

|f(x)| = 0, deci limx→0

f(x) = 0, ceea ce trebuia demonstrat.

Solutiile problemelor pentru pregatireaconcursurilor propuse ın nr. 2/2010

A. Nivel gimnazial

G186. Fie m,n, x, y numere naturale nenule astfel ıncat m < n,m+ n

(m,n)este

numar par, iar xm = yn. Demonstrati ca x− y nu este patrat perfect.Geanina Havarneanu, Iasi

Solutie. Cum xm = yn, numerele x si y au aceeasi factori primi distincti ındescompunerile lor: x = pα1

1 · pα22 · . . . · pαt

t , y = pβ1

1 · pβ2

2 · . . . pβt

t , cu mαi = nβi,∀i = 1, t. Fie (m,n) = d si m = dr, n = dq, unde (r, q) = 1. Cum m + n = 2dk,k ∈ N∗, atunci r+ q = 2k; din rαi = qβi, ∀i = 1, t si (r, q) = 1, deducem ca αi = qγi,βi = rγi, γi ∈ N∗, deci αi − βi = γi(q − r) = γi(2k − r) − r) = 2γi(k − r), ∀i = 1, t,

unde k − r =1

2(q − r) = 0. Atunci x − y = pα1

1 . . . pαtt (pα1−β1

1 · . . . · pαt−βt

t − 1) =

73

Page 78: Revista (format .pdf, 6.5 MB)

pα11 · . . . · pαt

t [(pγ1(k−r)1 · . . . · pγt(k−r)

t )2 − 1]. Paranteza nu poate fi patrat perfect, fiindde forma u2 − 1 si deducem ca nici x− y nu poate fi patrat perfect.

G187. Consideram numerele naturale a, b, c, d si x, astfel ıncat ac, bd si ad + bcsunt divizibile cu x. Aratati ca bc se divide cu x.

Ciprian Baghiu, IasiSolutie. Fie x = pα1

1 ·pα22 · . . . ·pαt

t descompunerea ın factori primi a numarului x;vom demonstra ca pαi

i | bc, ∀i = 1, t. Pentru i ∈ {1, 2, . . . , t} oarecare, din x|ac si x|bdrezulta ca pαi

i |ac si pαii |bd, deci p2αi

i |abcd = (ad)(bc). Atunci pαii |bc sau pαi

i |ad; ın cel

de-al doilea caz, cum pαii |ad + bc, vom avea ca pαi

i |bc. In concluzie, pαii |bc, ∀i = 1, t,

prin urmare x|bc.G188. Determinati tripletele (p, q, r) ∈ Z3 pentru care

[apq + bpq + (ap + bp)q]r = r[apqr + bpqr + (ap + bp)qr], ∀a, b ∈ R.

Temistocle Bırsan, IasiSolutie. Luand a = 1 si b = 0 obtinem 2r = 2r si, deci, r = 1 sau r = 2. In

cazul r = 1, se constata direct ca egalitatea din enunt este adevarata ∀a, b ∈ R si∀p, q ∈ Z. Ca urmare, (p, q, 1), p, q ∈ Z, sunt solutii ale problemei. In cazul r = 2,luand a = b = 1 obtinem (2 + 2q)2 = 2(2 + 22q) sau, dupa reduceri, 2q+2 = 22q, deunde q = 2. Se arata prin calcul direct ca

[a2p + b2p + (ap + bp)2]2 = 2[a4p + b4p + (ap + bp)4], ∀a, b ∈ R

si, ca urmare, tripletele (p, 2, 2), p ∈ Z sunt solutii ale problemei. Intr-adevar, ridicandla patrat si apoi grupand, avem:

a4p + b4p + (ap + bp)4 + 2a2pb2p + 2a2p(ap + bp)2 + 2b2p(a2 + b2)2 =

= 2a4p + 2b4p + 2(ap + bp)4

⇔ 2a2pb2p + 2a2p(ap + bp)2 + 2b2p(ap + bp)2 = a4p + b4p + (ap + bp)4 ⇔

⇔ (ap + bp)2[2a2p + 2b2p − (a2p + 2apbp + b2p)] = a4p + b4p − a2pb2p ⇔⇔ (ap + bp)2(a2p + b2p − 2apbp) = (a2p − b2p)2, adevarata

Rezumand, relatia din enunt este adevarata pentru (p, q, 1), p, q ∈ Z, si (p, 2, 2), p ∈ Z.G189. Se considera trei sfere avand volumele v1, v2, v3, ariile s1, s2, s3 si lungimile

cercurilor ecuatoriale l1, l2, l3. Demonstrati ca

v1l1 + l2

+v2

l2 + l3+

v3l3 + l1

≥ 1

12π(s1 + s2 + s3).

D.M. Batinetu-Giurgiu, Bucuresti

Solutie. Cum v1 =4πr313

, s1 = 4πr21, l1 = 2πr1 si analoagele, inegalitatea de

demonstrat devine

r31r1 + r2

+r32

r1 + r3+

r33r3 + r1

≥ 1

2(r21 + r22 + r23).

74

Page 79: Revista (format .pdf, 6.5 MB)

Insax3

x+ y≥ 5x2 − y2

8, ∀x, y ∈ R∗

+: dupa calcule, aceasta relatie poate fi scrisa sub

forma (3x+ y)(x− y)2 ≥ 0, ceea ce este adevarat pentru x, y ∈ (0,+∞), cu egalitatecand x = y. Astfel,X r31

r1 + r2≥ 1

8

X(5r21 − r22) =

1

2(r21 + r22 + r23)

cu egalitate cand cele trei sfere sunt congruente.

G190. Daca x si y sunt numere irationale astfel ıncat x2+y, y2+x si x+y suntrationale, demonstrati ca xy < 1

4 .

Neculai Moraru, Suceava si Silviu Boga, Iasi

Solutie. Cu notatiile x2 + y = A ∈ Q, y2 + x = B ∈ Q si x + y = C ∈ Q, avemca A−B = C(C − 1)− 2y(C − 1), unde A−B, C(C − 1) si 2(C − 1) sunt rationale,iar y este itational. Rezulta ca 2(C − 1) = 0, prin urmare C = 1. Pe de alta parte,

x2 − x ≥ −1

4si y2 − y ≥ −1

4, deci (x2 − y) + (y2 − x) ≥ −1

2, inegalitate echivalenta

cu (x+ y)2 − 2xy − (x+ y) > −1

2. Cum x+ y = 1, se obtine ca xy <

1

4.

G191. Fie A = {1, 2, . . . , n}, unde n ∈ N, n ≥ 5. Demonstrati ca A poate fi scrisaca reuniunea a doua multimi disjuncte B si C, astfel ıncat produsul elementelor luiB sa fie egal cu suma elementelor lui C.

Gheorghe Iurea, Iasi

Solutie. Cautam B de forma B = {1, a, b}, iar C = A\B. Din 1 · a · b =

(1 + 2 + . . . + n) − (1 + a + b), obtinem ca (a + 1)(b + 1) =n(n+ 1)

2. Daca n este

par, putem considera a =n

2− 1, b = n (pentru n ≥ 5, numerele 1, n si

n

2− 1 sunt

distincte), iar daca n este impar, luam a = n − 1, b =n− 1

2(din nou, numerele 1,

n− 1 sin− 1

2sunt distincte pentru n ≥ 5).

G192. Demonstrati ca ın orice triunghi neisoscel ABC, exista M,N ∈ [BC],P,Q ∈ [AB] si R,S ∈ [AC] astfel ıncat segmentele [AU ], [BT ] si [CV ] sunt laturi aleunui triunghi ascutitunghic, oricare ar fi punctele U ∈ [MN ], V ∈ [PQ] si T ∈ [RS].

Marius Dragan, Bucuresti

Solutie. Daca u, v ∈ R sunt astfel ıncat 0 < u < v < 2u, atunci oricare numerex < y < z din intervalul [u, v] sunt laturi ale unui triunghi, ıntrucat y ≥ u > v − u ≥z − x, deci x + y > z. Daca, ın plus, v < u

√2, atunci numerele oarecare x < y < z

din [u, v] pot forma chiar un triunghi ascutiunghic, dupa cum se observa usor.

Consideram cazul ın care laturile triunghiului ABC verifica c < a < b; atuncihb < ha < hc. Daca D ∈ [BC], E ∈ [AC], F ∈ [AB] cu AD = x, BE = y,CF = z, avem ca ha ≤ x ≤ b, hb ≤ y ≤ a, hc ≤ z ≤ b. In plus, hc ≤ adeoarece sinB ≤ 1, prin urmare hb < ha < hc ≤ a < b. Rezulta ca existaM,N ∈ [BC], P,Q ∈ [AB] si R,S ∈ [AC] astfel ıncat pentru orice U ∈ [MN ],V ∈ [PQ] si T ∈ [RS], lungimile segmentelor AU,BV si CT sa apartina intervalu-

75

Page 80: Revista (format .pdf, 6.5 MB)

lui [hc, a]. Cum a <√2hc

�90◦ > B > 60◦ > 45◦ ⇒ sinB >

1√2⇒ hc

a>

1√2

�, din

observatia initiala urmeaza concluzia problemei.

G193. Folosind efectiv 24 bete de chibrit, construiti un poligon avand.

.

. .

.

..

..

..

.

..

.

.

.

.

.

.

.

.

.

.

.

.

. .

. .

. .

.

.

.

.

.

.

.

.

.

.

.

..

..

.

aria de 7 unitati, unde unitatea de arie estepatratul cu latura de un bat de chibrit. (Inlegatura cu o problema din ”Amuzamentematematice” de Martin Gardner)

Dumitru Mihalache, BarladSolutie. Prezentam alaturat doua posi-

bile constructii; lasam ın seama cititorului jus-tificarea faptului ca ariile celor doua poligoanesunt egale cu 7!

G194. Fie ABCD un patrulater convex, iar semidreptele [CK si [CL sunt bi-

sectoarele unghiurilor ÕACD, respectiv ÕACB (K ∈ AD,L ∈ AB). Demonstrati cadreapta LK trece prin centrul cercului ınscris ın triunghiul ABD daca si numai dacapatrulaterul ABCD este inscriptibil.

Neculai Roman, Mircesti (Iasi)Solutie. Fie I centrul cercului ınscris ın triunghiul ABD. Folosind un rezultat

K

I

A B

CD

L

.

demonstrat de N. Oprea ın articolul Ceviene de rang k,

ın G.M. 8/1989, avem: I ∈ KL ⇔ ABKD

KA+ AD

LB

LA=

BD ⇔ ABCD

CA+AD

BC

AC= BD ⇔ AB ·CD+AD ·BC =

AC ·BD ⇔ ABCD este inscriptibil.G195. Punctele M si P apartin laturii neparalele

(AD) a trapezului ABCD, astfel ıncat M ∈ [AP ]. Par-alelele prin M si P la baze intersecteaza BC ın N , respectiv Q. Demonstrati catrapezele ABNM si PQCD au diagonalele respectiv paralele daca si numai daca CDsi PQ sunt direct proportionale cu MN si AB (In legatura cu G105 din RecMat1/2006.)

Claudiu Stefan Popa, IasiSolutie. Vom arata mai ıntai ca PC∥AN ⇔ DQ∥BM . Din teorema paralelelor

neechidistate deducem caDP

PM=

CQ

QNsi

PM

AM=

QN

BN, de unde, prin ınmultire

membru cu membru, se obtine caDP

AM=

CQ

BN. Acum, daca PC∥AN , din asemanarea

triunghiurilor CPD si NAM rezultaDP

AM=

CD

MNsi atunci

CQ

BN=

CD

MN. Observand

D C

P

M

A B

Q

N

E

Fca ÕDCQ ≡ ÖMNB (corespondente), rezulta ca△DCQ ∼ △MNB si cum CD∥MN obtinemDQ∥BM. Reciproc, din DQ∥BM se arata similarca PC∥AN.

Sa aratam, ın continuare, ca din PC∥AN si,

deci, DQ∥BM , rezultaCD

MN=

PQ

AB. Notam

76

Page 81: Revista (format .pdf, 6.5 MB)

DP

AP= α si

DM

AM= β, de unde

DP

AD=

α

α+ 1si

AM

AD=

1

β + 1. Cum PQ∥AE, avem △DPQ ∼ △DAE, de unde

DP

AD=

PQ

AEsi

deciPQ

AE=

α

α+ 1. Deoarece BM∥DQ, △ABM ∼ △AED si atunci

AB

AE=

AM

AD,

deciAB

AE=

1

β + 1. Impartind membru cu membru aceste egalitati, obtinem

PQ

AB=

α(β + 1)

α+ 1. Din asemanarea triunghiurilor AMN si ADF deducem ca

AM

AD=

MN

DF

si tinand cont de notatiile facute, rezultaMN

DF=

1

β + 1. Din asemanarea triunghi-

urilor DCP si DFA obtinemCD

DF=

DP

AD, deci

CD

DF=

α

α+ 1. Impartind membru cu

membru egalitatile gasite obtinem caCD

MN=

α(β + 1)

α+ 1. In concluzie,

CD

MN=

PQ

AB.

Reciproc,CD

MN=

PQ

AB, sa aratam ca PC∥AN. Presupunem, prin reducere la ab-

surd ca PC ∦ AN. Fie N ′ ∈ BC, N ′ = N astfel ıncat AN ′∥PC si paralela prinN ′ la AB intersecteaza AD ın M ′. Conform celor deja demonstrate, rezulta caCD

M ′N ′ =PQ

ABsi ımpreuna cu ipoteza asumata obtinem MN = M ′N ′ ⇒ MNN ′M ′

paralelogram, de unde MM ′∥NN ′ ⇔ BC∥AD, ın contradictie cu ABCD trapez.

B. Nivel licealL186. Determinati masurile unghiurilor triunghiului ABC, stiind ca

tgA+ 2ptgB + 3 3

ptgC = 6 6

ptgA · tgB · tgC.

Catalin Calistru, Iasi

Solutie. In orice triunghi ABC,

(∗) tgA+ tgB + tgC = tgA · tgB · tgC.

Pe de alta parte, este cunoscuta inegalitatea mediilor ponderata

mx1m + ny

1n + pz

1p ≥ (m+ n+ p)(xyz)

1m+n+p ,∀m,n, p ∈ N∗,∀x, y, z ∈ R∗

+

cu egalitate daca si numai daca x1m = y

1n = z

1p . Considerand m = 1, n = 2, p = 3,

x = tgA, y = tgB, z = tgC, obtinem ca tgA =√tgB = 3

√tgC = a; ınlocuind ın

(*), deducem ca a+a2+a3 = a6, adica a(a2+1)(a3−a−1) = 0. Singura solutie reala

si nenula a acestei ecuatii este a0 = 3

r2

27 + 3√69

+1

3

3

r27 + 3

√69

2(se obtine, de

exemplu, utilizand formulele lui Cardano). In concluzie, A = arctg a0, B = arctg a20si C = arctg a30.

L187. Se considera triunghiul ABC si punctele D,L ∈ (BC), E,M ∈ (CA)si F,N ∈ (AB). Notam {P} = FE ∩ AL, {Q} = DF ∩ BM, {S} = ED ∩ CN .

77

Page 82: Revista (format .pdf, 6.5 MB)

Demonstrati ca daca doua dintre urmatoarele afirmatii sunt adevarate, atunci esteadevarata si a treia:

i) dreptele AD,BE si CF sunt concurente;ii) dreptele AL,BM si CN sunt concurente;iii) dreptele DP,EQ si FS sunt concurente.

Titu Zvonaru, ComanestiSolutie. Conform teoremei lui Ceva (directa si reciproca), conditiile din enunt

sunt echivalente cuDB

DC

EC

EA

FA

FB= 1,

LB

LC

MC

MA

NA

NB= 1, respectiv

PE

PF

QF

QD

SD

SE= 1.

A

B

C

NM

E

QS

F

D L

P

Folosind (R1) din T. Zvonaru, B. Ionita - Rapoartedeterminate de o ceviana si o secanta ıntr-un triunghi,

RecMat 1/2005, obtinem caAF

AB

LB

LC

AC

AE

PE

PF= 1, de

undePE

PF=

AB

AF

LC

LB

AE

AC. Analog se deduc relatiile

QF

QD=

BC

BD

MA

MC

BF

ABsi

SD

SE=

AC

EC

NB

NA

DC

BC. Inmultind

membru cu membru aceste egalitati, rezulta ca�PE

PF

QF

QD

SD

SE

�=

�FB

FA

EA

EC

DC

DB

��LC

LB

NB

NA

MA

MC

�.

Daca doua dintre paranteze sunt egale cu 1, atunci si cea de-a treia este tot 1 si deaici urmeaza concluzia problemei.

Nota autorului. In problema L86 din RecMat 2/2005, dreptele AD,BE si CFsunt concurente ın punctul lui Nagel. Daca IaL este bisectoarea unghiului BIaC, cuteorema bisectoarei si teorema sinusurilor, obtinem ca

LB

LC=

IaB

IaC=

sinBCIasinCBIa

=sin(90◦ − C

2 )

sin(90◦ − B2 )

=cos C

2

cos B2

si, cu reciproca teoremei lui Ceva, rezulta ca dreptele AL, BM si CN sunt concurente.

L188. Fie Oxy un reper cartezian, dreapta d : x = 1 si un punct M ∈ d. PunctulP se obtine din M printr-o rotatie de centru O si unghi 2α, unde α este unghiul

orientat ÕyOM . Scrieti ecuatia y = f(x) a curbei descrise de P atunci cand Mparcurge dreapta d si desenati graficul acestei curbe.

Temistocle Bırsan, Iasi

Solutie. Observam ca panta dreptei OM este λ = tg�π2+ α

�= − ctgα, iar

y d

0

M(1, )l

P(x,y)a

2a

cea a dreptei OP este data de tg�π2+ 3α

�=

− ctg 3α = −ctg3 α− 3 ctgα

3 ctg2 α− 1=

λ(λ2 − 3)

3λ2 − 1. Coordo-

natele punctului P , functie de λ, se afla rezolvandsistemul format din ecuatiile cercului C(O,OM) sidreptei OP :

(1) x2 + y2 = λ2 + 1, y =λ(λ2 − 3)

3λ2 − 1x.

78

Page 83: Revista (format .pdf, 6.5 MB)

Inlocuind ın prima ecuatie pe y cu expresia lui data de a doua ecuatie, vom obtine

x2 =(3λ2 − 1)2

(λ2 + 1)2. Aceasta ecuatie combinata cu a doua din (1) conduce la ecuatiile

parametrice ale curbei:

(2) x =3λ2 − 1

λ2 + 1, y =

λ(λ− 3)

λ2 + 1.

Pentru a obtine ecuatia explicita a curbei, din prima relatie din (2) deducem λ2 =1 + x

3− xsi, deci, λ = ±

É1 + x

3− x. Inlocuind aceasta expresie a lui λ ın a doua ecuatie

din (2), obtinem ecuatia curbei sub forma explicita:

(3) y = ±(x− 2)

r1 + x

3− x, x ∈ [−1, 3).

Curba este simetrica ın raport cu axa x-lor; vom studia numai variatia ramurii cu”+”. Dreapta x = 3 asimptota verticala, iar punctul x = 2, y = 0 este un nod al

curbei. Prin calcul gasim (pentru ramura ”+”): y′ = −É

3− x

1 + x

(x− 2)2 − 3

(3− x)2. Suntem

condusi la urmatorul tabel al variatiei functiei:x -1 2-

√3 2 3

y′ | - 0 + + |y 0 ↘ -

È6√3− 9 ↗ 0 ↗ +∞

Graficele ramurii studiate si ale celei simetrice sunt indicate pe figura. Curbaobtinuta este de tip strofoida.

L189. Un sistem de n ≥ 3 puncte coplanare A1, A2, . . . , An, oricare trei necol-iniare, se numeste bun atunci cand oricare ar fi 1 ≤ i < j < k ≤ n si ori-care ar fi punctele M ∈ [AiAj ], N ∈ [AjAk] si P ∈ [AkAi], are loc inegalitateaMN2 +NP 2 +MP 2 ≤ AiA

2j +AjA

2k +AkA

2i . Determinati sistemele bune cu numar

maxim de puncte.Vlad Emanuel, student, Bucuresti

Solutie. Demonstram ıntai ca varfurile oricarui triunghi ABC neobtuzunghic

formeaza un sistem bun. Pentru aceasta, fie α =AM

AB, β =

BN

BC, γ =

CP

CA,

A

B C

M

N

P

cu α, β, γ ∈ [0, 1]. Conform teoremei cosinusului, avem caMN2 +NP 2 + PM2 = [α2 + (1− α)2]c2 + [β2 + (1− β)2]a2 +[γ2+(1−γ)2]b2−2(1−α)βac cosB−2(1−β)γab cosC−2(1−γ)αbc cosA. Fixand β si γ, observam caMN2+NP 2+PM2 estefunctie de gradul II ın α, avand coeficientul dominant 2c2 > 0;aceasta functie ısi atinge maximul ıntr-unul dintre capetele intervalului ın care variazaα, deci pentru α ∈ {0, 1}. Din considerente de simetrie, vom avea ca β, γ ∈ {0, 1},asadar suma MN2 + NP 2 + PM2 este maxima cand M ∈ {A,B}, N ∈ {B,C}si P ∈ {A,C}. Analizand cazurile posibile, obtinem ca MN2 + NP 2 + PM2 ≤max{a2 + b2 + c2, 2a2, 2b2, 2c2}. Insa a2 ≤ b2 + c2, b2 ≤ c2 + a2 si c2 ≤ a2 + b2,ıntrucat △ABC este neobtuzunghic si atunci maximul de mai sus este a2 + b2 + c2,ceea ce dovedeste ca A,B,C formeaza un sistem bun.

79

Page 84: Revista (format .pdf, 6.5 MB)

Sa aratam acum ca, daca A,B,C sunt varfurile unui triunghi obtuzunghic, ele nuformeaza un sistem bun. Fie bA unghiul obtuz si consideram M = B, N = C, iarP ∈ (AC), cu PC = αAC; demonstram ca exista α ∈ (0, 1) pentru care MN2 +BP 2 + PC2 > AB2 + AC2 + AB2. Aceasta inegalitate revine la f(α) = 2b2α2 −2ab cosCα+ a2 − b2 − c2 > 0. Functia f este continua si f(α) = 0, deci exista α > 0suficient de mic pentru care f(α) = 0.

In concluzie, sistemul A1, A2, . . . , An este bun daca si numai daca oricare ar fi1 ≤ i < j < k ≤ n, triunghiul AiAjAk este ascutitunghic sau dreptunghic. Cumdorim ca n sa fie maxim, trebuie ca n = 4, iar A1A2A3A4 sa fie dreptunghi; pentrudemonstratie, se poate consulta solutia problemei M130 din Kvant (ın traducerearomaneasca a lui H. Banea, la pagina 171).

L190. Demonstrati ca

8√15 ≤ (

√x+

√x+ 48)(

√16− x+

√1− x) ≤ 36, ∀x ∈ [0, 1].

Gheorghe Iurea, Iasi

Solutia 1 (a autorului). Cum√16− x +

√1− x =

15√16− x−

√1− x

, inegali-

tatea din stanga revine la 8(√16− x−

√1− x) ≤ (

√x+

√x+ 48)

√15. Scriem aceasta

inegalitate sun formaÈ15(x+ 48) + 8

√1− x ≥ 8

√16− x −

√15x; prin ridicare la

patrat, dupa efectuarea calculelor, ajungem laÈ(1− x)(x+ 48)+

Èx(16− x) ≥

√15.

Primul radical este nenegativ, iar al doilea este cel putin egal cu√15: x(16 − x) ≥

15 ⇔ (x − 1)(x − 115) ≤ 0, adevarat pentru x ∈ [0, 1]. Egalitatea se atinge candx = 1.

Procedand analog, inegalitatea din dreapta revine la 4(√16− x +

√1− x) ≤

3(√x+ 48−

√x), adica 4

√16− x+3

√x ≤ 3

√x+ 48−4

√1− x, ceea ce este echivalent

cuÈx(16− x)+

È(x+ 48)(1− x) ≤ 8. Insa, conform inegalitatii CBS, [

Èx(16− x)+È

(1− x)(x+ 48)2] ≤ (x+ 1− x)(16− x+ x+ 48) = 64, cu egalitate candx

16− x=

1− x

x+ 48, deci pentru x =

16

65.

Solutia 2 (Titu Zvonaru). Consideram functia f : [0, 1] → R, f(x) = (√x +√

48 + x)(√16− x+

√1− x). Avem ca

f ′(x) =1

2

(√x+

√48 + x)(

√16− x+

√1− x)È

x(48 + x)(16− x)(1− x)(È(16− x)(1− x)−

Èx(48 + x))

si rezulta ca f ′(x) = 0 ⇔ x =16

65; pentru x ∈

�0,

16

65

�; f ′(x) > 0, iar pentru

x ∈�16

65, 1

�, avem ca f ′(x) < 0. Deducem ca f admite un maxim pentru x =

16

65,

egal cu f

�16

65

�= 36, iar f(x) ≥ min(f(0), f(1)) = min(5

√48, 8

√15) = 8

√15, de

unde concluzia.

80

Page 85: Revista (format .pdf, 6.5 MB)

L191. Aratati ca, pentru orice numere reale pozitive x, y, z, are loc inegalitatea

(x2 + y2 + z2)

�1

(x+ y)(x+ z)+

1

(x+ y)(y + z)+

1

(x+ z)(y + z)

�≥ 9

4≥

≥ (xy + xz + yz)

�1

(x+ y)(x+ z)+

1

(x+ y)(y + z)+

1

(x+ z)(y + z)

�.

Marian Tetiva, BarladSolutie. Vom folosi notatiile S = x+y+z, Q = xy+yz+zx, P = xyz; se verifica

usor ca (x+y)(y+z)(z+x) = SQ−P si atunci inegalitatea din dreapta este echivalenta

cu Q2S

SQ− P≤ 9

4, adica 9P ≤ SQ si aceasta din urma se reduce la

Px(y− z)2 ≥ 0,

deci este adevarata. Ramane sa justificam inegalitatea din stanga. Cum (a+b+c)3 ≥27abc (inegalitatea mediilor), rezulta ca (a+b+c)2

�1

ab+

1

ac+

1

bc

�≥ 27. Consideram

a = x+ y, b = x+ z, c = y + z si obtinem ca

(X

x)2X 1

(x+ y)(x+ z)≥ 27

4⇒X

x2X 1

(x+ y)(x+ z)≥ 27

4−

− 2X

xyX 1

(x+ y)(x+ z)⇒X

x2X 1

(x+ y)(x+ z)≥ 27

4− 2

9

4=

9

4,

pentru ultima implicatie tinand seama de inegalitatea din dreapta, deja demonstrata.

Nota. Solutie corecta a dat dl. Titu Zvonaru, Comanesti

L192. Pe P multimea polinoamelor unitare avand coeficientii ın intervalul [0, 9].Determinati supremumul modulelor radacinilor complexe ale polinoamelor din P.

Adrian Reisner, ParisSolutie. Pentru n ∈ N∗, notam rn = sup{|z|;P (z) = 0, P ∈ P, grP = n}.

Evident ca r1 = 9. Pentru n ≥ 2, consideram polinomul Q = Xn− 9Xn−1− 9Xn−3−9Xn−5 − . . .; cum Q(9) < 0 si lim

x→∞Q(x) = ±∞, rezulta ca Q admite o cea mai

mare radacina reala αn ∈ (9,+∞). Vom demonstra ca rn = αn; pentru aceasta, estedestul sa aratam ca, daca P = Xn − an−1X

n−1 + . . . + a1X + a0 ∈ P si z = reiθ

este radacina a lui P , atunci r ≤ αn. Putem presupune ca r > 1 si ca θ ∈ [0, π](considerand eventual radacina z a lui P ∈ R[X]).

Daca θ ∈�0,

3

�, din P (z) = 0 si tinand seama ca r ∈ R, obtinem ca

rn = −an−1e−iθrn−1 − . . .− a0e

−inθ = −an−1rn−1 cos θ − . . .− a0 cosnθ ≤

≤ an−1

2rn−1 + an−2r

n−2 + . . . a0,

ıntrucat cos θ ≥ −1

2, ∀θ ∈

�0,

3

�, deci cos kθ ≥ −1, ∀k ≥ 2. Cum ak < 9, deducem

rn ≤ 9

2rn−1 + 9(rn−2 + . . .+ 1) =

9

2rn−1 + 9

rn−1 − 1

r − 1≤

≤ 9

2rn−1 + 9

rn−1

r − 1⇒ r ≤ 9

2+

9

r − 1.

81

Page 86: Revista (format .pdf, 6.5 MB)

Functia f(x) =9

2− 9

x− 1−x are derivata negativa, deci este descrescatoare pe (1,∞).

Cum f(7) < 0 si f(r) ≥ 0, rezulta ca r < 7 < αn.

Daca θ ∈�2π

3, π

�, atunci |a + beiθ| ≤ max{a, b}, ∀a, b ∈ R+, dupa cum rezulta

imediat. Din zn + zn−2(an−1z+ an−2)+ zn−4(an−3z+ an−4)+ . . . = 0, tinand seamade inegalitatea triunghiului, obtinem ca

rn ≤ rn−2 max{an−1r, an−2}+ rn−4 max{ab−3r, an−4}+ . . . ≤ 9rn−1 + 9rn−3 + . . . ,

prin urmare Q(rn) ≤ 0. Cum αn este cea mai mare radacina reala a lui Q silimx→∞

Q(x) = +∞, deducem ca r ≤ αn.

Ramane sa mai gasim sup{αn|n ∈ N∗}, unde αn este cea mai mare solutie reala

a ecuatiei xn = 9(xn−1 + xn−3 + . . .). Daca n = 2p, atunci α2p2p = 9α2p

α2p2p − 1

σ2p2p − 1

, de

unde α2p−12p (α2

2p − 9α2p − 1)+ 9 = 0. Insa limp→∞

α2p−12p = +∞, prin urmare lim

p→∞(α2

2p −

9α2p−1) = 0. Pe intervalul [9,+∞), functia h(x) = x2−9x−1 este continua si strictcrescatoare, deci este inversabila, cu inversa continua. Astfel, lim

p→∞α2p = h−1(0) =

9 +√85

2; mai mult, h(α2p) < 0 si atunci α2p < α =

9 +√85

2. La fel se arata ca

limp→∞

α2p+1 = α si α2p+1 < α. In concluzie, α =9 +

√85

2este supremumul cautat.

L193. Fie A ∈ M3(R) o matrice simetrica, avand urma egala cu determinantul.Demonstrati ca 2 · Tr(An+2) ≥ Tr A · (Tr(An+1)− Tr(An−1)), ∀n ∈ N∗.

Paul Georgescu si Gabriel Popa, IasiSolutie. Fie a, b, c valorile proprii ale matricei A (solutiile ecuatiei det(xI3−A) =

0). Matricea A fiind simetrica, vom avea ca a, b, c ∈ R. Intr-adevar, daca x este unvector propriu nenul corespunzator valorii proprii a, atunci Ax = ax si, prin conjugare,rezulta ca Ax = ax, de unde, trecand la transpusa, xtAt = axt, deci xtAx = axtx(am tinut seama de faptul ca A = At). Pe de alta parte, din Ax = ax, se obtineca xtAX = axtx, prin urmare (a − a)xtx = 0. Intrucat x este nenul, deducem caanuleaza paranteza si atunci a este real. Analog se procedeaza pentru b si c.

Cum TrA = detA, urmeaza ca a+ b+ c = abc (de exemplu, considerand matriceadiagonala echivalenta cu A sau folosind relatiile lui Viete pentru polinomul caracter-istic al lui A). Mai observam ca matricea An are urma egala cu an + bn + cn, oricarear fi n ∈ N∗ (folosind tot forma diagonala a lui A).

In acest fel, inegalitatea de demonstrat se scrie succesiv: 2(an+2 + bn+2 + cn+2) ≥abc(an+1+bn+1+cn+1−an−1−bn−1−cn−1) ⇔ an+2+bn+2+cn+2+anbc+bnac+cnab ≥an+1(abc−a)+bn+1(abc−b)+cn+1(abc−c) ⇔ an+2+bn+2+cn+2+anbc+bnac+cnab ≥an+1(b+ c) + bn+1(a+ c) + cn+1(a+ b) ⇔ an(b− a)(c− a) + bn(a− b)(c− b) + cn(a−c)(b− c) ≥ 0, iar aceasta ultima inegalitate este cunoscuta (inegalitatea lui Schur).

L194. Fie p ≥ 2 un numar natural si ecuatia1

p√1 + x

+1

p√2 + x

+ . . .+1

p√n+ x

=

p√xp−1. Demonstrati ca, pentru fiecare n ∈ N∗, ecuatia are o unica solutie pozitiva

82

Page 87: Revista (format .pdf, 6.5 MB)

xn, aratati ca sirul (xn)n≥1 este monoton si calculati limn→∞

n

xn.

Gabriel Dospinescu, student, Paris

Solutie. Pentru n ∈ N∗, consideram functia fn : (0,∞) → R, fn(x) =nX

k=1

1p√k + x

p√xp−1. Evident, fn este continua si lim

x→0fn(x) > 0, lim

x→∞f(x) = −∞, deci Imfn =

(−∞, A), cu A > 0. Cum fn este si strict descrescatoare, rezulta ca se anuleaza exacto data pe (0,∞). Observam acum ca

pÈxp−1n+1 −

pÈxp−1n =

1p√n+ 1 + xn+1

+nX

k=1

�1

ppk + xn+1

− 1p√k + xn

�.

Daca, prin absurd, ar exista n pentru care xn+1 ≤ xn, membrul stang al identitatiiprecedente ar fi negativ sau nul, ın timp ce membrul drept ar fi strict pozitiv, imposi-bil. In concluzie, sirul (xn)n≥1 este strict crescator.

Folosind teorema lui Lagrange sau inegalitatea mediilor, obtinem ca

p

p− 1(

p√xp−1 − p

È(x− 1)p−1) >

1p√x>

p

p− 1( p

È(x+ 1)p−1 − p

√xp−1), ∀x > 1,

de undep

p− 1( p

È(xn + n)p−1)− p

Èxp−1n >

nXk=1

1p√xn + k

>p

p− 1( p

È(xn + n+ 1)p−1−

p

È(xn + 1)p−1). Deducem ca

2p− 1

p− 1

xp−1n <

p

p− 1p

È(xn+n)p−1, adica

(∗) n

xn>

�2− 1

p

� pp−1

− 1.

De asemenea, mai avem sip

p− 1p

È(xn + n+ 1)p−1 <

2p− 1

p− 1p

È(xn + 1)p−1, de unde

n

xn<

��2− 1

p

� pp−1

− 1

��1 +

1

xn

�. Folosind criteriul clestelui, cum lim

n→∞xn = +∞

(din (*)), rezulta ca limn→∞

n

xn=

�2− 1

p

� pp−1

− 1.

L195. Fie (an)n≥1 un sir de numere reale mai mari ca 1, astfel ıncat an+1 ≥2an − 1, ∀n ∈ N∗. Definim xn =

1

a1+

1

a1a2+ . . .+

1

a1a2 . . . an, ∀n ≥ 1.

a) Demonsrati ca sirul (xn)n≥1 este convergent (notam cu x limita sa).

b) Aratati ca sirul (αn)n≥1, unde x = xn +αn

a1a2 . . . an(an+1 − 1), ∀n ≥ 1, este

monoton, convergent si determinati limita sa.Dumitru Mihalache si Marian Tetiva, Barlad

Solutie. Cum an+1 ≥ 2an−1 = an+an−1 > an, ∀n ≥ 1, rezulta ca sirul (an)n≥1

este strict crescator si se arata usor ca are limita +∞.

83

Page 88: Revista (format .pdf, 6.5 MB)

a) Evident ca (xn)n≥1 este un sir strict crescator si

xn <1

a1+

�1

a1

�2

+ . . .+

�1

a1

�n

=1

a1·1− ( 1

a1)n

1− 1a1

<1

a1 − 1, ∀n ≥ 1,

prin urmare (xn)n≥1 este convergent.

b) Daca x = limn→∞

xn si n ∈ N∗, atunci x − xn = limp→∞

pXk=1

1

a1a2 . . . an+k=

limp→∞

1

a1a2 . . . an

pXk=1

1

an+1 · . . . · an+k. Insa

pXk=1

1

an+1 . . . an+k<

pXk=1

�1

an+1

�k

<

1

an+1 − 1si rezulta ca x − xn ≤ 1

a1 . . . an(an+1 − 1), ∀n ∈ N∗; altfel spus, αn =

a1 . . . an(an+1 − 1)(x− xn) ≤ 1, ∀n ∈ N∗.Demonstram acum ca (αn)n≥1 este strict crescator; avem de aratat ca

a1 . . . an(an+1 − 1)(x− xn) < a1 . . . anan+1(an+2 − 1)(x− xn+1),

pentru orice n ∈ N∗. Cum xn+1 = xn +1

a1 . . . anan+1, dupa cateva calcule, ajungem

la inegalitatea echivalentaan+2 − 1

an+1an+2 − 2an+1 + 1< a1a2 . . . an(x − xn), ∀n ≥ 1.

Deoarece a1a2 . . . an(x − xn) = limp→∞

pXk=1

1

an+1 . . . an+k, pentru a demonstra aceasta

ultima inegalitate ar fi suficient sa aratam caan+2 − 1

an+1an+2 − 2an+1 + 1<

1

an+1+

1

an+1an+2+

1

an+1an+2an+3. Un calcul simplu aduce aceasta inegalitate la forma

2an+1 − 1− an+2

an+1an+2 − 2an+1 + 1<

1

an+1an+2an+3, care este adevarata ıntrucat membrul stang

este nepozitiv, conform enuntului, iar cel drept este evident strict pozitiv.Astfel, am aratat ca (αn)n≥1 este un sir strict crescator si, totodata, am obtinut

ıncadrarea(an+1 − 1)(an+2 − 1)

an+1an+2 − 2an+1 + 1< αn ≤ 1, ∀n ∈ N∗, din care va rezulta ca

limn→∞

αn = 1, daca mai dovedim ca limita fractiei din stanga este 1. Pentru aceasta,

observam ca(an+1 − 1)(an+2 − 1)

an+1an+2 − 2an+1 + 1= 1 − an+2 − an+1

an+1an+2 − 2an+1 + 1, iar sirul cu ter-

menul generalan+2 − an+1

an+1an+2 − 2an+1 + 1=

�1− an+1

an+2

�· 1

an+1 − 2an+1

an+2+ 1

an+2

, n ∈ N∗,

este produsul dintre un sir marginit si un sir cu limita zero. Cu aceasta, rezolvareaproblemei este completa.

84

Page 89: Revista (format .pdf, 6.5 MB)

Probleme propuse1

Clasele primareP.206. Dan a scris ın ordine descrescatoare numerele de la 75 la 23. Calculati

diferenta dintre al zecelea si al 32-lea numar.(Clasa I ) Andreea Bızdıga, eleva, Iasi

P.207. Fiecarei forme geometrice ıi corespunde un pret de cost: � → 10 lei,

→ 20 lei, △ → 30 lei, ⃝ → 20 lei. Cat costa confectionarea casutei?(Clasa I ) Mariana Nastasia, eleva, Iasi

P.208. Pe trei rafturi sunt 75 carti. Daca pe primul raft punemjumatate din cartile de pe cel de-al doilea, atunci vom avea pe rafturinumere consecutive de carti. Cate carti erau la ınceput pe fiecare raft?(Clasa a II-a) Iulia Sticea, eleva, Iasi

P.209. Numarul lalelelor dintr-o vaza este cu 7 mai mare decat numarul trandafir-ilor, care reprezinta jumatate din numarul lalelelor. Cate flori sunt ın vaza?(Clasa a II-a) Inst. Maria Racu, Iasi

P.210. Ce zi nu poate fi astazi, daca alaltaieri nu a fost luni si poimaine nu estesambata?(Clasa a III-a) Mihaela Galca, eleva, Iasi

P.211. Suma a doua numere este un numar de doua cifre al caror produs este 5.Care sunt cele doua numere, daca diferenta lor este 7?(Clasa a III-a) Ana Cojocariu, Iasi

P.212. Un elev a gresit la adunarea a doua numere: a scris cifra zero la sfarsitulprimului numar ın loc s-o scrie la sfarsitul celui de-al doilea si astfel a obtinut suma98 si nu 89 - suma corecta. Aflati cele doua numere.(Clasa a III-a) Cristina Timofte, Iasi

P.213. O vila turistica are apartamente cu 3 si 4 camere. Stiind ca ın usa deintrare a fiecarui apartement se afla cate 2 chei, iar numarul camerelor si al cheilor laun loc este 39, aflati cate apartamente cu 3 camere sunt ın vila.(Clasa a IV-a) Dorel Luchian, Iasi

P.214. La o masa rotunda stau cinci copii, fiecare avand cate un jeton pe careeste scris un numar. Toti copiii afirma ca vecinii lor au jetoane cu numere de paritatidiferite. Aratati ca macar un copil nu spune adevarul.(Clasa a IV-a) Iuliana Moldovan, studenta, Iasi

P.215. Intr-un rucsac sunt 12 sosete care pot forma sase perechi de culori diferite,iar doua dintre ele sunt rupte. Cate sosete trebuie scoase la ıntamplare din rucsacpentru a avea o pereche buna?(Clasa a IV-a) Petru Asaftei, Iasi

1Se primesc solutii pana la data de 31 iunie 2011.

85

Page 90: Revista (format .pdf, 6.5 MB)

Clasa a V-aV.130. Determinati numarul a0bb(3), daca a0bb(3) = bba(7).

Nicolae Ivaschescu, Craiova

V.131. Se considera numarul N = 1 + 7 + 13 + 19 + a + b + 37, unde termeniisumei sunt scrisi ın ordine strict crescatoare. Determinati numarul perechilor (a, b)pentru care N este patrat perfect.

Anca Chiritescu, Tiganasi (Iasi)

V.132. Numerele 0, 1, . . . , 2011 sunt aranjate ıntr-un tablou astfel:

0 7 8 15 . . . 20081 6 9 14 . . . 20092 5 10 13 . . . 20103 4 11 12 . . . 2011.

a) Stabiliti care sunt elementele coloanei 211 (de sus ın jos).b) Calculati suma elementelor de pe a treia linie.

Ioana Maria Popa, eleva, Iasi

V.133. Calculati sumele:a) S1 = 85 + 985 + 9985 + . . .+ 99 . . . 9| {z }

2011 de 9

85;

b) S2 = 17 + 197 + 1997 + . . .+ 1 99 . . . 9| {z }2011 de 9

7.

Eugeniu Blajut, Bacau

V.134. Lucian-Georges ramane nesupravegheat la calculator si, din neatentie,printeaza toate numerele naturale de 1 la 1 000 000. Drept pedeapsa, el trebuie sanumere de cate ori a fost tiparita cifra 5. Care este raspunsul corect pe care trebuiesa-l dea copilul?

Andrei Nedelcu si Catalin Budeanu, Iasi

V.135. Demonstrati ca 2122 < 1037.Titu Zvonaru, Comanesti

V.136. Se considera sirul de numere naturale 4, 19, 163, 1945, . . .. Determinatiultimele 501 cifre ale celui de-al 2011-lea termen al sirului.

Mihai Craciun, Pascani

Clasa a VI-aVI.130. Fie A1, A2, . . . , A2011 puncte echidistante pe o dreapta d. Notam cu

B1, B2, . . . , B2010 mijloacele segmentelorA1A2, A2A3, . . . , A2010A2011; apoi, fie C1, C2,. . . , C2009 mijloacele segmentelor B1B2, B2B3, . . . , B2009B2010 s.a.m.d., pana candobtinem un singur punct M . Determinati pozitia punctului M .

Elena Iurea, Iasi

VI.131. Se considera △ABC cu AC = BC si punctele M,N cu B ∈ [AM ],

N ∈ [AC] si BM = CN. Aratati ca MA = MN daca si numai daca ÖMBC ≡ ÖCNM .Petru Asaftei, Iasi

86

Page 91: Revista (format .pdf, 6.5 MB)

VI.132. Se considera triunghiurile isoscele ABC si ABD cu AB = AC = AD,

m(ÕBAC) = 28◦, m×(BAD) = 32◦, punctele C si D fiind de o parte si de alta a drepteiAB. Daca E este mijlocul segmentului AC si {M} = DE∩BC, determinati masurileunghiurilor triunghiului MAB.

Eugeniu Blajut, Bacau

VI.133. Masura unui unghi este u = ab′ab

′′, unde 0 < ab < 60, iar numarul

natural nenul n are proprietatea ca n · u exprima un numar ıntreg de grade. Aflati upentru care n este minim.

Marian Pantiruc, Iasi

VI.134. Aratati ca numarul A = 20102010 + 20122012 − 2 este divizibil cu 2011.Daniela Munteanu, Iasi

VI.135. Determinati cate fractii ireductibile si subunitare au proprietatea casuma dintre numarator si numitor este 1005.

Mirela Marin, Iasi

VI.136. Dupa ce fiecare echipa a jucat cu fiecare dintre celelalte cate un meci,clasamentul grupei A de la Campionatul mondial de fotbal 2010 arata astfel:

1. Uruguay 3 2 1 0 4-0 72. Mexic 3 1 1 1 3-2 43. Africa de Sud 3 1 1 1 3-5 44. Franta 3 0 1 2 1-4 1

Stiind ca ın meciul Uruguay-Franta nu s-a marcat niciun gol, aflati rezultatele fiecaruiadintre cele sase meciuri.

Titu Zvonaru, Comanesti

Clasa a VII-aVII.130. Fie C ′ mijlocul laturii AB a triunghiului ABC, iar P un punct pe

segmentul CC ′. Daca AP ∩BC = {R}, aratati ca AP · CR = BC · PR.Claudiu Stefan Popa, Iasi

VII.131. Intr-un triunghi ABC, lungimile laturilor verifica relatiaa

b=

b+ c

a.

Daca m( bA) = α, determinati masurile unghiurilor ÒB si ÒC.Neculai Stanciu, Buzau

VII.132. In cercul C(O, r) se considera coarda [AB]. Notam cu C mijlocul arcului

mare ÷AB, cu O′ simetricul punctului O fata de AB si fie x = m(ÕACB). Determinativalorile lui x pentru care O′ ∈ Int C.

Geanina Havarneanu, Iasi

VII.133. Se considera patratul ABCD cu latura de 3√5cm. Pe semidreptele

(AB si (BC se considera punctele M , respectiv N , astfel ıncat AM = 7√5cm si

BN = 10√5cm. Determinati masura unghiului ÖMND.

Constantin Apostol, Rm. Sarat

87

Page 92: Revista (format .pdf, 6.5 MB)

VII.134. Comparati numerele reale a = 2√1 + 2

√2 + 2

√3 + . . . + 2

√64 si

b = (√0 +

√2) + (

√1 +

√3) + (

√2 +

√4) + . . .+ (

√63 +

√65).

Cosmin Manea si Dragos Petrica, Pitesti

VII.135. Rezolvati ın numere ıntregi ecuatia x3 + y3 + 3xy = 9.Cristina Ene, eleva, Craiova

VII.136. Numerele ıntregi x, y si z verifica relatia 17x+ 5y − 2z = 0. Aratati ca

numarul A =(3x+ y)(z − x)(2x+ 2y + z)(3y + 3z − x)

210este natural, patrat perfect.

Mihai Haivas, Iasi

Clasa a VIII-aVIII.130. Date fiind punctele A,B si C, determinati punctele P din spatiu cu

proprietatea ca PA2 + PB2 − PC2 = AB2 −AC2 −BC2.Dan Popescu, Suceava

VIII.131. Demonstrati caÈ(1− x)(1 + y) +

È(1 + x)(1− y) ≤

È4− (x+ y)2,

∀x, y ∈ [−1, 1].Rodica Pop si Ovidiu Pop, Satu Mare

VIII.132. Daca a, b, c ∈ R∗+ si

√a2 + ab+ b2 +

√a2 + ac+ c2 +

√b2 + bc+ c2 =√

3(a+ b+ c), aratati ca a = b = c.D.M. Batinetu-Giurgiu, Bucuresti

VIII.133. Fie E(x, y, z) =(1 + x)(1 + y)

1 +√xy

+(1 + y)(1 + z)

1 +√yz

+(1 + z)(1 + x)

1 +√zx

.

a) Aratati ca E(x, y, z) ≥ x+ y + z + 3, ∀x, y, z ∈ (0, 1).b) Aratati ca E(x, y, z) ≤ x+ y + z + 3, ∀x, y, z ∈ (1,∞).

Ion Nedelcu, Ploiesti si Liviu Smarandache, Craiova

VIII.134. Daca a, b, c ∈ R∗+ sunt laturile unui triunghi, demonstrati ca aa

2+2ac ·

bb2+2ab · cc2+2bc <

�a+ b+ c

2

�(a+b+c)2

.

Razvan Ceuca, elev, Iasi

VIII.135. Fie n ≥ 2 un numar natural fixat si x1, x2, . . . , xn numere reale pozitiveastfel ıncat x1 + x2 + . . . + xn = n. Gasiti cea mai mica si cea mai mare valoare aleexpresiei E =

√x1 + x1x2 +

√x2 + x2x3 + . . .

√xn + xnx1.

Gheorghe Iurea, Iasi

VII.136. Fie m,n ∈ Z, m < n si x1, x2, . . . , xn−m numere ıntregi dinstincte douacate doua, cuprinse ıntre 2m si 2n. Demonstrati ca printre numerele x1, x2, . . . , xn−m

ori exista unul egal cu m+ n, ori exista doua avand suma 2m+ 2n.Dan Nedeianu, Drobeta Tr. Severin

Clasa a IX-aIX.116. Rezolvati ın numere ıntregi ecuatia 8x3 + y3 + 12xy = 8.

Vasile Chiriac, Bacau

IX.117. Determinati functiile f : N → N cu proprietatea ca f(x + y) = f(x) +f(y) + 3xy(x+ y), ∀x, y ∈ N.

Mihaly Bencze, Brasov

88

Page 93: Revista (format .pdf, 6.5 MB)

IX.118. Daca x, y, z ∈ R∗+, demonstrati ca are loc inegalitateaX

cyclic

√xy

x+ y≥ 2

Xcyclic

z√xy

(x+ z)(y + z).

Marian Tetiva, Barlad

IX.119. Fie P un punct ın interiorul triunghiului echilateral ABC si P1, P2, P3

proiectiile lui pe laturile triunghiului. Daca G este centrul de greutate al triunghiuluiABC, demonstrati ca mijlocul G1 al segmentului PG este centru de greutate pentrutriunghiul P1P2P3.

Petru Asaftei, Iasi

IX.120. Fie P un punct ın interiorul triunghiului ABC astfel ıncat ÕPAB ≡ÕPBC ≡ ÕPCA. Daca AB ·−→AP + BC ·

−−→BP + CA ·

−−→CP =

−→0 , aratati ca △ABC este

echilateral.Claudiu Mandrila, elev, Targoviste

Clasa a X-aX.116. O urna contine x bile rosii si n − x bile verzi, unde x, n ∈ N∗, x < n.

x

3bile rosii si

x

2bile verzi sunt marcate cu 1, iar celelalte bile sunt marcate cu 2. Din

urna se extrage o bila si se considera evenimentele A: ,,obtinem o bila rosie” si B:,,obtinem o bila marcata cu 1”.

a) Determinati n si x pentru care A si B sunt evenimente independente.b) Aflati P (B|A).

Laurentiu Modan, Bucuresti

X.117. Demonstrati ca√n+ 1−

√n > 3

√n+ 1− 3

√n, ∀n ∈ N∗.Ionut Ivanescu, Craiova

X.118. Determinati n ∈ N pentru care

sin2n x+ cos2n x+ n sin2 x cos2 x = 1, ∀x ∈ R.

Ani Draghici si Ileana Mandruleanu, Craiova

X.119. Demonstrati ca pentru orice z1, z2, z3 ∈ C are loc inegalitatea

|z1 + z2|2 + |z2 + z3|2 + |z3 + z1|2 ≥ |z1 + z2 + z3|2 + Im(z1z2 + z2z3 + z3z1).

Dan Nedeianu, Drobeta Tr. Severin

X.120. FieABCD un patrulater ınscris ın cercul C(M,R), avandm(ÕBOC) ≤ 90◦,unde {O} = AC ∩BD. Demonstrati ca

AB2 −BC2 + CD2 −DA2 ≥ AC ·BD − 4R2.

Cand se realizeaza egalitatea?Florin Stanescu, Gaesti

89

Page 94: Revista (format .pdf, 6.5 MB)

Clasa a XI-aXI.116. a) Fie α ∈ R+ si A ∈ M2(R) astfel ıncat det(αI2+A2) = 0; demonstrati

ca detA = α.b) Aratati ca exista α ∈ R∗

− si A ∈ M2(R) astfel ıncat det(αI2 + A2) = 0 sidetA = α.

Dan Popescu, Suceava

XI.117. Daca a, b ∈ R∗+, aratati ca ax ≥ ln(bx+1), ∀x ∈

�−1

b,∞�daca si numai

daca a = b.Dumitru Savulescu, Bucuresti

XI.118. Fie α1, α2, α3, α4 numere reale pentru care limita l = limx→∞

(α1{x} +

α2

§x+

1

2

ª+α3

§x+

2

3

ª+α4{3x}) este finita (unde {·} desemneaza partea fractio-

nara). Aratati ca α1 + α2 + α3 + 3α4 = 0.Marius Dragan, Bucuresti

XI.119. Se considera sirul (un)n≥0 definit prin u0 = 1, u1 = 2, un+2 = 1 +1

4arctg

un+1

un, n ∈ N∗. Aratati ca sirul este convergent si aflati-i limita.

Moubinool Omarjee, Paris

XI.120. Fie k, a, b ∈ (0,∞), a < b si sirurile (xn)n≥1, (yn)n≥1 definite prin

x1 = a, y1 = b, xn+1 =k2 + kxn + xnyn

yn, yn+1 =

k2 + kyn + xnynxn

, ∀n ∈ N∗.

a) Demonstrati ca xn <k2 + 2ka+ ab

b− a, ∀n ∈ N∗.

b) Aratati ca sirurile date au limite si determinati aceste limite.Lucian Tutescu si Mircea Tereujanu, Craiova

Clasa a XII-aXII.116. Determinati functia f : Z8 → Z8 cu proprietatea ca

f(3x) + f(5x) + f(7x) = 3x2 + 5x+ 6, ∀x ∈ Z8.

Bogdan Chiriac, student, Iasi

XII.117. Aflati n ∈ N pentru care exista α ∈ R∗− astfel ıncat

Z α

α−1(2t3+3)dt = n.

Romeo Cernat, Iasi

XII.118. Fie f : [0, 1] → R o functie continua cu proprietatea caR 10 f(x)dx =

lna+ 1

a, unde a ∈ (0, 1) este dat. Demonstrati ca exista x0 ∈ (0, 1) astfel ıncat

1

x0 + 1< f(x0) <

1

x0.

Bogdan Victor Grigoriu, Falticeni

XII.119. Demonstrati ca limn→∞

n

È(n2 + 12)(n2 + 22) · . . . · (n2 + n2)

n=

2

e2· eπ

2 .

Adrian Corduneanu, Iasi

90

Page 95: Revista (format .pdf, 6.5 MB)

XII.120. Fie f : R → (0,∞) o functie continua cu limx→∞

f(x) = 0, astfel ıncat

oricare ar fi n ∈ N∗, exista un unic xn ∈ R∗+ pentru care

Z xn

0

ln(1 + f(t))

f(t)dt =Z n

0

sin f(t)

f(t)dt. Calculati lim

n→∞

xn

n.

Florin Stanescu, Gaesti

Probleme pentru pregatirea concursurilor

A. Nivel gimnazial

G196. Fie M multimea numerelor naturale nenule scrise numai cu cifre pare, careau cel mult 2011 cifre. Aratati ca suma inverselor elementelor lui M este mai micadecat 4.

Cecilia Deaconescu, Pitesti

G197. Determinati cea mai mare putere a lui 3 care divide numarul N = 162011−2 · 82011 + 3 · 42011 − 2 · 22011 + 1.

Pedro H.O. Pantoja, Brazil

G198. Rezolvati ın numere naturale ecuatia 6n + 2800 = m6.Andrei Eckstein, Timisoara

G199. Determinati b ∈ N∗ pentru care exista a ∈ N∗ astfel ıncat a2 + ab+ b2 safie patrat perfect.

Gheorghe Iurea, Iasi

G200. Aratati caa2

2b3 + 2c3 + 5+

b2

2a3 + 2c3 + 5+

c2

2a3 + 2b3 + 5≤ 1

3, ∀a, b, c ∈

[0, 1].Dan Nedeianu, Drobeta Tr. Severin

G201. Se considera triunghiul ABC cu AC = BC > AB. Daca exista un punct

D ∈ (BC) pentru care AB2 = BD · BC si AD2 = BD · DC, aratati ca −1

3≤

AB2

BC2 −AC2≤ 1.

Ovidiu Pop, Satu Mare

G202. Fie ABCD patrat, iar punctele M si N pe laturile AB, respectiv ADsunt astfel ıncat AM = DN = k · AB. Notam {P} = CN ∩ DM si {Q} = AP ∩CD. Determinati valorile lui k pentru care PQ este bisectoare, respectiv mediana ıntriunghiul CDP.

Neculai Roman, Mircesti (Iasi)

G203. Fie date numerele reale a si b cu a < b < 2a. Triunghiurile isosceleABC si A′B′C ′ au aceeasi axa de simetrie, aceeasi dreapta suport a bazelor, iarBC = a, AB = AC = b, B′C ′ = b, A′B′ = A′C ′ = a. Daca {M} = AB ∩ A′B′,

91

Page 96: Revista (format .pdf, 6.5 MB)

{N} = AC ∩ A′C ′, aratati ca MN este linie mijlocie ın △ABC daca si numai dacaa3 + b3 = 2a2b.

Temistocle Bırsan, Iasi

G204. Despre un punct de pe o muchie a unui tetraedru vom spune ca este punctbisector daca el este piciorul a doua bisectoare ale unor unghiuri ale fetelor. Aratatica numarul punctelor bisectoare ale unui tetraedru este 0, 2 sau 6.

Silviu Boga, Iasi

G205. Doua panouri luminoase sunt situate ın plane paralele (verticale). Ele auforma a doua dreptunghiuri identice, ımpartite fiecare ın cate zece patrate congruentecu ajutorul cate unei linii orizontale si a cate patru linii verticale. In cele 18 varfuri depatrate care se formeaza pe fiecare panou sunt instalate beculete. La un moment datpe fiecare panou se aprind cate doua beculete, la ıntamplare. Care este probabilitateaca cele patru beculete aprinse sa se afle ıntr-un acelasi plan?

Gabriel Popa si Cristian Lazar, Iasi

B. Nivel liceal

L196. Demonstrati ca ın orice triunghi ascutitunghic are loc inegalitatea

(ctgA+ ctgB + ctgC)3

(ctgA+ ctgB)(ctgB + ctgC)(ctgC + ctgA)≥ 8

3√3

(1 + cosA cosB cosC)3

sinA sinB sinC.

Gheorghe Costovici, Iasi

L197. Fie ABCDEFGH un paralelipiped dreptunghic, iar S o sfera prin A careintersecteaza segmentele AB,AD,AE si AG ın M,N,P , respectiv Q. Aratati caAM ·AB +AN ·AD +AP ·AE = AQ ·AG.

Claudiu Stefan Popa, Iasi

L198. Fie ABC un triunghi ascutitunghic ınscris ın cercul C. Cercul C′ estetangent cercului C ın punctul A si laturii BC ın punctul D. Aratati ca AD este

bisectoarea unghiului ÕBAC.Titu Zvonaru, Comanesti

L199. Fie O centrul cercului circumscris triunghiului ABC si {M} = OB ∩ AC,{N} = OC∩AB. Daca OM = ON, aratati ca triunghiul este isoscel sau dreptunghic.

Temistocle Bırsan, Iasi

L200. In raport cu un reper cartezian xOy, se considera punctele M

�3

2, 1

�,

B

�3a− 2

a, 0

�si C

�0,

2a+ 3

a

�, unde a ∈ R\

§0,

2

3,−3

2

ª, precum si familia de drepte

dm : y = mx+2− 3m

2, m ∈ R. Notam cu Ca cercul circumscris triunghiului OBC.

a) Demonstrati ca, pentru orice m ∈ R, dreapta dm intersecteaza Ca ın douapuncte distincte P si Q.

b) Aratati ca produsul MP ·MQ este independent de a si m.Gabriel Popa si Paul Georgescu, Iasi

92

Page 97: Revista (format .pdf, 6.5 MB)

L201. Demonstrati ca pentru orice numar prim p > 22k + 1, ∀k ∈ N∗, numarul

p22k+1 − 1 se divide cu 22(k+1) ·

mQj=1

qj , unde {qj |j = 1,m} este multimea numerelor

prime din multimea {2i + 1|i = 1, 2k}.Neculai Roman, Mircesti (Iasi)

L202. Determinati numerele reale a si b pentru care a

√n+ 2

n+ 1≤

√n+ 2 −

√n− 1 ≤ b

√n+ 1

n, ∀n ∈ N∗.

Gheorghe Iurea, Iasi

L203. Fie f, g polinoame cu coeficientii reali, nu ambele constante, iar P =f + ig ∈ C[X]. Presupunem ca radacinile lui P sunt numere complexe cu partileimaginare strict negative. Daca λ, µ ∈ R, λ2+µ2 = 0, aratati ca radacinile polinomuluiQ = λf + µg sunt reale.

Adrian Reisner, Paris

L204. Fie A o matrice patratica de ordinul n avand elementele aij din multimea{0, 1} si urmatoarele proprietati: i) aii = 0 pentru orice i ∈ {1, 2, . . . , n}; ii) dacaaij = 1 (pentru i = j din multimea {1, 2, . . . , n}), atunci aji = 0; iii) pentru fiecarep ∈ {0, 1, . . . , n− 1}, matricea are o linie pe care se afla exact p elemente egale cu 1.

Sa se arate ca exista o permutare a multimii {1, 2, . . . , n} astfel ıncat, daca seaplica aceasta permutare liniilor matricei A si apoi coloanelor matricei astfel obtinute,rezulta ın final o matrice cu toate elementele care sunt egale cu 1 situate deasupradiagonalei principale. Care este polinomul caracteristic al unei asemenea matrice?

Marian Tetiva, Barlad

L205. Calculati∞Xk=1

(−1)k−1 ln k

k.

Marian Tetiva, Barlad

Training problems for mathematical contests

A. Junior highschool level

G196. Let M be the set of nonzero natural numbers consisting of even digitsonly, and having at most 2011 digits. Show that the sum of the inverses of elementsin M is less that 4.

Cecilia Deaconescu, Pitesti

G197. Find the highest power of 3 which divides the number N = 162011 − 2 ·82011 + 3 · 42011 − 2 · 22011 + 1.

Pedro H.O. Pantoja, Brazil

G198. Solve, in the set of natural numbers, the equation 6n + 2800 = m6.Andrei Eckstein, Timisoara

G199. Find b ∈ N∗ such that there exists a ∈ N∗ with the property that a2 +ab+ b2 is a perfect square.

Gheorghe Iurea, Iasi

93

Page 98: Revista (format .pdf, 6.5 MB)

G200. Show thata2

2b3 + 2c3 + 5+

b2

2a3 + 2c3 + 5+

c2

2a3+2b3 + 5≤1

3, ∀a, b, c ∈ [0, 1].

Dan Nedeianu, Drobeta Tr. Severin

G201. Let ABC a triangle with AC = BC > AB. If there exists a point D ∈

(BC) such AB2 = BD ·BC and AD2 = BD ·DC, show that −1

3≤ AB2

BC2 −AC2≤ 1.

Ovidiu Pop, Satu Mare

G202. Let ABCD be a square, and the points M and N situated on the sidesAB, respectively AD, such that AM = DN = k ·AB. Let us denote {P} = CN∩DMand {Q} = AP ∩ CD. Determine the values of the coefficient k for which PQ is anangle-bisector line, respectively a side-bisector in the triangle CDP.

Neculai Roman, Mircesti (Iasi)

G203. Let us consider the real numbers a and b with a < b < 2 a. The isoscelestriangles ABC si A′B′C ′ have the same axis of symmetry, the same support line oftheir bases and their side lengths are BC = a, AB = AC = b, B′C ′ = b, A′B′ =A′C ′ = a. If {M} = AB ∩A′B′, {N} = AC ∩A′C ′, show that MN midpoint line in△ABC if and only if a3 + b3 = 2a2b.

Temistocle Bırsan, Iasi

G204. A point situated on the edge of a tetrahedron is said to be a bisector pointif it is the intersection point of the angle-bisector lines of two angles of two triangularsides. Show that the number of bisector points of a tetrahedron can be 0, 2 or 6.

Silviu Boga, Iasi

G205. Two bright panels are situated in (vertical) parallel planes. They have theshapes of two identical rectangles, each of them divided into ten congruent squaresby means of a horizontal line and four vertical lines. A bulb is installed at each ofthe 18 corners of the squares resulting from this division. At a certain moment, twobulbs randomly shine on each panel. Which is the probability that the four shiningbulbs lie in the same plane?

Gabriel Popa and Cristian Lazar, Iasi

B. Highschool LevelL196. Show that the following inequality holds in any acute-angled triangle:

(ctgA+ ctgB + ctgC)3

(ctgA+ ctgB)(ctgB + ctgC)(ctgC + ctgA)≥ 8

3√3

(1 + cosA cosB cosC)3

sinA sinB sinC.

Gheorghe Costovici, Iasi

L197. Let ABCDEFGH be a right-angled parallelepiped, and S a sphere passingthrough point A that intersects the line segments AB,AD,AE and AG at M,N,P ,respectively Q. Show that AM ·AB +AN ·AD +AP ·AE = AQ ·AG.

Claudiu Stefan Popa, Iasi

L198. Let ABC be an acute-angled triangle inscribed in the circle C. The circleC′ is tangent to the circle C at point A and also to the side BC at point D. Show

that AD is the angle-bisector line of angle ÕBAC.Titu Zvonaru, Comanesti

94

Page 99: Revista (format .pdf, 6.5 MB)

L199. Let O be the center of the circumcircle of triangle ABC and {M} =OB ∩ AC, {N} = OC ∩ AB. If OM = ON, show that the triangle is either isoscelesor right-angled.

Temistocle Bırsan, Iasi

L200. With respect to a Cartesian system of coordinates xOy, three points

are considered, namely M

�3

2, 1

�, B

�3a− 2

a, 0

�and C

�0,

2a+ 3

a

�, where a ∈

R\§0,

2

3,−3

2

ª, as well as the family of lines dm : y = mx+

2− 3m

2, m ∈ R. Denote

by Ca the circumcircle of the triangle OBC.a) Prove that, for any m ∈ R, the line dm intersects Ca at two distinct points P

and Q.b) Show that the product MP ·MQ is independent of both a and m.

Gabriel Popa and Paul Georgescu, Iasi

L201. Prove that, for any prime number p > 22k + 1, ∀k ∈ N∗, the number

p22k+1 − 1 is divisible by 22(k+1) ·

mQj=1

qj , where {qj | j = 1,m} is the set of the prime

numbers included in the set {2i + 1| i = 1, 2k}.Neculai Roman, Mircesti (Iasi)

L202. Find the real numbers a and b such that

a

√n+ 2

n+ 1≤

√n+ 2−

√n− 1 ≤ b

√n+ 1

n, ∀n ∈ N∗.

Gheorghe Iurea, Iasi

L203. Let f, g be two polynomials with real coefficients, not both of them con-stant, and P = f + ig ∈ C[X]. Assume that the roots of P are complex numbers withstrictly negative imaginary parts. If λ, µ ∈ R, λ2 +µ2 = 0, show that the roots of thepolynomial Q = λf + µg are real.

Adrian Reisner, Paris

L204. Let A be a square matrix of order n having its entries aij in the set {0, 1},also enjoying the following properties: i) aii = 0 for any i ∈ {1, 2, . . . , n}; ii) if aij = 1(for i = j in the set {1, 2, . . . , n}), then aji = 0; iii) for any p ∈ {0, 1, . . . , n− 1}, thematrix has a row containing exactly p entries equal to 1.

Show that a permutation of the set {1, 2, . . . , n} exists such that, if this permuta-tion is applied to the rows of matrix A and the same permutation is then applied tothe matrix just obtained, it is finally obtained a matrix whose all entries equal to 1are situated above the main diagonal. Which is the characteristic polynomial of sucha matrix?

Marian Tetiva, Barlad

L205. Calculate∞Xk=1

(−1)k−1 ln k

k.

Marian Tetiva, Barlad

95

Page 100: Revista (format .pdf, 6.5 MB)

Pagina rezolvitorilor

CRAIOVAColegiul National ”Fratii Buzesti”. Clasa a VI-a (prof. IONESCU Maria).

VIRLAN Leonard: P(192,193), V(116,118-120), VI(116,117), VII.116. Clasa a VIII-a (prof. TUTESCU Lucian). ENE Cristina: VII(124-126), VIII.125, IX(111,112).Clasa a IX-a (prof. TUTESCU Lucian). GOLEA Monica: VIII(124,125,127),IX.111, XI.111.

IASIScoala nr. 3 ”Al. Vlahuta”. Clasa a II-a (inst. MAXIM Gabriela). CARACAS

Sorin: P(196-198,200,203); DASCALU Lorena: P(196-198,200,203); NICA Ioan-Da-niel: P(196-198,200,203); ROBU Carmen: P(196-198,200,203); SERBANOIU Alexan-dru: P(196-198,200,203). Clasa a IV-a (ınv. MARIUTA Valentina). ENEA CodrutAlexandru: P(192,196-198,201,203,204); LUNGU Lucian: P(196-198,201,203,204);HERGHILIGIU-HENEA Malina: P(192,196-198,201,203,204); HREAPCA Alin:P(192,196-198,201,203,204); HUHU Paula: P(192,196,197,201,203,204); PERES Cris-ti: P(192,196-198,201,203,204); POPOVICI Teodor-Andrei: P(196-198,201,203,204);ROMILA Andreea-Maria: P(192,196-198,201,203,204). Clasa a V-a (prof. MARINMirela). CIOCOIU Alexandra: P(203-205), V(125,127); MARIN Marius: P(203-205), V(126,127). Clasa a VI-a (prof. MARIN Mirela). CRETU Cristiana-Paula:V(125,126), VI(124,127,129); IFTIME Ioana Evelina: V(124-126,129), VI(124,125,127); SOFIAN Marina: V(126,129), VI(123,124,127). Clasa a VIII-a (prof. MARINMirela). ASAVEI Alexandra: VI(123,124,127), VII(126-128), VIII.124; CELMARERaluca-Iuliana: VI(123-125,127), VII(123,126-128), VIII(123,124,127); MARCUAnca:VI(123,124), VII(123,126,128), VIII(123,124,127); TIBA Stefana-Alexandra: VI(123,124), VII(126,128), VIII(123,124,127).

Scoala nr. 11 ”Otilia Cazimir”. Clasa a III-a (inst. MARDARE CarmenGabriela). OLINIUC Iulian: P(184-188). Clasa a IV-a (inst. PARAIALA Du-mitru). POPA Ioana-Maria: P(186-195).

Scoala nr. 13 ”Alexandru cel Bun”. Clasa a III-a (inst. COJOCARIU Ana).ACATRINEI Andra: P(196-200); BEJANMatei: P(196-200); BONCU Tudor: P(196-200); BULEI Iasmina-Ioana: P(196-200); COSTIN Mihaita-Alexandru: P(196-200);MUNTIANU Ioana-Andreea: P(196-200); PERDUN Patricia-Maria: P(196-200);PRISECARU Alexandru: P(196-200); SAMSON Constantin-Catalin: P(196-200);STRATILA Diana-Liliana: P(196-200); STEFAN Tudor: P(196-200); ZAHARIAStefan-Eusebiu: P(196-200).

Scoala nr. 26 ”George Cosbuc”. Clasa a II-a (inst. RACU Maria). CIOPEICASebastian-Andrei: P(196-200); GROSU Victor-Alessandru: P(196-200); LUCHIANMaria-Clara: P(196-200); PASNICU Cosmin-Constantin: P(196-200); POPESCUAndrei-Eduard: P(196-200); RAILEANU Razvan-Constantin: P(196-200). Clasa aIV-a (inst. VARLAN Elena). AMARIEI Romeo: P(196-199,202,204); GHEBAN An-dreea: P(196-199,202,204); PAVALUC Ana-Maria: P(196-199,202,204); TATARUAlice: P(196-199,202,204); TIPLEA Iulian: P(196-199,202,204).

96

Page 101: Revista (format .pdf, 6.5 MB)

Colegiul ”National”. Clasa a V-a (prof. POPA Gabriel). BAHRIM Antonia:P(203-205), V(124,126,127); GANTATUC Bogdan: P(192-194,204,205), V(118,119,125-127); GEAMBEC Lavinia: P(192,193,195,203-205), V(116,123,126,127). Clasa aVIII-a (prof. POPA Gabriel). ABUNEI Radu: VI(123-125), VII(125,126), VIII.125;CIOBANU Stefan: VI(123,124), VII(126-128), VIII.125; MURGU George: VI(123,124,128), VII.123, VIII(124,125); PLUGARU Daniel: VI(123,124), VII(124,126-128),VIII.125; POPOVICI Smaranda: VI(123,124), VII(124,126-128), VIII.125; PURICEDumitru: VI(123,124), VII(124,126-128), VIII.125; STOLERU Ingrid: VI.126,VII(123,124,126), VIII(123-125,127-129), IX.111.

PASCANIColegiul National ”Mihail Sadoveanu”. Clasa a IX-a (prof. CRACIUN Mihai).

VOLINTIR Roxana: VII(125-127), VIII(123,125,126), IX.111.

TIGANASI (IASI)Scoala cu clasele I-VIII ”M. Kogalniceanu”. Clasa a IV-a (ınv. GALIA Pa-

raschiva). CAZADOI Ioana-Cristina: P(196-198,200,203); DUCA Mihaela-Cristina:P(196-198,200,203); DUCA Vasile-Ilie: P(196-198,200,203); ILIOAE Mihai-Iulian:P(196-198,200,203); SANDU Rebeca: P(196-198,200,203). Clasa a V-a (prof. CHI-RITESCU Anca). GHIOANCA Bianca-Maria: P(203,204), V(125,126), VI.124; PIUDiana Raluca: P(203-205), V.126, VI.123; TOMA Evelina: P(203-205), V(125,126),VI.124. Clasa a VI-a (prof. CHIRITESCU Anca). DUCA Mirela Beatrice: P(203-205), VI(123,124); GANEANU Tudor: P(203-205), VI(123,124); VERNER MadalinaGeorgiana: P(203-205), VI(123,124). Clasa a VII-a (prof. CHIRITESCU Anca).BOROS Paula Mihaela: V(124,126), VI(123,124), VII.126; DUCA Liliana Daniela:V(124-126), VI(123,124), VII.126; IACOB Oana Alisa: V(124-126), VI(123,124),VII.126; PIU Debora ROXANA: V(124-126), VI(123,124), VII.126; PIU Mioara Flo-rentina: V(124-126), VI(123,124), VII.126; RUSU Andrei: V(124,125), VI(123,124),VII.126; TANASE Madalina: V(124-126), VI(123,124), VII.126; TIHULCA Catalina:V(124-126), VI(123,124), VII.126. Clasa a VIII-a (prof. CHIRITESCU Anca).GAVRILAS Marius Alexandru: VI(123,124), VII(124,126), VIII.124; GANEANUStefan Bogdan: VI(123,124), VII(124,126), VIII.124; GHIOANCA Andreea Lavinia:VI(123,124), VII(124,126), VIII.124; GHIOANCA Claudia Oana: VI(123,124),VII(124,126), VIII.124; PIU Mihaita: VI(123,124), VII(124,126), VIII.124.

Elevi rezolvitori premiati

Colegiul National ”Fratii Buzesti”, Craiova

1. ENE Cristina (cl. a VII-a): 1/2009(5pb), 1/2010(5pb), 1/2011(6pb).

Scoala nr. 3 ”Al. Vlahuta”

1. NICA Daniel (cl. a III-a): 2/2009(7pb), 1/2010(5pb); 1/2011(5pb).

97

Page 102: Revista (format .pdf, 6.5 MB)

2. HERGHILIGIU HENEA-Malina (cl. a IV-a): 2/2009(7pb), 2/2010(6pb),1/2011(7pb).

3. HREAPCA Alin (cl. a IV-a): 2/2009(7pb), 2/2010(6pb), 1/2011(7pb).

4. HUHU Paula (cl. a IV-a): 2/2009(7pb), 2/2010(6pb), 1/2011(6pb).

5. POPOVICI Teodor-Andrei (cl. a IV-a): 2/2009(7pb), 2/2010(6pb),1/2011(6pb).

6. ROMILA Andreea-Maria (cl. a IV-a): 2/2009(7pb), 2/2010(6pb), 1/2011(7pb).

Scoala nr. 11 ”Otilia Cazimir”

1. POPA Ioana-Maria (cl. a IV-a): 1/2010(13pb), 2/2010(12pb), 1/2011(10pb).

Scoala nr. 13 ”Alexandru cel Bun”

1. BEJAN Matei (cl. a III-a): 1/2010(5pb), 2/2010(5pb), 1/2011(5pb).

2. BULEI Iasmina-Ioana (cl. a III-a): 1/2010(5pb), 2/2010(5pb), 1/2011(5pb).

3. COSTIN Mihaita-Catalin (cl. a III-a): 1/2010(5pb), 2/2010(5pb), 1/2011(5pb).

4. MUNTIANU Ioana-Andreea (cl. a III-a): 1/2010(5pb), 2/2010(5pb),1/2011(5pb).

Scoala nr. 26 ”George Cosbuc”

1. PAVALUC Ana-Maria (cl. a IV-a): 2/2009(8pb), 2/2010(7pb), 1/2011(6pb).

Scoala cu clasele I-VIII, ”M. Kogalniceanu”, Tiganasi (Iasi)

1. DUCA Mirela Beatrice (cl. a VI-a): 1/2010(5pb), 2/2010(5pb), 1/2011(5pb).

2. GANEANU Tudor (cl. a VI-a): 1/2010(6pb), 2/2010(5pb), 1/2011(5pb).

3. VERNER Madalina Georgiana (cl. a VI-a): 1/2010(7pb), 2/2010(6pb),1/2011(5pb).

4. BOROS Paula Mihaela (cl. a VII-a): 1/2010(6pb), 2/2010(5pb), 1/2011(5pb).

5. DUCA Liliana Daniela (cl. a VII-a): 1/2010(7pb), 2/2010(7pb), 1/2011(6pb).

6. PIU Debora Roxana (cl. a VII-a): 1/2010(6pb), 2/2010(6pb), 1/2011(6pb).

7. GAVRILAS Marius Alexandru (cl. a VIII-a): 1/2010(5pb), 2/2010(5pb),1/2011(5pb).

8. GANEANU Stefan Bogdan (cl. a VIII-a): 1/2010(5pb), 2/2010(5pb),1/2011(5pb).

9. GHIOANCA Claudia Oana (cl. a VIII-a): 1/2010(5pb), 2/2010(5pb),1/2011(5pb).

98

Page 103: Revista (format .pdf, 6.5 MB)

Revista semestrială RECREAŢII MATEMATICE este editată de ASOCIAŢIA “RECREAŢII MATEMATICE”. Apare la datele de 1 martie şi 1 septembrie şi se adresează elevilor, profesorilor, studenţilor şi tuturor celor pasionaţi de matematica elementară.

În atenţia tuturor colaboratorilor Materialele trimise redacţiei spre publicare (note şi articole, chestiuni de

metodică, probleme propuse etc.) trebuie prezentate îngrijit, clar şi concis; ele trebuie să prezinte interes pentru un cerc cât mai larg de cititori. Se recomandă ca textele să nu depăşească patru pagini. Evident, ele trebuie să fie originale şi să nu fi apărut sau să fi fost trimise spre publicare altor reviste. Rugăm ca mate-rialele tehnoredactate să fie însoţite de fişierele lor (trimise la adresa de mai jos).

Problemele destinate rubricilor: Probleme propuse şi Probleme pentru pregătirea concursurilor vor fi redactate pe foi separate cu enunţ şi demonstra-ţie/rezolvare (câte una pe fiecare foaie) şi vor fi însoţite de numele autorului, şcoa-la şi localitatea unde lucrează/învaţă.

Redacţia va decide asupra oportunităţii publicării materialelor primite. În atenţia elevilor Numele elevilor ce vor trimite redacţiei soluţii corecte la problemele din

rubricile de Probleme propuse şi Probleme pentru pregatirea concursurilor vor fi menţionate în Pagina rezolvitorilor. Elevii vor ţine seama de regulile:

1. Pot trimite soluţii la minimum cinci probleme propuse în ultimul număr al revistei. Pe o foaie va fi redactată soluţia unei singure probleme şi vor fi menţionate datele: numele şi prenumele, clasa, profesorul, şcoala şi localitatea.

2. Elevii din clasele VI-XII au dreptul să trimită soluţii la problemele propuse pentru clasa lor, pentru orice clasă mai mare, din două clase mai mici şi imediat anterioare. Elevii din clasa a V-a pot trimite soluţii la problemele propuse pentru clasele a IV-a, a V-a şi orice clasă mai mare, iar elevii claselor I-IV pot trimite soluţii la problemele propuse pentru oricare din clasele primare şi orice cla-să mai mare. Orice elev poate trimite soluţii la problemele de concurs (tip G şi L).

3. Oricine poate trimite soluţii la problemele de tip L; la publicare, soluţiile acestora vor fi urmate de numele tuturor rezolvitorilor.

4. Plicul cu probleme rezolvate se va trimite prin poştă (sau va fi adus direct) la adresa Redacţiei:

Prof. dr. Temistocle Bîrsan

Str. Aurora, nr. 3, sc. D, ap. 6, 700 474, Iaşi

Jud. IAŞI E-mail: [email protected]

Page 104: Revista (format .pdf, 6.5 MB)

CUPRINS

Un secol de excelenţă a Învăţământului Superior Electrotehnic la Iaşi (Al. SĂLCEANU) . 1 Restaurarea mormântului lui C. Climescu, fost rector al Universităţii din Iaşi (T. BÎRSAN) .................... 3

ARTICOLE ŞI NOTE D. MIHEŢ – Condiţii suficiente pentru inele Boole ................................................................ 5 M. DICU – Asupra calculării unor integrale definite............................................................... 7 N. BOURBĂCUŢ – O identitate cu numere complexe şi consecinţele sale geometrice........ 11

NOTA ELEVULUI F. TOMA – Proprietăţi caracteristice ale triunghiului echilateral .......................................... 17

CORESPONDENŢE F. SMARANDACHE – A Group-Permutation Algorithm to Solve the Generalized SUDOKU ................ 20 A. REISNER – Graphes et matrices de Moore ...................................................................... 23

CHESTIUNI METODICE M.-D. PANŢIRUC – Asupra unor probleme de desfăşurare.................................................. 28

CUM CONCEPEM . . . CUM REZOLVĂM M. TETIVA – Despre numere, corpuri de numere şi căutarea esenţei................................... 31

DIN ISTORIA MATEMATICII E. ROGAI, L. MODAN, M. ROGAI – File din istoria revistei „Curierul Matematic”...... 35

ŞCOLI ŞI DASCĂLI M. NEGREA – Emil Briul – un dascăl de altădată .............................................................. 40

CONCURSURI ŞI EXAMENE Concursul "Recreaţii Matematice", ed. a VIII-a, 2010....................................................... 42 Concursul de matematică "Gaudeamus", ed. a II-a, 2010 .................................................... 44 Concursul "Speranţe Olimpice", ed. a X-a, 2010 ................................................................... 45

PROBLEME ŞI SOLUŢII Soluţiile problemelor propuse în nr. 1/2010.......................................................................... 49 Soluţiile problemelor pentru pregătirea concursurilor din nr. 1/2010 ................................. 65 Soluţiile problemelor pentru pregătirea concursurilor din nr. 2/2010 ................................. 73 Probleme propuse..................................................................................................................... 85 Probleme pentru pregătirea concursurilor .............................................................................. 91 Training problems for mathematical contests ....................................................................... 93

Pagina rezolvitorilor .............................................................................................................. 96 Elevi rezolvitori premiaţi........................................................................................................ 97 ISSN 1582 – 1765 8 lei